summaryrefslogtreecommitdiffstats
diff options
context:
space:
mode:
-rw-r--r--controle-20190408.tex734
-rw-r--r--controle-2020qcm.tex1332
-rw-r--r--controle-20210412.tex887
-rw-r--r--controle-20220413.tex441
-rw-r--r--controle-20230417.tex793
-rwxr-xr-xmisc/randomize-test.pl183
-rw-r--r--notes-mitro206.tex911
-rw-r--r--sample-2020qcm.tex335
8 files changed, 5327 insertions, 289 deletions
diff --git a/controle-20190408.tex b/controle-20190408.tex
new file mode 100644
index 0000000..6e4da38
--- /dev/null
+++ b/controle-20190408.tex
@@ -0,0 +1,734 @@
+%% This is a LaTeX document. Hey, Emacs, -*- latex -*- , get it?
+\documentclass[12pt,a4paper]{article}
+\usepackage[francais]{babel}
+\usepackage[utf8]{inputenc}
+\usepackage[T1]{fontenc}
+%\usepackage{ucs}
+\usepackage{times}
+% A tribute to the worthy AMS:
+\usepackage{amsmath}
+\usepackage{amsfonts}
+\usepackage{amssymb}
+\usepackage{amsthm}
+%
+\usepackage{mathrsfs}
+\usepackage{wasysym}
+\usepackage{url}
+%
+\usepackage{graphics}
+\usepackage[usenames,dvipsnames]{xcolor}
+\usepackage{tikz}
+\usetikzlibrary{matrix,calc}
+\usepackage{hyperref}
+%
+%\externaldocument{notes-mitro206}[notes-mitro206.pdf]
+%
+\theoremstyle{definition}
+\newtheorem{comcnt}{Tout}
+\newcommand\thingy{%
+\refstepcounter{comcnt}\smallskip\noindent\textbf{\thecomcnt.} }
+\newcommand\exercice{%
+\refstepcounter{comcnt}\bigskip\noindent\textbf{Exercice~\thecomcnt.}\par\nobreak}
+\renewcommand{\qedsymbol}{\smiley}
+%
+\newcommand{\outnb}{\operatorname{outnb}}
+\newcommand{\downstr}{\operatorname{downstr}}
+\newcommand{\precs}{\operatorname{precs}}
+\newcommand{\mex}{\operatorname{mex}}
+\newcommand{\id}{\operatorname{id}}
+\newcommand{\limp}{\Longrightarrow}
+\newcommand{\gr}{\operatorname{gr}}
+\newcommand{\rk}{\operatorname{rk}}
+\newcommand{\fuzzy}{\mathrel{\|}}
+%
+\DeclareUnicodeCharacter{00A0}{~}
+%
+\DeclareMathSymbol{\tiret}{\mathord}{operators}{"7C}
+\DeclareMathSymbol{\traitdunion}{\mathord}{operators}{"2D}
+%
+\DeclareFontFamily{U}{manual}{}
+\DeclareFontShape{U}{manual}{m}{n}{ <-> manfnt }{}
+\newcommand{\manfntsymbol}[1]{%
+ {\fontencoding{U}\fontfamily{manual}\selectfont\symbol{#1}}}
+\newcommand{\dbend}{\manfntsymbol{127}}% Z-shaped
+\newcommand{\danger}{\noindent\hangindent\parindent\hangafter=-2%
+ \hbox to0pt{\hskip-\hangindent\dbend\hfill}}
+%
+\newcommand{\spaceout}{\hskip1emplus2emminus.5em}
+\newif\ifcorrige
+\corrigetrue
+\newenvironment{corrige}%
+{\ifcorrige\relax\else\setbox0=\vbox\bgroup\fi%
+\smallbreak\noindent{\underbar{\textit{Corrigé.}}\quad}}
+{{\hbox{}\nobreak\hfill\checkmark}%
+\ifcorrige\par\smallbreak\else\egroup\par\fi}
+%
+%
+%
+\begin{document}
+\ifcorrige
+\title{MITRO206\\Contrôle de connaissances — Corrigé\\{\normalsize Théories des jeux}}
+\else
+\title{MITRO206\\Contrôle de connaissances\\{\normalsize Théories des jeux}}
+\fi
+\author{}
+\date{8 avril 2019}
+\maketitle
+
+\pretolerance=8000
+\tolerance=50000
+
+\vskip1truein\relax
+
+\noindent\textbf{Consignes.}
+
+Les exercices sont totalement indépendants. Ils pourront être traités
+dans un ordre quelconque, mais on demande de faire apparaître de façon
+très visible dans les copies où commence chaque exercice.
+
+La longueur du sujet ne doit pas effrayer : d'une part, l'énoncé est
+long parce que des rappels ont été faits et que la rédaction des
+questions cherche à éviter toute ambiguïté ; d'autre part, il ne sera
+pas nécessaire de tout traiter pour obtenir la totalité des points
+(cf. barème).
+
+\medbreak
+
+L'usage de tous les documents (notes de cours manuscrites ou
+imprimées, feuilles d'exercices, livres) est autorisé.
+
+L'usage des appareils électroniques est interdit.
+
+\medbreak
+
+Durée : 2h
+
+Barème \emph{indicatif} : exercice 1 : $9$ points ; exercice 2 :
+$9$ points ; exercice 3 : $4$ points.
+
+\ifcorrige
+Ce corrigé comporte 10 pages (page de garde incluse).
+\else
+Cet énoncé comporte 6 pages (page de garde incluse).
+\fi
+
+\vfill
+{\noindent\tiny
+\immediate\write18{sh ./vc > vcline.tex}
+Git: \input{vcline.tex}
+\immediate\write18{echo ' (stale)' >> vcline.tex}
+\par}
+
+\pagebreak
+
+
+%
+%
+%
+
+\exercice
+
+On s'intéresse dans cet exercice à des jeux à deux joueurs (que l'on
+appellera Alice et Bob) de la forme suivante :
+\begin{itemize}
+\item Alice et Bob sont autour d'une table sur laquelle se trouvent un
+ certain nombre (fini) de \emph{jetons} ; chaque jeton porte un
+ entier naturel qu'on appellera son \emph{type} ; il peut y avoir
+ plusieurs jetons de même type (par exemple, trois jetons de
+ type $0$, deux jetons de type $1$ et un jeton de type $1729$) ; le
+ nombre (fini) de jetons de jetons de chaque type constitue l'état du
+ jeu, et il est visible de tous ;
+\item Alice et Bob jouent tour à tour, et chacun, quand vient son
+ tour, doit retirer un jeton de la table et le \emph{remplacer}
+ éventuellement par des jetons de type(s) strictement plus petit(s) :
+ les règles exactes de remplacement seront différentes d'une question
+ à l'autre, mais prendront toujours la forme « un joueur peut
+ remplacer un jeton de type $n$ par telle ou telle combinaison de
+ jetons de types $<n$ » ; dans tous les cas, un coup consiste à
+ retirer un unique jeton et à en poser éventuellement plusieurs ; il
+ y aura toujours au moins une manière de remplacer un jeton de
+ type $n$ donné ;
+\item il n'y a aucune interaction entre les jetons, c'est-à-dire que
+ les remplacements permis pour un jeton de type $n$ ne dépendront que
+ de $n$ et pas des autres jetons présents sur la table (ni de
+ l'identité du joueur, ni du numéro du coup, ni de quelque autre
+ information) ;
+\item on notera que les jetons de type $0$ ne peuvent être que retirés
+ (il n'y a aucun remplacement possible puisqu'il n'existe pas de
+ jeton de type $<0$) ;
+\item le gagnant est celui qui retire le dernier jeton (puisque son
+ adversaire ne peut plus jouer).
+\end{itemize}
+
+\smallskip
+
+Dans les questions (1) à (3), on ne fait pas d'hypothèse particulière
+sur les règles de remplacement autre que celles qui sont indiquées
+ci-dessus. Dans les questions (4) à (6), on traite le cas de règles
+de remplacement particulières.
+
+\medskip
+
+(1) Montrer que le jeu termine toujours en temps fini. On pourra pour
+cela associer judicieusement un ordinal à l'état du jeu et montrer
+qu'il décroît strictement à chaque tour.
+
+\begin{corrige}
+On associe à la position $J(n_1,\ldots,n_r)$ (définie en (2)
+ci-dessous), quitte à supposer $n_1>\cdots>n_r$ l'ordinal
+$\omega^{n_1} + \cdots + \omega^{n_r}$ ; ou, ce qui revient au même,
+s'il y a $r_n := \#\{i : n_i = n\}$ jetons de type $n$, on considère
+l'ordinal $\omega^N r_n + \cdots + \omega^2 r_2 + \omega\, r_1 + r_0$
+où $N := \max\{n_i\}$ est le plus grand type d'un jeton sur la table.
+Par la comparaison des ordinaux écrits en forme normale de Cantor, cet
+ordinal décroît à chaque coup (puisqu'on remplace un $\omega^n$ par
+des $\omega^{n'}$ avec $n' < n$, la suite $(r_n,\ldots,r_0)$ décroît
+lexicographiquement). Le jeu termine donc en temps fini.
+\end{corrige}
+
+\medskip
+
+(2)(a) En notant $J(n_1,\ldots,n_r)$ l'état du jeu dans lequel $r$
+jetons se trouvent sur la table et ont les types $n_1,\ldots,n_r$
+(éventuellement répétés, p.ex. $J(0,0,0,1,1,1729)$), expliquer
+pourquoi $J(n_1,\ldots,n_r)$ peut s'identifier à la somme de nim des
+positions $J(n_1),\ldots,J(n_r)$ (où $J(n)$ désigne l'état du jeu
+ayant un unique jeton de type $n$).\quad(b) En déduire la valeur de
+Grundy $\gr J(n_1,\ldots,n_r)$ de $J(n_1,\ldots,n_r)$ en fonction de
+celles des $J(n_i)$.\quad(c) Qui a une stratégie gagnante
+dans une position du type $J(n,n)$ ? Expliciter une telle stratégie
+en termes simples.
+
+(\emph{Convention :} $J()$ est le jeu nul dans lequel il ne reste plus
+aucun jeton sur la table. Notamment, on a $\gr J() = 0$. On ne le
+confondra pas avec $J(0)$ où il reste un jeton de type $0$.)
+
+\begin{corrige}
+(a) Il s'agit simplement d'observer que les différents jetons
+ n'interagissent pas du tout. Jouer à la somme de nim de
+ $J(n_1),\ldots,J(n_r)$ revient à jouer sur $r$ tables différentes à
+ partir d'un jeton de type $n_i$ sur chacune, c'est équivalent à
+ jouer à $J(n_1,\ldots,n_r)$.
+
+(b) On en déduit que $\gr J(n_1,\ldots,n_r) = \gr(J(n_1) \oplus \cdots
+ \oplus J(n_r)) = \gr J(n_1) \oplus \cdots \oplus \gr J(n_r)$ (la
+ première égalité par (a), la seconde d'après le fait que la valeur
+ de Grundy d'une somme de nim est la somme de nim des valeur de
+ Grundy).
+
+(c) Le second joueur a une stratégie gagnante à $J(n,n)$ (ceci se voit
+ par le fait que sa valeur de Grundy vaut $0$). Cette stratégie
+ consiste à reproduire systématiquement les coups de son adversaire
+ (ce qui maintiendra un nombre pair de jetons de chaque type à la fin
+ de son coup).
+\end{corrige}
+
+\medskip
+
+(3)(a) Pour une règle de remplacement donnée, expliquer comment se
+calcule $\gr J(n)$ si on suppose connus tous les $\gr J(n')$
+pour $n'<n$.\quad(b) On rappelle que le seul remplacement possible
+d'un jeton de type $0$ est de l'enlever purement et simplement : en
+déduire la valeur de $\gr J(0)$ et celle de $\gr J(0,\ldots,0)$ (où il
+y a $r$ jetons de type $0$).
+
+\begin{corrige}
+(a) Puisque $\gr x$, pour une position $x$ d'un jeu combinatoire
+ impartial bien-fondé, vaut $\mex\{\gr y : y\in\outnb(x)\}$, on a ici
+ $\gr J(n) = \mex\{\gr J(n'_1,\ldots,n'_r) : J(n'_1,\ldots,n'_r)
+ \in\outnb J(n)\}$, c'est-à-dire $\gr J(n) = \mex\{\gr J(n'_1) \oplus
+ \cdots \oplus \gr J(n'_r) : J(n'_1,\ldots,n'_r)\in\outnb J(n)\}$
+ d'après (2)(b) ; la règle de remplacement consiste justement à
+ décrire les $J(n'_1,\ldots,n'_r)\in\outnb J(n)$.
+
+(b) Dans le cas de $n=0$, comme $\outnb J(0) = \{J()\}$, on trouve
+ $\gr J(0) = \mex\{0\} = 1$, et par conséquent $\gr J(0,\ldots,0) =
+ 1\oplus \cdots\oplus 1$ vaut $0$ ou $1$ selon que $r$ est pair ou
+ impair.
+\end{corrige}
+
+\medskip
+
+(4) On suppose dans cette question que la règle de remplacement est la
+suivante : un joueur peut remplacer un jeton de type $n$ par un nombre
+quelconque (y compris $0$) de jetons de type $n-1$. (Autrement dit, à
+chaque coup, le joueur retire un jeton de type $n$ et si $n>0$ il
+ajoute ensuite, optionnellement, le nombre qu'il souhaite de jetons de
+type $n-1$. Par exemple, on peut remplacer un jeton de type $1729$
+par $42$ jetons de type $1728$ ; on peut aussi le retirer sans
+remplacement.)\quad(a) Dans ces conditions, que vaut $\gr J(n)$ ? (On
+pourra par exemple commencer par calculer $\gr J(n)$ pour $n=0,1,2,3$,
+conjecturer une formule générale, et la démontrer par récurrence
+sur $n$.)\quad(b) Exprimer de façon simple la stratégie gagnante du
+jeu considéré dans cette question.
+
+\begin{corrige}
+(a) On a vu en (3)(b) que $\gr J(0,\ldots,0)$ vaut $0$ ou $1$ selon
+ que le nombre de jetons est pair ou impair ; on en déduit que $\gr
+ J(1) = \mex\{0,1\} = 2$, et alors (en se rappelant que $2\oplus
+ 2=0$) on trouve que $\gr J(1,\ldots,1)$ vaut $0$ ou $2$ selon que le
+ nombre de jetons est pair ou impair ; on en déduit que $\gr J(2) =
+ \mex\{0,2\} = 1$, et donc $\gr J(2,\ldots,2)$ vaut $0$ ou $1$ selon
+ que le nombre de jetons est pair ou impair ; par conséquent, $\gr
+ J(3) = \mex\{0,1\} = 2$. En général on imagine facilement que $\gr
+ J(n)$ vaut $1$ ou $2$ selon que $n$ est pair ou impair, et ceci se
+ démontre par une récurrence immédiate avec exactement le même
+ argument qu'on vient de faire.
+
+(b) La valeur de Grundy de $\gr J(n_1,\ldots,n_r)$ est le XOR (= la
+ somme de nim) de $r$ valeurs $1$ si $r$ est le nombre de jetons de
+ type pair, et $r'$ valeurs $2$ si $r'$ est le nombre de jetons de
+ type impair : ce nombre vaut $0$, $1$, $2$ ou $3$ selon que $r$ et
+ $r'$ sont tous les deux pairs, que $r$ est impair et $r'$ pair,
+ qu'on a le contraire, ou qu'ils sont tous les deux impairs. La
+ stratégie gagnante consiste donc à jouer de façon que le nombre $r$
+ de jetons de type pair et le nombre $r'$ de jetons de type impair
+ soient tous les deux pairs (de façon à annuler Grundy).
+\end{corrige}
+
+\medskip
+
+(5) On suppose dans cette question que la règle de remplacement est la
+suivante : un joueur peut remplacer un jeton de type $n$ par un nombre
+quelconque (y compris $0$) de jetons de type $k<n$, le type $k$
+pouvant être quelconque mais doit être le même pour tous les jetons
+posés. (Autrement dit, à chaque coup, le joueur retire un jeton de
+type $n$ et si $n>0$ il ajoute ensuite, optionnellement, le nombre
+qu'il souhaite de jetons d'un même type $k\leq n-1$. Par exemple, on
+peut remplacer un jeton de type $1729$ par $42$ jetons de type $1728$
+ou $1728$ jetons de type $42$ ; on peut aussi le retirer sans
+remplacement.) Dans ces conditions, que vaut $\gr J(n)$ ? (On pourra
+par exemple commencer par calculer $\gr J(n)$ pour $n=0,1,2,3$,
+conjecturer une formule générale, et la démontrer par récurrence
+sur $n$.)
+
+\begin{corrige}
+On a vu en (3)(b) que $\gr J(0,\ldots,0)$ vaut $0$ ou $1$ selon que le
+nombre de jetons est pair ou impair ; on en déduit que $\gr J(1) =
+\mex\{0,1\} = 2$, et alors (en se rappelant que $2\oplus 2=0$) on
+trouve que $\gr J(1,\ldots,1)$ vaut $0$ ou $2$ selon que le nombre de
+jetons est pair ou impair ; on en déduit que $\gr J(2) = \mex\{0,1,2\}
+= 3$ (la différence avec (4)(a) est que maintenant on peut aussi aller
+en $\gr J(0,\ldots,0)$ donc $1$ apparaît dans le $\mex$), et donc $\gr
+J(2,\ldots,2)$ vaut $0$ ou $3$ selon que le nombre de jetons est pair
+ou impair ; par conséquent, $\gr J(3) = \mex\{0,1,2,3\} = 4$. En
+général on imagine facilement que $\gr J(n)$ vaut $n+1$, et ceci se
+démontre par une récurrence immédiate avec exactement le même argument
+qu'on vient de faire.
+\end{corrige}
+
+\medskip
+
+(6) On suppose dans cette question que la règle de remplacement est la
+suivante : un joueur peut remplacer un jeton de type $n$ par un nombre
+quelconque (y compris $0$) de jetons de types $<n$, qui cette fois
+n'ont pas d'obligation d'être tous du même type. (Autrement dit, à
+chaque coup, le joueur retire un jeton de type $n$ et si $n>0$ il
+ajoute ensuite, optionnellement, le nombre qu'il souhaite de jetons de
+n'importe quels types $<n$. Par exemple, on peut remplacer un jeton
+de type $1729$ par $42$ jetons de type $1728$ et $666$ de type $0$ ;
+on peut aussi le retirer sans remplacement.)\quad(a) Dans ces
+conditions, que vaut $\gr J(n)$ ? (On pourra par exemple commencer
+par calculer $\gr J(n)$ pour $n=0,1,2,3$, conjecturer une formule
+générale, et la démontrer par récurrence sur $n$.)\quad(b) Exprimer de
+façon simple la stratégie gagnante du jeu considéré dans cette
+question.
+
+\begin{corrige}
+(a) On a vu en (3)(b) que $\gr J(0,\ldots,0)$ vaut $0$ ou $1$ selon
+ que le nombre de jetons est pair ou impair ; on en déduit que $\gr
+ J(1) = \mex\{0,1\} = 2$, et alors (en se rappelant que $2\oplus
+ 2=0$) on trouve que $\gr J(0,\ldots,0,\penalty0\relax 1,\ldots,1)$
+ vaut $0$, $1$, $2$ ou $3$ selon que le nombre de jetons de chaque
+ type est pair ou impair (comme expliqué en (4)(b)) ; on en déduit
+ que $\gr J(2) = \mex\{0,1,2,3\} = 4$, et donc $\gr
+ J(0,\ldots,0,\penalty0\relax 1,\ldots,1,\penalty0\relax 2,\ldots,2)$
+ prend exactement les valeurs entre $0$ et $7$ selon la parité des
+ jetons de chaque type ; par conséquent, $\gr J(3) =
+ \mex\{0,\ldots,7\} = 8$. En général on imagine facilement que $\gr
+ J(n)$ vaut $2^n$, et ceci se démontre par une récurrence immédiate
+ en remarquant que le $\gr J(n'_1,\ldots,n'_r)$, si les $n'_i$
+ sont $<n$, peut prendre toutes les valeurs de $0$ à $2^n - 1$ selon
+ la parité des jetons de chaque type (donnant l'écriture binaire du
+ résultat).
+
+(b) La stratégie gagnante consiste donc à jouer de façon que le nombre
+ de jetons de chaque type soit pair.
+\end{corrige}
+
+
+%
+%
+%
+
+\exercice
+
+On définit une opération binaire $\alpha\boxplus\beta$ (appelée
+« somme naturelle » ou « somme de Hessenberg ») sur les ordinaux (ici
+notés $\alpha,\beta$) par la formule suivante :
+\[
+\alpha \boxplus \beta = \sup\nolimits^+ \Big( \{\alpha'\boxplus\beta :
+\alpha' < \alpha\} \cup \{\alpha\boxplus\beta' : \beta' <
+\beta\}\Big)
+\]
+où on rappelle que $\sup^+ S$, si $S$ est un ensemble d'ordinaux,
+désigne \emph{le plus petit ordinal strictement plus grand que tous
+ les éléments de $S$} (c'est aussi $\sup\{\gamma+1 : \gamma\in S\}$
+mais c'est probablement moins utile d'y penser sous cette forme).
+Autrement dit, $\alpha\boxplus\beta$ désigne le plus petit ordinal qui
+soit strictement supérieur à tous les $\alpha'\boxplus\beta$ pour
+$\alpha'<\alpha$ ainsi qu'à tous les $\alpha\boxplus\beta'$
+pour $\beta'<\beta$. Cette définition a bien un sens par induction
+bien-fondée.
+
+\medskip
+
+%% À toutes fins utiles, on signale le fait évident qu'on a $\xi =
+%% \sup^+ S$ si et seulement si on a (i) $\xi$ est strictement
+%% supérieur à tout élément de $S$, et (ii) tout ordinal $<\xi$ est
+%% inférieur ou égal à un élément de $S$.
+
+À toutes fins utiles, on signale le fait évident que $\sup^+ S$ ne
+change pas si on insère dans $S$ des nouveaux éléments qui sont
+majorés par des éléments déjà dans $S$.
+
+\medskip
+
+(1)(a) Calculer $m\boxplus n$ pour $0\leq m\leq 5$ et $0\leq n\leq
+5$.\quad(b) Conjecturer une formule générale pour $m\boxplus n$
+lorsque $m,n\in\mathbb{N}$ (c'est-à-dire
+$m,n<\omega$).\quad(c) Démontrer cette formule.
+
+\begin{corrige}
+(a)/(b) On construit le tableau de proche en proche en inscrivant dans
+ chaque case le plus petit entier strictement supérieur à tous les
+ nombres écrits plus haut dans la colonne ou plus à gauche dans la
+ ligne : on obtient $m\boxplus n = m + n$, et on peut imaginer que
+ c'est vrai en général.
+
+(c) Montrons que $m\boxplus n = m + n$ pour tous $m,n\in\mathbb{N}$ :
+ par récurrence (sur $m+n$), on peut supposer connu le fait que
+ $m'\boxplus n = m' + n$ pour tout $m'<m$ et $m\boxplus n' = m + n'$
+ pour tout $n'<n$. On a alors $m \boxplus n = \sup^+ \big( \{m'
+ \boxplus n : m' < m\} \penalty0 \cup \penalty0 \{m \boxplus n' : n'
+ < n\}\big) = \sup^+ \big( \{m' + n : m' < m\} \penalty0 \cup
+ \penalty0 \{m + n' : n' < n\}\big)$ ; or l'ensemble dont on prend le
+ $\sup^+$ ne contient que des entiers $<m+n$ et contient $m+n-1$ donc
+ ce $\sup^+$ vaut $m+n$, ce qui conclut la récurrence.
+\end{corrige}
+
+\medskip
+
+(2)(a) Montrer que $\boxplus$ est commutative, c'est-à-dire que
+$\alpha_1\boxplus\alpha_2 = \alpha_2\boxplus\alpha_1$ quels que
+soient $\alpha_1,\alpha_2$.\quad(b) Montrer que $\boxplus$ admet $0$
+pour élément neutre, c'est-à-dire que $\alpha\boxplus 0 =
+0\boxplus\alpha = \alpha$ pour tout ordinal $\alpha$.\quad(c) Montrer
+que si $\alpha'\leq\alpha$ et $\beta'\leq\beta$ alors
+$\alpha'\boxplus\beta' \leq \alpha\boxplus\beta$, avec inégalité stricte
+dans la conclusion si au moins une d'elles est stricte dans
+l'hypothèse.
+
+\begin{corrige}
+(a) Par induction transfinie sur $\alpha_1$ et $\alpha_2$, on prouve
+ $\alpha_2\boxplus\alpha_1 = \alpha_1\boxplus\alpha_2$ : en effet,
+ $\alpha_2\boxplus\alpha_1 = \sup^+ (\{\alpha_2\boxplus\beta_1:
+ \beta_1<\alpha_1\} \penalty0 \cup \penalty0
+ \{\beta_2\boxplus\alpha_1: \beta_2<\alpha_2\})$, et par hypothèse
+ d'induction ceci vaut $\sup^+ (\{\beta_1\boxplus\alpha_2:
+ \beta_1<\alpha_1\} \penalty0 \cup \penalty0
+ \{\alpha_1\boxplus\beta_2: \beta_2<\alpha_2\}) =
+ \alpha_1\boxplus\alpha_2$.
+
+(b) Par induction sur $\alpha$, on prouve $\alpha \boxplus 0 =
+ \alpha$ : en effet, $\alpha \boxplus 0 = \sup^+ \{\beta\boxplus 0:
+ \beta<\alpha\}$, et par hypothèse d'induction ceci vaut $\sup^+
+ \{\beta: \beta<\alpha\} = \alpha$. Par la commutativité déjà
+ prouvée, $0 \boxplus \alpha$ vaut lui aussi $\alpha$.
+
+(c) Si $\alpha'<\alpha$ alors $\alpha'\boxplus\beta <
+ \alpha\boxplus\beta$ par définition même de $\alpha\boxplus\beta$,
+ et de même si $\beta'<\beta$ alors $\alpha\boxplus\beta' <
+ \alpha\boxplus\beta$. Si on a à la fois $\alpha'<\alpha$ et
+ $\beta'<\beta$ alors $\alpha'\boxplus\beta' < \alpha'\boxplus\beta <
+ \alpha\boxplus\beta$ d'après ce qu'on vient de dire. C'est
+ exactement ce qu'il fallait démontrer.
+\end{corrige}
+
+\medskip
+
+On \underline{admettra} pour la suite que $\boxplus$ est associative,
+c'est-à-dire que $(\alpha_1\boxplus\alpha_2) \boxplus \alpha_3 =
+\alpha_1 \boxplus (\alpha_2\boxplus\alpha_3)$ quels que soient
+$\alpha_1,\alpha_2,\alpha_3$ ; et on admettra aussi que
+$\alpha_1\boxplus\cdots\boxplus\alpha_n$ est le plus petit ordinal
+strictement supérieur à tous les
+$\alpha_1\boxplus\cdots\boxplus\beta_i\boxplus
+\cdots\boxplus\alpha_n$, où exactement un des $\alpha_i$ a été
+remplacé par un ordinal $\beta_i$ strictement plus petit
+(généralisation de la définition au cas de $n$ termes, analogue à un
+résultat vu en cours sur les sommes de nim).
+
+\smallskip
+
+Si $\alpha$ est un ordinal et $n\geq 1$ un entier naturel, on
+\underline{notera} $\alpha\boxdot n$ pour la somme naturelle $\alpha
+\boxplus \cdots \boxplus \alpha$ de $n$ fois l'ordinal $\alpha$ (on
+pourra aussi poser $\alpha\boxdot 0 = 0$). Dans ces conditions, il
+est trivial que $(\alpha\boxdot n) \boxplus (\alpha\boxdot n') =
+\alpha\boxdot (n+n')$ ; par ailleurs, on a $1\boxdot n = n$ d'après la
+question (1).
+
+\medskip
+
+Considérons l'affirmation suivante :
+
+{\narrower\noindent\leavevmode\llap{(*) }si $\alpha =
+ \omega^{\gamma_1} n_1 + \cdots + \omega^{\gamma_r} n_r$ où
+ $\gamma_1 > \cdots > \gamma_r$ sont des ordinaux en ordre
+ strictement décroissant et $n_1,\ldots,n_r$ des entiers naturels non
+ nuls (c'est-à-dire qu'il s'agit de la forme normale de Cantor
+ de $\alpha$), alors on a $\alpha = (\omega^{\gamma_1} \boxdot n_1)
+ \boxplus \cdots \boxplus (\omega^{\gamma_r} \boxdot n_r)$\par}
+
+\noindent (ceci est à comparer au fait, démontré en cours, que si
+$\alpha = 2^{\gamma_1} + \cdots + 2^{\gamma_r}$, où $\gamma_1 > \cdots
+> \gamma_r$ sont des ordinaux en ordre strictement décroissant,
+c'est-à-dire qu'il s'agit de l'écriture binaire de $\alpha$, alors on
+a $\alpha = 2^{\gamma_1} \oplus \cdots \oplus 2^{\gamma_r}$).
+
+On pourrait démontrer (*) par induction transfinie sur $\alpha$.
+Comme c'est notationnellement un peu fastidieux, on va seulement, dans
+la question suivante, montrer un cas particulier assez représentatif
+de l'idée générale :
+
+\medskip
+
+(3)(a) Pour $n_0,n_1 \in \mathbb{N}$, montrer que $(\omega\boxdot n_1)
+\boxplus n_0 = \sup^+ \big( \{(\omega\boxdot n'_1) \boxplus n_0' :
+n'_1 < n_1 \;\hbox{et}\; n'_0 \in\mathbb{N}\} \penalty0 \cup
+\penalty0 \{(\omega\boxdot n_1) \boxplus n_0' : n'_0 <
+n_0\}\big)$.\quad (b) En déduire par induction transfinie sur $\alpha$
+que si $\alpha = \omega\, n_1 + n_0$ où $n_0,n_1 \in \mathbb{N}$,
+alors $\alpha = (\omega \boxdot n_1) \boxplus n_0$ (ceci est un cas
+particulier de (*)).
+
+\begin{corrige}
+(a) D'après ce qu'on a admis ci-dessus, $(\omega\boxdot n_1) \boxplus
+ n_0$, qui est une somme naturelle de $n_1$ copies de $\omega$ et
+ $n_0$ fois $1$, est le plus petit ordinal strictement supérieur à
+ toutes les sommes naturelles obtenues en remplaçant un
+ des ($n_1+n_0$) termes par un terme strictement plus petit,
+ c'est-à-dire à tous les $(\omega\boxdot (n_1-1)) \boxplus b \boxplus
+ n_0$ pour $b<\omega$ (cas où on remplace un $\omega$ par $b$) et à
+ $(\omega\boxdot n_1) \boxplus (n_0-1)$ (cas où on remplace un $1$
+ par $0$). En se rappelant que $b \boxplus n_0 = b + n_0$ (qui prend
+ donc toutes les valeurs $\geq n_0$) et qu'on a le droit d'insérer
+ dans un ensemble dont on prend le $\sup^+$ des nouveaux éléments qui
+ sont majorés par des éléments déjà dedans (et comme $\boxplus$ est
+ croissante en chaque variable d'après (2)(c)), on peut écrire
+ $(\omega\boxdot n_1) \boxplus n_0 = \sup^+ \big( \{(\omega\boxdot
+ n'_1) \boxplus n_0' : n'_1 < n_1 \;\hbox{et}\; n'_0 \in\mathbb{N}\}
+ \penalty0 \cup \penalty0 \{(\omega\boxdot n_1) \boxplus n_0' : n'_0
+ < n_0\}\big)$ comme annoncé.
+
+(b) On procède par induction transfinie sur $\alpha = \omega n_1 +
+ n_0$. On veut montrer que $\alpha = (\omega \boxdot n_1) \boxplus
+ n_0$. Or, d'après ce qu'on vient de voir, $(\omega\boxdot n_1)
+ \boxplus n_0 = \sup^+ \big( \{(\omega\boxdot n'_1) \boxplus n_0' :
+ n'_1 < n_1 \;\hbox{et}\; n'_0 \in\mathbb{N}\} \penalty0 \cup
+ \penalty0 \{(\omega\boxdot n_1) \boxplus n_0' : n'_0 < n_0\}\big)$.
+ D'après l'hypothèse d'induction, et en utilisant le fait que les
+ formes normales de Cantor des ordinaux se comparent
+ lexicographiquement, ceci vaut encore $\sup^+ \big( \{\omega\, n'_1
+ + n_0' : n'_1 < n_1 \;\hbox{et}\; n'_0 \in\mathbb{N}\} \penalty0
+ \cup \penalty0 \{\omega\, n_1 + n_0' : n'_0 < n_0\}\big)$, qui est
+ bien $\omega\, n_1 + n_0$ comme souhaité (il s'agit précisément du
+ $\sup^+$ de l'ensemble des ordinaux $<\omega\, n_1 + n_0$).
+\end{corrige}
+
+\medskip
+
+(4)(a) On admet maintenant l'affirmation (*) en toute généralité. En
+déduire la manière dont on calcule $\alpha\boxplus\beta$ à partir des
+formes normales de Cantor de $\alpha$ et $\beta$.\quad(b) En
+particulier, calculer $(\omega+1)\boxplus(\omega+1)$, et comparer avec
+$(\omega+1) + (\omega+1)$.
+
+\begin{corrige}
+(a) Soient $\alpha = \omega^{\gamma_1} p_1 + \cdots +
+ \omega^{\gamma_r} p_r$ et $\beta = \omega^{\gamma_1} q_1 + \cdots +
+ \omega^{\gamma_r} q_r$ (quitte à insérer des $p_i$ et $q_i$ nuls
+ dans la forme normale de Cantor) avec $\gamma_1 > \cdots >
+ \gamma_r$. En utilisant (*), on peut écrire $\alpha =
+ (\omega^{\gamma_1} \boxdot p_1) \boxplus \cdots \boxplus
+ (\omega^{\gamma_r} \boxdot p_r)$ et $\beta = (\omega^{\gamma_1}
+ \boxdot q_1) \boxplus \cdots \boxplus (\omega^{\gamma_r} \boxdot
+ q_r)$. En utilisant la commutativité et l'associativité
+ de $\boxdot$ et en regroupant les puissances égales de $\omega$, on
+ obtient $\alpha \boxplus \beta = (\omega^{\gamma_1} \boxdot
+ (p_1+q_1)) \boxplus \cdots \boxplus (\omega^{\gamma_r} \boxdot
+ (p_r+q_r))$. Et quitte à utiliser de nouveau (*), on trouve $\alpha
+ \boxplus \beta = \omega^{\gamma_1} (p_1+q_1) + \cdots +
+ \omega^{\gamma_r} (p_r+q_r)$. On a ainsi montré que la somme
+ naturelle se fait en ajoutant simplement terme à terme les formes
+ normales de Cantor, i.e., on ajoute les chiffres (=coefficients) de
+ chaque puissance de $\omega$.
+
+(b) En particulier, $(\omega+1)\boxplus(\omega+1) = \omega\,2 + 2$
+ (qui est aussi $(\omega\boxdot 2)\boxplus 2$ comme on l'a vu), alors
+ que $(\omega+1) + (\omega+1) = \omega + (1+\omega) + 1 = \omega +
+ \omega + 1 = \omega\,2 + 1$ est strictement plus petit.
+\end{corrige}
+
+\medskip
+
+(La question qui suit ne fait pas appel aux questions (3) et (4).)
+
+(5) On considère le jeu à deux joueurs suivant : les deux joueurs
+s'appellent Blaise et Roxane, et ils jouent tour à tour (on ne précise
+pas qui commence) ; l'état du jeu est défini par trois ordinaux, qu'on
+notera $(\alpha,\beta,\rho)$, et il est visible de tous ; les coups
+possibles de Blaise consistent à diminuer strictement soit l'ordinal
+$\alpha$ soit l'ordinal $\beta$ (c'est-à-dire passer de
+$(\alpha,\beta,\rho)$ à $(\alpha',\beta,\rho)$ avec $\alpha'<\alpha$
+ou à $(\alpha,\beta',\rho)$ avec $\beta'<\beta$), tandis que les coups
+possibles de Roxane consistent à diminuer strictement l'ordinal $\rho$
+(c'est-à-dire passer de $(\alpha,\beta,\rho)$ à $(\alpha,\beta,\rho')$
+avec $\rho'<\rho$) ; le joueur qui ne peut plus jouer a perdu.
+
+Montrer que, dans ce jeu, Blaise possède une stratégie gagnante
+lorsque $\rho < \alpha\boxplus\beta$, que Roxane possède une stratégie
+gagnante lorsque $\rho > \alpha\boxplus\beta$, et que le second joueur
+possède une stratégie gagnante lorsque $\rho = \alpha\boxplus\beta$.
+
+\begin{corrige}
+On procède par induction transfinie (sur
+$\alpha\boxplus\beta\boxplus\rho$, si l'on veut) : on peut supposer la
+conclusion déjà connue pour tous les $(\alpha',\beta,\rho)$,
+$(\alpha,\beta',\rho)$ et $(\alpha,\beta,\rho')$ tels que dans la
+question.
+
+Si Blaise joue en premier : si $\rho < \alpha\boxplus\beta$, alors par
+définition de $\alpha\boxplus\beta$, il existe un
+$\alpha'\boxplus\beta$ avec $\alpha'<\alpha$ ou $\alpha\boxplus\beta'$
+avec $\beta'<\beta$ qui majore $\rho$ (au sens large), et Blaise peut
+faire le coup correspondant $(\alpha',\beta,\rho)$ ou
+$(\alpha,\beta',\rho)$ auquel cas il aura une stratégie gagnante comme
+second joueur par hypothèse d'induction ; si $\rho \geq
+\alpha\boxplus\beta$, alors quel que soit le coup
+$(\alpha',\beta,\rho)$ ou $(\alpha,\beta',\rho)$ fait par Blaise, on
+aura $\rho > \alpha'\boxplus\beta$ ou $\rho > \alpha\boxplus\beta'$
+donc Roxane aura une stratégie gagnante par hypothèse d'induction.
+
+Si Roxane joue en premier : si $\rho > \alpha\boxplus\beta$, alors
+elle peut jouer vers $(\alpha,\beta,\rho')$ avec $\rho' =
+\alpha\boxplus\beta$, auquel cas elle comme second joueur aura une
+stratégie gagnante par hypothèse d'induction. Si $\rho \leq
+\alpha\boxplus\beta$, alors quel que soit le coup qu'elle fait, elle
+arrivera à un $(\alpha,\beta,\rho')$ avec $\rho' <
+\alpha\boxplus\beta$ auquel cas Blaise a une stratégie gagnante par
+hypothèse d'induction.
+\end{corrige}
+
+{\footnotesize Autrement dit, on a montré que l'addition des « nombres
+ (surréels) » de Conway coïncide, dans le cas particulier des
+ ordinaux, avec l'opération $\boxplus$ de somme naturelle.\par}
+
+
+%
+%
+%
+
+\exercice
+
+Soit $n\geq 3$ un entier naturel. On considère le jeu suivant : Alice
+et Bob choisissent chacun en secret un entier naturel $0\leq i\leq
+n-1$ et le révèlent simultanément. Si les deux nombres sont égaux, la
+partie est nulle ; sinon, le gagnant est celui qui a choisi le plus
+grand, \emph{sauf} dans le cas où un joueur a choisi $n-1$ et l'autre
+joueur a choisi $0$, et alors c'est celui qui a choisi $0$ qui gagne.
+
+(On considérera qu'un gain apporte une valeur de $+1$ à celui qui
+gagne, une perte une valeur de $-1$ à celui qui perd, et qu'une partie
+nulle a une valeur de $0$ pour les deux joueurs.)
+
+(1) De quelle sorte de jeu s'agit-il ? Écrire explicitement sa
+matrice de gains dans le cas $n=5$. Pour des raisons de symétrie,
+quelle est la valeur du jeu ?
+
+\begin{corrige}
+Il s'agit d'un jeu en forme normale et à somme nulle. Pour $n=5$, la
+matrice des gains d'Alice vaut :
+
+\begin{center}
+\begin{tabular}{r|ccccc}
+$\downarrow$Alice, Bob$\rightarrow$&${0}$&${1}$&${2}$&${3}$&${4}$\\\hline
+${0}$&$0$&$-1$&$-1$&$-1$&$+1$\\
+${1}$&$+1$&$0$&$-1$&$-1$&$-1$\\
+${2}$&$+1$&$+1$&$0$&$-1$&$-1$\\
+${3}$&$+1$&$+1$&$+1$&$0$&$-1$\\
+${4}$&$-1$&$+1$&$+1$&$+1$&$0$\\
+\end{tabular}
+\end{center}
+
+Pour des raisons de symétrie (la matrice étant antisymétrique,
+c'est-à-dire que les deux joueurs sont dans la même situation
+vis-à-vis du jeu), la valeur du gain vaut $0$.
+\end{corrige}
+
+(2) On considère la stratégie mixte consistant à jouer chacune des
+options $0$, $n-2$ et $n-1$ avec probabilité $\frac{1}{3}$ (et jamais
+les autres). On l'appellera $s_0$. Si Alice joue selon cette
+stratégie $s_0$ et si Bob joue l'option $i$, quel est le gain espéré
+d'Alice en fonction de $i$ ?
+
+\begin{corrige}
+Si Bob joue $0$, Alice obtient le gain espéré $\frac{1}{3}(0 + 1 - 1)
+= 0$. Si Bob joue une option entre $1$ et $n-3$ inclus, Alice obtient
+le gain espéré $\frac{1}{3}(-1 + 1 + 1) = \frac{1}{3} > 0$. Si Bob
+joue $n-2$, Alice obtient le gain espéré $\frac{1}{3}(-1 + 0 + 1) =
+0$. Si Bob joue $n-1$, Alice obtient le gain espéré $\frac{1}{3}(+1 -
+1 + 0) = 0$.
+\end{corrige}
+
+(3) Pourquoi $s_0$ est-elle une stratégie optimale ?
+
+\begin{corrige}
+Pour vérifier que $s_0$ est optimale, il s'agit de vérifier qu'elle
+réalise au moins la valeur du jeu contre toute option (=stratégie
+pure) de l'adversaire. C'est ce qu'on vient de faire puisque la
+valeur du jeu est $0$.
+\end{corrige}
+
+(4) Déduire de (2) qu'aucune stratégie optimale ne peut avoir une
+option autre que $0$, $n-2$ ou $n-1$ dans son support. (On pourra
+faire jouer une telle stratégie contre $s_0$.)
+
+\begin{corrige}
+Si $t$ est une stratégie ayant une autre option que $0$, $n-2$
+et $n-1$ dans son support, les espérances trouvées en (2) montrent que
+son espérance de gain contre $s_0$ est strictement négative
+(strictement positive pour le joueur qui applique $s_0$, donc
+strictement négative pour celui qui applique $t$). Donc $t$ ne peut
+pas être optimale.
+\end{corrige}
+
+(5) Montrer que la stratégie $s_0$ décrite en (2) est la seule
+stratégie optimale de ce jeu.
+
+\begin{corrige}
+On vient de voir que toute stratégie optimale $s$ a un support inclus
+dans $\{0, n-2, n-1\}$. Si on appelle $p_0, p_{-2}, p_{-1}$ les
+probabilités respectives de ces options dans $s$, on sait que $s$ doit
+avoir une espérance de gain nulle contre $s_0$, donc contre chacune
+des options pures $0$, $n-2$ et $n-1$, ce qui donne $p_{-2} = p_{-1}$,
+$p_{-1} = p_0$ et $p_0 = p_{-2}$, bref, la seule possibilité est $p_0
+= p_{-2} = p_{-1} = \frac{1}{3}$.
+\end{corrige}
+
+
+%
+%
+%
+\end{document}
diff --git a/controle-2020qcm.tex b/controle-2020qcm.tex
new file mode 100644
index 0000000..f33cec5
--- /dev/null
+++ b/controle-2020qcm.tex
@@ -0,0 +1,1332 @@
+%% This is a LaTeX document. Hey, Emacs, -*- latex -*- , get it?
+\documentclass[12pt,a4paper]{article}
+\usepackage[francais]{babel}
+\usepackage[utf8]{inputenc}
+\usepackage[T1]{fontenc}
+%\usepackage{ucs}
+\usepackage{times}
+% A tribute to the worthy AMS:
+\usepackage{amsmath}
+\usepackage{amsfonts}
+\usepackage{amssymb}
+\usepackage{amsthm}
+%
+\usepackage{mathrsfs}
+\usepackage{wasysym}
+\usepackage{url}
+%
+\usepackage{graphics}
+\usepackage[usenames,dvipsnames]{xcolor}
+\usepackage{tikz}
+\usetikzlibrary{matrix,calc}
+\usepackage{hyperref}
+%
+%\externaldocument{notes-mitro206}[notes-mitro206.pdf]
+%
+\newenvironment{qcm}{\relax}{\relax}
+\newenvironment{qvar}{\relax}{\relax}
+\newcounter{quescnt}
+\newenvironment{question}%
+{\stepcounter{quescnt}\bigskip\noindent\textbf{Question~\arabic{quescnt}.}\par\nobreak}
+{\relax}
+\newcounter{answcnt}[quescnt]
+\newcommand\answer{%
+\stepcounter{answcnt}\smallskip\textbf{(\Alph{answcnt})}~}
+\let\rightanswer=\answer
+%
+\newcommand{\outnb}{\operatorname{outnb}}
+\newcommand{\downstr}{\operatorname{downstr}}
+\newcommand{\precs}{\operatorname{precs}}
+\newcommand{\mex}{\operatorname{mex}}
+\newcommand{\id}{\operatorname{id}}
+\newcommand{\limp}{\Longrightarrow}
+\newcommand{\gr}{\operatorname{gr}}
+\newcommand{\rk}{\operatorname{rk}}
+\newcommand{\fuzzy}{\mathrel{\|}}
+%
+\newcommand{\dblunderline}[1]{\underline{\underline{#1}}}
+%
+\DeclareUnicodeCharacter{00A0}{~}
+%
+\DeclareMathSymbol{\tiret}{\mathord}{operators}{"7C}
+\DeclareMathSymbol{\traitdunion}{\mathord}{operators}{"2D}
+%
+\newif\ifcorrige
+\corrigefalse
+\def\seedval{test}
+%
+%
+%
+\begin{document}
+\ifcorrige
+\title{MITRO206\\Contrôle de connaissances — Corrigé\\{\normalsize Théories des jeux}}
+\else
+\title{MITRO206\\Contrôle de connaissances\\{\normalsize Théories des jeux}}
+\fi
+\author{}
+\date{26 juin 2020}
+\maketitle
+
+\pretolerance=8000
+\tolerance=50000
+
+\vskip1truein\relax
+
+\noindent\textbf{Consignes.}
+
+Ce contrôle de connaissances est un QCM (questionnaire à choix
+multiples). Chaque question admet une unique réponse correcte. Les
+questions sont totalement indépendantes les unes des autres (mais
+certaines peuvent se ressembler). La sélection des questions et
+l'ordre ont été tirés aléatoirement et n'obéissent donc à aucune
+logique particulière.
+
+La réponse est attendue sous forme d'une liste de numéros de question
+suivie de la réponse proposée : par exemple, « \verb=1A 2B 4D= » pour
+signifier que la réponse proposée à la question 1 est (A), la réponse
+proposée à la question 2 est (B), et la réponse proposée à la
+question 4 est (D).
+
+Une réponse incorrecte sera (possiblement jusqu'à deux fois) plus
+fortement pénalisée qu'une absence de réponse : il est donc préférable
+de ne pas répondre à une question que de répondre aléatoirement.
+
+\medbreak
+
+Durée : 1h de 17h30 à 18h30
+
+\vfill
+
+\noindent
+Sujet généré pour : \texttt{\seedval}
+
+\medskip
+
+{\tiny\noindent
+\immediate\write18{sh ./vc > vcline.tex}
+Git: \input{vcline.tex}
+\immediate\write18{echo ' (stale)' >> vcline.tex}
+\par}
+
+\pagebreak
+
+\begin{qcm}
+
+
+%
+%
+%
+
+\begin{question}
+
+Considérons le jeu suivant : Alice choisit une ligne du tableau
+suivant, \emph{puis} (en ayant connaissance du choix qu'Alice vient de
+faire) Bob choisit une colonne ; Alice gagne alors un score égal au
+nombre inscrit dans le tableau (à la ligne et la colonne choisies) et
+Bob gagne le score opposé. (Chaque joueur cherche à maximiser son
+score.)
+
+\begin{center}
+\begin{tabular}{r|rrrrr}
+$\downarrow$Alice, Bob$\rightarrow$&U&V&W&X&Y\\\hline
+U&$0$&$0$&$+1$&$+1$&$-1$\\
+V&$0$&$0$&$-1$&$-1$&$+1$\\
+W&$-1$&$+1$&$0$&$0$&$+2$\\
+X&$-1$&$+1$&$0$&$0$&$-1$\\
+Y&$+1$&$-1$&$-2$&$+1$&$0$\\
+%% m = Matrix(QQ, 5, 5, [[0, 0, 1, 1, -1], [0, 0, -1, -1, 1], [-1, 1, 0, 0, 2], [-1, 1, 0, 0, -1], [1, -1, -2, 1, 0]])
+\end{tabular}
+\end{center}
+
+Laquelle des affirmations suivantes est correcte ?
+
+\rightanswer
+il s'agit d'un jeu à information parfaite, et Alice a une stratégie
+lui garantissant un score $\geq -1$
+
+\answer
+il s'agit d'un jeu à information parfaite, et Alice a une stratégie
+lui garantissant un score $\geq +2$
+
+\answer
+il s'agit d'un jeu en forme normale, à information imparfaite, et les
+deux joueurs ont une stratégie leur donnant à chacun un score espéré
+$\geq 0$
+
+\answer
+il s'agit d'un jeu en forme normale, à information imparfaite, et les
+deux joueurs ont une stratégie leur donnant à chacun un score espéré
+$\geq +1$
+
+\end{question}
+
+
+%
+%
+%
+
+\begin{question}
+
+Alice et Bob jouent au jeu suivant : chacun tour à tour place un pion
+sur un échiquier $8\times 8$ de manière à n'être ni sur la même ligne,
+ni sur la même colonne, ni sur une même diagonale (dans un sens ou
+dans l'autre) qu'un pion déjà placé (par un joueur ou l'autre) ;
+autrement dit, les pions ne doivent pas pouvoir se prendre selon un
+mouvement de dame aux échecs. Comme d'habitude, le premier joueur qui
+ne peut pas jouer a perdu.
+
+Chloé tient le raisonnement suivant (imitant le raisonnement
+justifiant que dans le jeu de chomp le premier joueur a une stratégie
+gagnante) : « (1) on a affaire à un jeu à information parfaite,
+impartial, défini par un graphe bien-fondé, donc l'un des deux joueurs
+a une stratégie gagnante ; (2) ce joueur est forcément Alice : en
+effet, supposons par l'absurde que ce soit Bob, alors Bob aurait un
+coup gagnant à jouer en réponse à n'importe quel coup initial d'Alice,
+mais Alice pourrait jouer ce coup dès le premier tour, se mettant
+ainsi dans la position supposément gagnante ».
+
+Que pensez-vous de ce raisonnement (on ne demande pas de se prononcer
+sur l'exactitude de la conclusion, i.e., si Alice a une stratégie
+gagnante ou pas, mais sur le \emph{raisonnement} qu'on vient
+d'écrire) ?
+
+\rightanswer
+la partie (1) est correcte, mais la partie (2) ne l'est pas (Alice ne
+peut pas forcément se ramener à une position supposément gagnante)
+
+\answer
+la partie (1) est incorrecte (il ne s'agit pas d'un jeu à information
+parfaite impartial défini par un graphe bien-fondé)
+
+\answer
+le raisonnement est correct (les deux parties (1) et (2) le sont)
+
+\end{question}
+
+
+%
+%
+%
+
+\begin{qvar}
+
+\begin{question}
+
+Considérons le jeu en forme normale à somme nulle, symétrique, entre
+Alice et Bob, dont la matrice des gains est donnée par le tableau
+suivant (Alice choisit la ligne, Bob choisit la colonne, le tableau
+donne le gain d'Alice et le gain de Bob est l'opposé de la valeur
+indiquée) :
+
+\begin{center}
+\begin{tabular}{r|rrrrr}
+$\downarrow$Alice, Bob$\rightarrow$&U&V&W&X&Y\\\hline
+U&$0$&$0$&$+1$&$+1$&$-1$\\
+V&$0$&$0$&$-1$&$-1$&$+1$\\
+W&$-1$&$+1$&$0$&$0$&$+2$\\
+X&$-1$&$+1$&$0$&$0$&$-1$\\
+Y&$+1$&$-1$&$-2$&$+1$&$0$\\
+%% m = Matrix(QQ, 5, 5, [[0, 0, 1, 1, -1], [0, 0, -1, -1, 1], [-1, 1, 0, 0, 2], [-1, 1, 0, 0, -1], [1, -1, -2, 1, 0]])
+\end{tabular}
+\end{center}
+
+Laquelle des réponses suivantes est une stratégie optimale à ce jeu ?
+(Chaque réponse proposée est la liste des probabilités de jouer les
+options U,V,W,X,Y dans cet ordre.)
+
+\rightanswer
+$(\frac{1}{2}, 0, \frac{1}{4}, 0, \frac{1}{4})$
+
+\answer
+$(1, 0, 0, 0, 0)$
+
+\answer
+$(0, \frac{1}{3}, 0, \frac{1}{3}, \frac{1}{3})$
+
+\answer
+$(0, 0, 0, 0, 1)$
+
+\end{question}
+
+\begin{question}
+
+Considérons le jeu en forme normale à somme nulle, symétrique, entre
+Alice et Bob, dont la matrice des gains est donnée par le tableau
+suivant (Alice choisit la ligne, Bob choisit la colonne, le tableau
+donne le gain d'Alice et le gain de Bob est l'opposé de la valeur
+indiquée) :
+
+\begin{center}
+\begin{tabular}{r|rrrrr}
+$\downarrow$Alice, Bob$\rightarrow$&U&V&W&X&Y\\\hline
+U&$0$&$+1$&$-1$&$0$&$-1$\\
+V&$-1$&$0$&$+2$&$-1$&$-1$\\
+W&$+1$&$-2$&$0$&$+1$&$+2$\\
+X&$0$&$+1$&$-1$&$0$&$0$\\
+Y&$+1$&$+1$&$-2$&$0$&$0$\\
+%% m = Matrix(QQ, 5, 5, [[0, 1, -1, 0, -1], [-1, 0, 2, -1, -1], [1, -2, 0, 1, 2], [0, 1, -1, 0, 0], [1, 1, -2, 0, 0]])
+\end{tabular}
+\end{center}
+
+Laquelle des réponses suivantes est une stratégie optimale à ce jeu ?
+(Chaque réponse proposée est la liste des probabilités de jouer les
+options U,V,W,X,Y dans cet ordre.)
+
+\rightanswer
+$(\frac{1}{4}, \frac{1}{4}, \frac{1}{4}, \frac{1}{4}, 0)$
+
+\answer
+$(\frac{1}{5}, \frac{1}{5}, \frac{1}{5}, \frac{1}{5}, \frac{1}{5})$
+
+\answer
+$(0, 0, 0, 0, 1)$
+
+\answer
+$(\frac{1}{3}, \frac{1}{3}, \frac{1}{3}, 0, 0)$
+
+\end{question}
+
+\begin{question}
+
+Considérons le jeu en forme normale à somme nulle, symétrique, entre
+Alice et Bob, dont la matrice des gains est donnée par le tableau
+suivant (Alice choisit la ligne, Bob choisit la colonne, le tableau
+donne le gain d'Alice et le gain de Bob est l'opposé de la valeur
+indiquée) :
+
+\begin{center}
+\begin{tabular}{r|rrrrr}
+$\downarrow$Alice, Bob$\rightarrow$&U&V&W&X&Y\\\hline
+U&$0$&$+2$&$+1$&$0$&$-2$\\
+V&$-2$&$0$&$+1$&$+1$&$+1$\\
+W&$-1$&$-1$&$0$&$-1$&$+2$\\
+X&$0$&$-1$&$+1$&$0$&$-2$\\
+Y&$+2$&$-1$&$-2$&$+2$&$0$\\
+%% m = Matrix(QQ, 5, 5, [[0, 2, 1, 0, -2], [-2, 0, 1, 1, 1], [-1, -1, 0, -1, 2], [0, -1, 1, 0, -2], [2, -1, -2, 2, 0]])
+\end{tabular}
+\end{center}
+
+Laquelle des réponses suivantes est une stratégie optimale à ce jeu ?
+(Chaque réponse proposée est la liste des probabilités de jouer les
+options U,V,W,X,Y dans cet ordre.)
+
+\rightanswer
+$(\frac{2}{5}, 0, \frac{2}{5}, 0, \frac{1}{5})$
+
+\answer
+$(\frac{1}{2}, 0, \frac{1}{2}, 0, 0)$
+
+\answer
+$(0, 0, \frac{2}{5}, \frac{2}{5}, \frac{1}{5})$
+
+\answer
+$(0, 0, \frac{1}{2}, \frac{1}{2}, 0)$
+
+\end{question}
+
+\end{qvar}
+
+
+%
+%
+%
+
+\begin{qvar}
+
+\begin{question}
+
+Considérons le jeu en forme normale à somme nulle, entre Alice et Bob,
+dont la matrice des gains est donnée par le tableau suivant (Alice
+choisit la ligne, Bob choisit la colonne, le tableau donne le gain
+d'Alice et le gain de Bob est l'opposé de la valeur indiquée) :
+
+\begin{center}
+\begin{tabular}{r|rrr}
+$\downarrow$Alice, Bob$\rightarrow$&X&Y&Z\\\hline
+U&$-3$&$0$&$+3$\\
+V&$0$&$-1$&$-3$\\
+W&$-2$&$+2$&$-1$\\
+%% m = Matrix(QQ, 3, 3, [[-3, 0, 3], [0, -1, -3], [-2, 2, -1]])
+\end{tabular}
+\end{center}
+
+Quelle est la stratégie optimale de chacun des deux joueurs à ce jeu ?
+(Chaque réponse proposée est la liste des probabilités pour Alice de
+jouer les options U,V,W dans cet ordre puis pour Bob de jouer X,Y,Z.)
+
+\rightanswer
+$(\frac{1}{3}, \frac{2}{3}, 0)$ pour Alice, et $(\frac{2}{3}, 0,
+\frac{1}{3})$ pour Bob
+
+\answer
+$(\frac{1}{4}, \frac{3}{4}, 0)$ pour Alice, et $(\frac{1}{4},
+\frac{3}{4}, 0)$ pour Bob
+
+\answer
+$(0,1,0)$ pour Alice, et $(1,0,0)$ pour Bob
+
+\end{question}
+
+\begin{question}
+
+Considérons le jeu en forme normale à somme nulle, entre Alice et Bob,
+dont la matrice des gains est donnée par le tableau suivant (Alice
+choisit la ligne, Bob choisit la colonne, le tableau donne le gain
+d'Alice et le gain de Bob est l'opposé de la valeur indiquée) :
+
+\begin{center}
+\begin{tabular}{r|rrr}
+$\downarrow$Alice, Bob$\rightarrow$&X&Y&Z\\\hline
+U&$-3$&$-2$&$+2$\\
+V&$+1$&$+2$&$0$\\
+W&$0$&$-2$&$-3$\\
+%% m = Matrix(QQ, 3, 3, [[-3, -2, 2], [1, 2, 0], [0, -2, -3]])
+\end{tabular}
+\end{center}
+
+Quelle est la stratégie optimale de chacun des deux joueurs à ce jeu ?
+(Chaque réponse proposée est la liste des probabilités pour Alice de
+jouer les options U,V,W dans cet ordre puis pour Bob de jouer X,Y,Z.)
+
+\rightanswer
+$(\frac{1}{6}, \frac{5}{6}, 0)$ pour Alice, et $(\frac{1}{3}, 0,
+\frac{2}{3})$ pour Bob
+
+\answer
+$(\frac{3}{8}, 0, \frac{5}{8})$ pour Alice, et $(\frac{5}{8}, 0,
+\frac{3}{8})$ pour Bob
+
+\answer
+$(0,1,0)$ pour Alice, et $(0,0,1)$ pour Bob
+
+\end{question}
+
+\begin{question}
+
+Considérons le jeu en forme normale à somme nulle, entre Alice et Bob,
+dont la matrice des gains est donnée par le tableau suivant (Alice
+choisit la ligne, Bob choisit la colonne, le tableau donne le gain
+d'Alice et le gain de Bob est l'opposé de la valeur indiquée) :
+
+\begin{center}
+\begin{tabular}{r|rrr}
+$\downarrow$Alice, Bob$\rightarrow$&X&Y&Z\\\hline
+U&$-3$&$-2$&$+2$\\
+V&$+3$&$+1$&$0$\\
+W&$+1$&$+1$&$-1$\\
+%% m = Matrix(QQ, 3, 3, [[-3, -2, 2], [3, 1, 0], [1, 1, -1]])
+\end{tabular}
+\end{center}
+
+Quelle est la stratégie optimale de chacun des deux joueurs à ce jeu ?
+(Chaque réponse proposée est la liste des probabilités pour Alice de
+jouer les options U,V,W dans cet ordre puis pour Bob de jouer X,Y,Z.)
+
+\rightanswer
+$(\frac{1}{5}, \frac{4}{5}, 0)$ pour Alice, et $(0, \frac{2}{5},
+\frac{3}{5})$ pour Bob
+
+\answer
+$(\frac{1}{3}, 0, \frac{2}{3})$ pour Alice, et $(0, \frac{1}{2},
+\frac{1}{2})$ pour Bob
+
+\answer
+$(0,1,0)$ pour Alice, et $(0,0,1)$ pour Bob
+
+\end{question}
+
+\end{qvar}
+
+
+%
+%
+%
+
+\begin{question}
+
+Considérons le jeu (en forme normale) analogue à
+pierre-papier-ciseaux, sauf que les deux joueurs \emph{détestent}
+choisir tous les deux la même option, si bien que la matrice des gains
+est la suivante (ce n'est plus un jeu à somme nulle !) :
+
+\begin{center}
+\begin{tabular}{r|ccc}
+$\downarrow$Alice, Bob$\rightarrow$&Pierre&Papier&Ciseaux\\\hline
+Pierre&$-100,-100$&$-1,+1$&$+1,-1$\\
+Papier&$+1,-1$&$-100,-100$&$-1,+1$\\
+Ciseaux&$-1,+1$&$+1,-1$&$-100,-100$\\
+\end{tabular}
+\end{center}
+
+On considère deux profils de stratégies mixtes : (x) Alice et Bob
+jouent tous les deux une option entre pierre, papier ou ciseaux
+choisie aléatoirement avec probabilité $\frac{1}{3}$ (comme la
+stratégie optimale dans le cas du jeu à somme nulle) ; et : (y) Alice
+et Bob jouent tous les deux : papier avec probabilité $\frac{101}{200}
+= 0.505$, pierre avec probabilité $\frac{99}{200} = 0.495$ et jamais
+ciseaux. Que pensez-vous de ces deux profils ?
+
+\rightanswer
+(x) est un équilibre de Nash, mais (y) n'en est pas un
+
+\answer
+(x) n'est pas un équilibre de Nash, mais (y) en est un
+
+\answer
+(x) et (y) sont tous les deux des équilibres de Nash
+
+\answer
+ni (x) ni (y) n'est un équilibre de Nash
+
+\end{question}
+
+
+%
+%
+%
+
+\begin{qvar}
+
+\begin{question}
+
+Douze joueurs jouent au jeu suivant : chacun choisit une option parmi
+« rouge », « vert » ou « bleu ». Chacun reçoit alors un score (entre
+$1$ et $12$) égal au nombre (total) de joueurs ayant choisi cette
+option. (Autrement dit, chaque joueur choisit sa couleur et essaie
+d'être dans un groupe aussi grand que possible.)
+
+On considère trois profils de stratégies mixtes : (x) tous les joueurs
+jouent « rouge » ; (y) chaque joueur joue une option tirée au hasard
+uniformément (c'est-à-dire avec probabilité $\frac{1}{3}$ pour
+chacune) ; et (z) chaque joueur joue une option tirée au hasard mais
+uniquement entre « rouge » et « vert », chacune avec
+probabilité $\frac{1}{2}$. Que pensez-vous de ces profils ?
+
+\rightanswer
+(x), (y) et (z) sont tous les trois des équilibres de Nash
+
+\answer
+(x) est un équilibre de Nash, mais (y) et (z) n'en sont pas
+
+\answer
+(y) est un équilibre de Nash, mais (x) et (z) n'en sont pas
+
+\answer
+(y) et (z) sont tous les deux des équilibres de Nash, mais (x) n'en
+est pas
+
+\answer
+(x) et (y) sont tous les deux des équilibres de Nash, mais (z) n'en
+est pas
+
+\answer
+ni (x) ni (y) ni (z) n'est un équilibre de Nash
+
+\end{question}
+
+\begin{question}
+
+Douze joueurs jouent au jeu suivant : chacun choisit une option parmi
+« rouge », « vert » ou « bleu ». Chacun reçoit alors un score (entre
+$-1$ et $-12$) égal à \emph{l'opposé} du nombre (total) de joueurs
+ayant choisi cette option. (Autrement dit, chaque joueur choisit sa
+couleur et essaie d'être dans un groupe aussi petit que possible.)
+
+On considère trois profils de stratégies mixtes : (x) tous les joueurs
+jouent « rouge » ; (y) chaque joueur joue une option tirée au hasard
+uniformément (c'est-à-dire avec probabilité $\frac{1}{3}$ pour
+chacune) ; et (z) chaque joueur joue une option tirée au hasard mais
+uniquement entre « rouge » et « vert », chacune avec
+probabilité $\frac{1}{2}$. Que pensez-vous de ces profils ?
+
+\rightanswer
+(y) est un équilibre de Nash, mais (x) et (z) n'en sont pas
+
+\answer
+(x), (y) et (z) sont tous les trois des équilibres de Nash
+
+\answer
+(x) est un équilibre de Nash, mais (y) et (z) n'en sont pas
+
+\answer
+(y) et (z) sont tous les deux des équilibres de Nash, mais (x) n'en
+est pas
+
+\answer
+(x) et (y) sont tous les deux des équilibres de Nash, mais (z) n'en
+est pas
+
+\answer
+ni (x) ni (y) ni (z) n'est un équilibre de Nash
+
+\end{question}
+
+\end{qvar}
+
+
+%
+%
+%
+
+\begin{question}
+
+Ambre et Bastien ($8$ ans) jouent à
+pierre-papier-ciseaux-éléphant-souris, dont les règles sont les
+suivantes : chacun choisit une des cinq options (pierre, papier,
+ciseau, éléphant ou souris) indépendamment de l'autre, et
+\begin{itemize}
+\item si les deux joueurs ont choisi la même option, le jeu est nul,
+ sinon :
+\item si les deux joueurs ont choisi parmi pierre, papier ou ciseaux,
+ le gagnant est déterminé comme à pierre-papier-ciseaux (i.e., le
+ papier gagne sur la pierre, les ciseaux gagnent sur le papier et la
+ pierre gagne sur les ciseaux),
+\item l'éléphant gagne sur tout sauf la souris,
+\item la souris gagne sur l'éléphant et perd contre tout le reste.
+\end{itemize}
+(On accordera la valeur $+1$ au fait de gagner, $-1$ au fait de
+perdre, et $0$ à un match nul.)
+
+Quelle est la stratégie optimale à ce jeu ?
+
+\rightanswer
+jouer chacun de pierre, papier et ciseaux avec probabilité
+$\frac{1}{9}$, éléphant avec probabilité $\frac{1}{3}$ et souris avec
+probabilité $\frac{1}{3}$
+
+\answer
+jouer chacun de pierre, papier et ciseaux avec probabilité
+$\frac{1}{3}$, et jamais éléphant ni souris
+
+\answer
+jouer chacun de pierre, papier, ciseaux avec probabilité
+$\frac{1}{6}$, éléphant avec probabilité $\frac{1}{2}$ et jamais
+souris
+
+\answer
+jouer chacun de pierre, papier et ciseaux avec probabilité
+$\frac{1}{4}$, éléphant avec probabilité $\frac{1}{8}$ et souris avec
+probabilité $\frac{1}{8}$
+
+\answer
+jouer chacune des cinq options avec probabilité $\frac{1}{5}$
+
+\end{question}
+
+
+%
+%
+%
+
+\begin{qvar}
+
+\begin{question}
+
+Alice et Bob jouent au jeu de type Gale-Stewart suivant : chacun à son
+tour choisit un chiffre binaire (soit $0$ soit $1$ : Alice choisit
+$a_1$ puis Bob choisit $a_2$ puis Alice choisit $a_3$ et ainsi de
+suite). Au bout d'un nombre infini de tours, on considère le nombre
+réel $x$ entre $0$ et $1$ dont l'écriture binaire fractionnaire est
+formée de ces chiffres (c'est-à-dire $x = \sum_{i=1}^{+\infty}
+a_i\,2^{-i}$, ou $0{.}a_1a_2a_3\ldots$ en écriture binaire) : si $x
+\leq \frac{1}{3}$, Alice gagne, tandis que si $x > \frac{1}{3}$, Bob
+gagne. (À toutes fins utiles, on rappelle que $\frac{1}{3}$ s'écrit
+$0{.}01010101\ldots$ en binaire.) Que pensez-vous de ce jeu ?
+
+\rightanswer
+Alice a une stratégie gagnante
+
+\answer
+Bob a une stratégie gagnante
+
+\answer
+aucun joueur n'a de stratégie gagnante
+
+\answer
+un joueur a une stratégie gagnante, mais il est impossible de savoir
+lequel
+
+\end{question}
+
+\begin{question}
+
+Alice et Bob jouent au jeu de type Gale-Stewart suivant : chacun à son
+tour choisit un chiffre binaire (soit $0$ soit $1$ : Alice choisit
+$a_1$ puis Bob choisit $a_2$ puis Alice choisit $a_3$ et ainsi de
+suite). Au bout d'un nombre infini de tours, on considère le nombre
+réel $x$ entre $0$ et $1$ dont l'écriture binaire fractionnaire est
+formée de ces chiffres (c'est-à-dire $x = \sum_{i=1}^{+\infty}
+a_i\,2^{-i}$, ou $0{.}a_1a_2a_3\ldots$ en écriture binaire) : si $x
+< \frac{2}{3}$, Alice gagne, tandis que si $x \geq \frac{2}{3}$, Bob
+gagne. (À toutes fins utiles, on rappelle que $\frac{2}{3}$ s'écrit
+$0{.}10101010\ldots$ en binaire.) Que pensez-vous de ce jeu ?
+
+\rightanswer
+Alice a une stratégie gagnante
+
+\answer
+Bob a une stratégie gagnante
+
+\answer
+aucun joueur n'a de stratégie gagnante
+
+\answer
+un joueur a une stratégie gagnante, mais il est impossible de savoir
+lequel
+
+\end{question}
+
+\end{qvar}
+
+
+%
+%
+%
+
+\begin{question}
+
+Soit $A \subseteq \{0,1\}^{\mathbb{N}_{>0}}$ l'ensemble des suites
+binaires $\dblunderline{a} := (a_1,a_2,a_3,\ldots)$ (indicées par
+$\mathbb{N}_{>0} = \{1,2,3,4,\ldots\}$) ayant la propriété suivante :
+il existe $k\geq 1$ tel que $a_{k+i} = a_i$ pour tout $1\leq i\leq k$,
+autrement dit, les $k$ premiers termes de la suite $(a_1,\ldots,a_k)$
+sont immédiatement répétés en $(a_{k+1},\ldots,a_{2k})$. (À titre
+d'exemple, toute suite commençant par $(0,1,0,1,\ldots)$ appartient
+à $A$ puisque les $k=2$ premiers termes sont immédiatement répétés.)
+
+Alice et Bob jouent au jeu de type Gale-Stewart suivant : chacun à son
+tour choisit un chiffre binaire (soit $0$ soit $1$ : Alice choisit
+$a_1$ puis Bob choisit $a_2$ puis Alice choisit $a_3$ et ainsi de
+suite). Au bout d'un nombre infini de tours, Alice gagne si la suite
+$\dblunderline{a} := (a_1,a_2,a_3,\ldots)$ appartient à $A$, tandis
+que Bob gagne dans le cas contraire. (Bref, Alice cherche à ce qu'il
+existe $k\geq 1$ tel que les $k$ premiers termes de la suite se
+répètent immédiatement, Bob cherche à empêcher ce fait.)
+
+Que pensez-vous de cette partie $A$ et de ce jeu ?
+
+\rightanswer
+$A$ est ouvert mais n'est pas fermé ; Bob a une stratégie gagnante
+
+\answer
+$A$ est fermé mais n'est pas ouvert ; Bob a une stratégie gagnante
+
+\answer
+$A$ est ouvert mais n'est pas fermé ; Alice a une stratégie gagnante
+
+\answer
+$A$ est fermé mais n'est pas ouvert ; Alice a une stratégie gagnante
+
+\end{question}
+
+
+%
+%
+%
+
+\begin{qvar}
+
+\begin{question}
+
+Quel joueur a une stratégie gagnante dans la configuration du jeu de
+nim où il y a $5$, $7$, $9$ et $11$ bâtonnets sur les (quatre) lignes
+du jeu ?
+
+\rightanswer
+le second joueur (celui qui vient de jouer)
+
+\answer
+le premier joueur (celui qui doit jouer)
+
+\end{question}
+
+\begin{question}
+
+Quel joueur a une stratégie gagnante dans la configuration du jeu de
+nim où il y a $7$, $9$, $11$ et $13$ bâtonnets sur les (quatre) lignes
+du jeu ?
+
+\rightanswer
+le premier joueur (celui qui doit jouer)
+
+\answer
+le second joueur (celui qui vient de jouer)
+
+\end{question}
+
+\end{qvar}
+
+
+%
+%
+%
+
+\begin{qvar}
+
+\begin{question}
+
+C'est à votre tour de jouer au jeu de nim dans une configuration où il
+y a $1$, $4$, $10$ et $12$ bâtonnets sur les (quatre) lignes du jeu.
+Que faites-vous (pour suivre la stratégie gagnante) ?
+
+\rightanswer
+retirer $1$ bâtonnet de la ligne qui en a $10$ (qui passe donc à $9$)
+
+\answer
+retirer $3$ bâtonnets de la ligne qui en a $10$ (qui passe donc à $7$)
+
+\answer
+retirer le seul bâtonnet de la ligne qui en a $1$ (qui disparaît donc)
+
+\answer
+retirer $3$ bâtonnets de la ligne qui en a $12$ (qui passe donc à $9$)
+
+\end{question}
+
+\begin{question}
+
+C'est à votre tour de jouer au jeu de nim dans une configuration où il
+y a $1$, $6$, $8$ et $12$ bâtonnets sur les (quatre) lignes du jeu.
+Que faites-vous (pour suivre la stratégie gagnante) ?
+
+\rightanswer
+retirer $1$ bâtonnet de la ligne qui en a $6$ (qui passe donc à $5$)
+
+\answer
+retirer $3$ bâtonnets de la ligne qui en a $6$ (qui passe donc à $3$)
+
+\answer
+retirer le seul bâtonnet de la ligne qui en a $1$ (qui disparaît donc)
+
+\answer
+retirer $3$ bâtonnets de la ligne qui en a $8$ (qui passe donc à $5$)
+
+\end{question}
+
+\end{qvar}
+
+
+%
+%
+%
+
+\begin{qvar}
+
+\begin{question}
+
+On considère le jeu combinatoire (impartial, à information parfaite)
+associé au graphe orienté acyclique représenté ci-dessous, la position
+de départ étant notée $s$ :
+
+\begin{center}
+\begin{tikzpicture}[>=stealth,thick,text width=5bp,text height=5bp,text depth=0bp]
+\node (n00) at (0bp,0bp) [draw,circle] {};
+\node (n01) at (40bp,0bp) [draw,circle] {};
+\node (n02) at (80bp,0bp) [draw,circle] {};
+\node (n10) at (0bp,-40bp) [draw,circle] {};
+\node (n11) at (40bp,-40bp) [draw,circle] {};
+\node (n12) at (80bp,-40bp) [draw,circle] {};
+\node (n20) at (0bp,-80bp) [draw,circle] {};
+\node (n21) at (40bp,-80bp) [draw,circle] {};
+\node (n22) at (80bp,-80bp) [draw,circle] {$s$};
+\draw[->] (n01) -- (n00); \draw[->] (n02) -- (n01);
+\draw[->] (n11) -- (n10); \draw[->] (n12) -- (n11);
+\draw[->] (n21) -- (n20); \draw[->] (n22) -- (n21);
+\draw[->] (n11) -- (n00); \draw[->] (n12) -- (n01);
+\draw[->] (n21) -- (n10); \draw[->] (n22) -- (n11);
+\draw[->] (n10) -- (n00); \draw[->] (n20) -- (n10);
+\draw[->] (n11) -- (n01); \draw[->] (n21) -- (n11);
+\draw[->] (n12) -- (n02); \draw[->] (n22) -- (n12);
+\end{tikzpicture}
+\end{center}
+
+Quelle est la valeur de Grundy du jeu (i.e., celle de la
+position $s$) ?
+
+\rightanswer
+$0$
+
+\answer
+$1$
+
+\answer
+$2$
+
+\answer
+$3$
+
+\answer
+$4$
+
+\end{question}
+
+\begin{question}
+
+On considère le jeu combinatoire (impartial, à information parfaite)
+associé au graphe orienté acyclique représenté ci-dessous, la position
+de départ étant notée $s$ :
+
+\begin{center}
+\begin{tikzpicture}[>=stealth,thick,text width=5bp,text height=5bp,text depth=0bp]
+\node (n00) at (0bp,0bp) [draw,circle] {};
+\node (n01) at (40bp,0bp) [draw,circle] {};
+\node (n02) at (80bp,0bp) [draw,circle] {};
+\node (n10) at (0bp,-40bp) [draw,circle] {};
+\node (n11) at (40bp,-40bp) [draw,circle] {};
+\node (n12) at (80bp,-40bp) [draw,circle] {};
+\node (n20) at (0bp,-80bp) [draw,circle] {};
+\node (n21) at (40bp,-80bp) [draw,circle] {};
+\node (n22) at (80bp,-80bp) [draw,circle] {$s$};
+\draw[->] (n01) -- (n00); \draw[->] (n02) -- (n01);
+\draw[->] (n11) -- (n10); \draw[->] (n12) -- (n11);
+\draw[->] (n21) -- (n20); \draw[->] (n22) -- (n21);
+\draw[->] (n21) -- (n10); \draw[->] (n22) -- (n11);
+\draw[->] (n10) -- (n00); \draw[->] (n20) -- (n10);
+\draw[->] (n11) -- (n01); \draw[->] (n21) -- (n11);
+\draw[->] (n12) -- (n02); \draw[->] (n22) -- (n12);
+\end{tikzpicture}
+\end{center}
+
+Quelle est la valeur de Grundy du jeu (i.e., celle de la
+position $s$) ?
+
+\rightanswer
+$3$
+
+\answer
+$0$
+
+\answer
+$1$
+
+\answer
+$2$
+
+\answer
+$4$
+
+\end{question}
+
+\end{qvar}
+
+
+%
+%
+%
+
+\begin{question}
+
+Quel joueur a une stratégie gagnante dans le jeu combinatoire
+(impartial, à information parfaite) associé au graphe orienté
+acyclique représenté ci-dessous, la position de départ étant
+notée $s$ ?
+
+\begin{center}
+\begin{tikzpicture}[>=stealth,thick,text width=5bp,text height=5bp,text depth=0bp]
+\node (l0) at (-40bp,0bp) [draw,circle] {};
+\node (c0) at (0bp,0bp) [draw,circle] {};
+\node (r0) at (40bp,0bp) [draw,circle] {};
+\node (l1) at (-40bp,-40bp) [draw,circle] {};
+\node (c1) at (0bp,-40bp) [draw,circle] {};
+\node (r1) at (40bp,-40bp) [draw,circle] {};
+\node (l2) at (-40bp,-80bp) [draw,circle] {};
+\node (c2) at (0bp,-80bp) [draw,circle] {};
+\node (r2) at (40bp,-80bp) [draw,circle] {};
+\node (l3) at (-40bp,-120bp) [draw,circle] {};
+\node (c3) at (0bp,-120bp) [draw,circle] {};
+\node (r3) at (40bp,-120bp) [draw,circle] {};
+\node (l4) at (-40bp,-160bp) [draw,circle] {};
+\node (c4) at (0bp,-160bp) [draw,circle] {};
+\node (r4) at (40bp,-160bp) [draw,circle] {};
+\node (c5) at (0bp,-200bp) [draw,circle] {$s$};
+\draw[->] (l0) -- (c0); \draw[->] (r0) -- (c0);
+\draw[->] (l1) -- (l0); \draw[->] (l1) -- (c0);
+\draw[->] (c1) -- (l0); \draw[->] (c1) -- (r0);
+\draw[->] (r1) -- (r0); \draw[->] (r1) -- (c0);
+\draw[->] (l1) -- (c1); \draw[->] (r1) -- (c1);
+\draw[->] (l2) -- (l1); \draw[->] (l2) -- (c1);
+\draw[->] (c2) -- (l1); \draw[->] (c2) -- (r1);
+\draw[->] (r2) -- (r1); \draw[->] (r2) -- (c1);
+\draw[->] (l2) -- (c2); \draw[->] (r2) -- (c2);
+\draw[->] (l3) -- (l2); \draw[->] (l3) -- (c2);
+\draw[->] (c3) -- (l2); \draw[->] (c3) -- (r2);
+\draw[->] (r3) -- (r2); \draw[->] (r3) -- (c2);
+\draw[->] (l3) -- (c3); \draw[->] (r3) -- (c3);
+\draw[->] (l4) -- (l3); \draw[->] (l4) -- (c3);
+\draw[->] (c4) -- (l3); \draw[->] (c4) -- (r3);
+\draw[->] (r4) -- (r3); \draw[->] (r4) -- (c3);
+\draw[->] (l4) -- (c4); \draw[->] (r4) -- (c4);
+\draw[->] (c5) -- (l4); \draw[->] (c5) -- (r4);
+\end{tikzpicture}
+\end{center}
+
+\rightanswer
+le second joueur
+
+\answer
+le premier joueur
+
+\answer
+aucun des deux
+
+\end{question}
+
+
+%
+%
+%
+
+\begin{question}
+
+Alice et Bob jouent au jeu suivant sur un échiquier $8\times 8$ sur
+lequel est positionné un unique pion (commun aux deux joueurs) :
+\begin{itemize}
+\item le pion démarre sur la case au coin sud-est de l'échiquier, et
+ Alice joue en premier,
+\item chaque joueur, tour à tour, déplace le pion d'une seule case,
+ soit vers le nord, soit vers l'ouest, soit (en diagonale) vers le
+ nord-ouest, mais sans dépasser les limites de l'échiquier,
+\item celui qui atteint la case au coin nord-ouest de l'échiquier a
+ \emph{gagné} (il revient au même de dire que le joueur qui ne peut
+ plus jouer a perdu).
+\end{itemize}
+
+Que pensez-vous de ce jeu ? (On pourra par exemple calculer de proche
+en proche la valeur de Grundy, ou le type $\mathtt{P}$ ou
+$\mathtt{N}$, de chacune des $64$ positions possibles.)
+
+\rightanswer
+Alice a une stratégie gagnante, et (pour la suivre) doit commencer par
+déplacer le pion en diagonale (d'une case vers le nord-ouest)
+
+\answer
+Alice a une stratégie gagnante, et (pour la suivre) doit commencer par
+déplacer le pion d'une case vers le nord ou, indifféremment, vers
+l'ouest
+
+\answer
+Alice a une stratégie gagnante, et (pour la suivre) peut commencer par
+un coup quelconque
+
+\answer
+Bob a une stratégie gagnante
+
+\end{question}
+
+
+%
+%
+%
+
+\begin{question}
+
+On considère une variante du jeu de nim, mais avec la différence qu'on
+peut retirer \emph{un ou deux} bâtonnets d'une seule ligne (et dans la
+limite du nombre effectivement présent sur cette ligne !). Par
+exemple, à partir de la position $(1,2,3)$ (c'est-à-dire la position
+dans laquelle il y $1$ bâtonnet sur une ligne, $2$ sur une autre, et
+$3$ sur la troisième), on pourrait aller en $(0,2,3)$ ou $(1,1,3)$ ou
+$(1,0,3)$ ou $(1,2,2)$ ou $(1,2,1)$ mais pas en $(1,2,0)$. Comme
+d'habitude, le jeu se termine quand un joueur ne peut plus jouer
+(c'est-à-dire quand il n'y a plus de bâtonnets), et le joueur qui
+devait jouer a alors perdu.
+
+Laquelle des descriptions suivantes définit la stratégie gagnante de
+ce jeu ? (On pourra commencer par la valeur de Grundy de la position
+où il y a une unique ligne avec $n$ bâtonnets, et se rappeler que la
+valeur de Grundy de la somme de nim de deux jeux est la somme de nim
+des valeurs de Grundy.)
+
+\rightanswer
+jouer de manière à ce qu'il y ait un nombre pair de lignes ayant un
+nombre de bâtonnets congru à $1$ modulo $3$ et aussi un nombre pair de
+lignes ayant un nombre de bâtonnets congru à $2$ modulo $3$
+
+\answer
+jouer de manière à ce qu'il y ait un nombre impair de lignes ayant un
+nombre de bâtonnets congru à $1$ modulo $3$ et aussi un nombre pair de
+lignes ayant un nombre de bâtonnets congru à $2$ modulo $3$
+
+\answer
+jouer de manière à ce qu'il y ait un nombre pair de lignes ayant un
+nombre de bâtonnets non multiple de $3$
+
+\answer
+jouer de manière à ce que le nombre total de bâtonnets (sur toutes les
+lignes) soit multiple de $3$
+
+\end{question}
+
+
+%
+%
+%
+
+\begin{question}
+
+On considère le jeu suivant : on a une (unique) ligne de bâtonnets, et
+chaque joueur, quand vient son tour, peut retirer un nombre de
+bâtonnets égal à une puissance de $2$ (c'est-à-dire que s'il y a $n$
+bâtonnets avant de jouer, il en laisse $n-2^k$ pour un certain $k$
+entier avec $2^k \leq n$ ; par exemple, s'il y a $17$ bâtonnets, on
+peut en laisser $16$, $15$, $13$, $7$ ou $1$). Comme d'habitude, le
+jeu se termine quand un joueur ne peut plus jouer (c'est-à-dire quand
+il n'y a plus de bâtonnets), et le joueur qui devait jouer a alors
+perdu.
+
+Laquelle des suites suivantes donne la valeur de Grundy de la position
+où il y a $n$ bâtonnets (pour $n=0,1,2,3,\ldots$) ?
+
+\rightanswer
+$0, 1, 2, 0, 1, 2, 0, 1, 2, 0, 1, 2\ldots$
+
+\answer
+$0, 1, 2, 3, 4, 5, 6, 7, 8, 9, 10, 11\ldots$
+
+\answer
+$0, 1, 0, 1, 0, 1, 0, 1, 0, 1, 0, 1\ldots$
+
+\answer
+$0, 1, 1, 2, 1, 2, 2, 3, 1, 2, 2, 3\ldots$
+
+\answer
+$0, 1, 1, 1, 1, 1, 1, 1, 1, 1, 1, 1\ldots$
+
+\answer
+$0, 1, 1, 0, 1, 0, 0, 1, 1, 0, 0, 1\ldots$
+
+\end{question}
+
+
+%
+%
+%
+
+\begin{question}
+
+On considère le jeu suivant : l'état du jeu est formé d'un certain
+nombre de tas de pierres, chaque tas comportant $\geq 1$ pierre.
+Chaque joueur, quand vient son tour, choisit un tas ayant $\geq 2$
+pierres et le scinde en deux tas ayant chacun $\geq 1$ pierres
+(autrement dit, il remplace un tas de $n \geq 2$ pierres par deux tas
+ayant $n_1$ et $n_2$ pierres, avec $n = n_1 + n_2$ et $n_1\geq 1$ et
+$n_2 \geq 1$). En particulier, le nombre total de pierres ne change
+jamais. Comme d'habitude, le jeu se termine quand un joueur ne peut
+plus jouer (c'est-à-dire quand il n'y a plus que des tas de
+$1$ pierre), et le joueur qui devait jouer a alors perdu.
+
+Quelle formule de récurrence permet de calculer la fonction de Grundy
+$f(n) := \gr(H_n)$ de l'état $H_n$ du jeu ayant un unique tas de $n$
+pierres ?
+
+\rightanswer
+$f(1) = 0$ et $f(n) = \mex\{f(k)\oplus f(n-k) : 1\leq k\leq n-1\}$
+
+\answer
+$f(1) = 0$ et $f(n) = \mex\{f(k) : 1\leq k\leq n-1\}$
+
+\answer
+$f(1) = 0$ et $f(n) = \mex\{f(k) + f(n-k) : 1\leq k\leq n-1\}$
+
+\answer
+$f(1) = 0$ et $f(n) = \mex\{f(k_1) \oplus \cdots \oplus f(k_r) : k_1,\ldots,k_r \geq 1 \ \text{et}\ k_1+\cdots+k_r = n\}$
+
+\end{question}
+
+
+%
+%
+%
+
+\begin{qvar}
+
+\begin{question}
+
+Lequel des ordinaux suivants est le plus grand ?
+
+\rightanswer
+$(\omega^{(\omega^{(\omega\cdot 4)})\cdot 2})\cdot 3$
+
+\answer
+$(\omega^{(\omega^{(\omega\cdot 3)})\cdot 4})\cdot 2$
+
+\answer
+$(\omega^{(\omega^{(\omega\cdot 2)})\cdot 3})\cdot 4$
+
+\end{question}
+
+\begin{question}
+
+Lequel des ordinaux suivants est le plus grand ?
+
+\rightanswer
+$(\omega^{(\omega^{(\omega\cdot 2)})\cdot 2})+ 2$
+
+\answer
+$(\omega^{(\omega^{(\omega\cdot 2)})+ 2})\cdot 2$
+
+\answer
+$(\omega^{(\omega^{(\omega+ 2)})\cdot 2})\cdot 2$
+
+\end{question}
+
+\end{qvar}
+
+
+%
+%
+%
+
+\begin{question}
+
+Auquel ordinaux suivants est égal $\omega + \omega^2 +
+\omega^\omega$ ?
+
+\rightanswer
+$\omega^\omega$
+
+\answer
+$\omega^\omega + \omega^2 + \omega$
+
+\answer
+$\omega$
+
+\answer
+$\omega^{\omega+1}$
+
+\answer
+$\omega^\omega\cdot 2$
+
+\end{question}
+
+
+%
+%
+%
+
+\begin{question}
+
+Auquel ordinaux suivants est égal $\omega \cdot \omega^2 \cdot
+\omega^\omega$ (produit de $\omega$, de $\omega^2$ et de
+$\omega^\omega$ dans cet ordre) ?
+
+\rightanswer
+$\omega^\omega$
+
+\answer
+$\omega^{\omega+3}$
+
+\answer
+$\omega$
+
+\answer
+$\omega^{\omega+1}$
+
+\answer
+$\omega^{\omega\cdot 2}$
+
+\end{question}
+
+
+%
+%
+%
+
+\begin{question}
+
+Auquel ordinaux suivants est égal $2^{\omega\cdot 2}$ (lire :
+$2$ puissance $\omega\cdot 2$) ?
+
+\rightanswer
+$\omega^2$
+
+\answer
+$\omega$
+
+\answer
+$\omega^\omega$
+
+\answer
+$\omega^{\omega^2}$
+
+\answer
+$\omega^{\omega^\omega}$
+
+\end{question}
+
+
+%
+%
+%
+
+\begin{question}
+
+Sachant que $\varepsilon_0 = \omega^{\varepsilon_0}$, auquel ordinaux
+suivants est égal $\varepsilon_0^{\varepsilon_0}$ ?
+
+\rightanswer
+$\omega^{\omega^{(\varepsilon_0\cdot 2)}}$ (lire : $\omega$ puissance
+[$\omega$ puissance $\varepsilon_0\cdot 2$])
+
+\answer
+$\omega^{\omega^{(\varepsilon_0^2)}}$ (lire : $\omega$ puissance
+[$\omega$ puissance $\varepsilon_0^2$])
+
+\answer
+$\omega^{(\varepsilon_0\cdot 2)}$ (lire : $\omega$ puissance
+$\varepsilon_0\cdot 2$)
+
+\answer
+$\omega^{(\varepsilon_0 + 1)}$ (lire : $\omega$ puissance
+$\varepsilon_0 + 1$)
+
+\end{question}
+
+
+%
+%
+%
+
+\begin{question}
+
+Patience et Raoul jouent au jeu suivant : l'état du jeu est constitué
+d'un certain nombre de tas de pierres, les tas étant numérotés
+$0,1,2,3\ldots,r$ ; à chaque tour de jeu, Patience retire une pierre
+d'un des tas, disons du tas numéroté $k$, puis Raoul rajoute autant de
+pierre qu'il le souhaite sur les tas de numéro $<k$ (si Patience a
+retié une pierre du tas $0$, Raoul ne joue pas). Patience gagne si
+elle finit par retirer toutes les pierres, tandis que Raoul gagne si
+cela ne se produit jamais (i.e., si le jeu dure infiniment longtemps).
+
+On souhaite montrer que Patience gagne forcément, quoi qu'elle fasse
+et quoi que fasse Raoul. Quel ordinal proposez-vous d'associer à
+l'état du jeu où il y a $n_i$ pierres dans le tas $i$ (i.e., $n_0$
+pierres dans le tas numéroté $0$ et $n_1$ dans le tas numéroté $1$,
+etc.), de manière à s'assurer qu'il décroisse forcément ?
+
+\rightanswer
+$\omega^r\, n_r + \omega^{r-1}\, n_{r-1} + \cdots + \omega n_1 + n_0$
+
+\answer
+$\omega^{n_r}\, r + \omega^{n_{r-1}}\, (r-1) + \cdots + \omega^{n_1}$
+
+\answer
+$\omega^{n_r} + \omega^{n_{r-1}} + \cdots + \omega^{n_1} + \omega^{n_0}$
+
+\answer
+$n_0 + \omega\, n_1 + \cdots + \omega^{r-1}\, n_{r-1} + \omega^r\, n_r$
+
+\answer
+$\omega^{n_1} + \cdots + \omega^{n_{r-1}}\, (r-1) + \omega^{n_r}\, r$
+
+\answer
+$\omega^{n_0} + \omega^{n_1} + \cdots + \omega^{n_{r-1}} + \omega^{n_r}$
+
+\end{question}
+
+
+\end{qcm}
+%
+%
+%
+\end{document}
diff --git a/controle-20210412.tex b/controle-20210412.tex
new file mode 100644
index 0000000..8440d92
--- /dev/null
+++ b/controle-20210412.tex
@@ -0,0 +1,887 @@
+%% This is a LaTeX document. Hey, Emacs, -*- latex -*- , get it?
+\documentclass[12pt,a4paper]{article}
+\usepackage[francais]{babel}
+\usepackage[utf8]{inputenc}
+\usepackage[T1]{fontenc}
+%\usepackage{ucs}
+\usepackage{times}
+% A tribute to the worthy AMS:
+\usepackage{amsmath}
+\usepackage{amsfonts}
+\usepackage{amssymb}
+\usepackage{amsthm}
+%
+\usepackage{mathrsfs}
+\usepackage{wasysym}
+\usepackage{url}
+%
+\usepackage{graphics}
+\usepackage[usenames,dvipsnames]{xcolor}
+\usepackage{tikz}
+\usetikzlibrary{matrix,calc}
+\usepackage{hyperref}
+%
+%\externaldocument{notes-mitro206}[notes-mitro206.pdf]
+%
+\theoremstyle{definition}
+\newtheorem{comcnt}{Tout}
+\newcommand\thingy{%
+\refstepcounter{comcnt}\smallskip\noindent\textbf{\thecomcnt.} }
+\newcommand\exercice{%
+\refstepcounter{comcnt}\bigskip\noindent\textbf{Exercice~\thecomcnt.}\par\nobreak}
+\renewcommand{\qedsymbol}{\smiley}
+%
+\newcommand{\outnb}{\operatorname{outnb}}
+\newcommand{\downstr}{\operatorname{downstr}}
+\newcommand{\precs}{\operatorname{precs}}
+\newcommand{\mex}{\operatorname{mex}}
+\newcommand{\id}{\operatorname{id}}
+\newcommand{\limp}{\Longrightarrow}
+\newcommand{\gr}{\operatorname{gr}}
+\newcommand{\rk}{\operatorname{rk}}
+\newcommand{\fuzzy}{\mathrel{\|}}
+%
+\DeclareUnicodeCharacter{00A0}{~}
+%
+\DeclareMathSymbol{\tiret}{\mathord}{operators}{"7C}
+\DeclareMathSymbol{\traitdunion}{\mathord}{operators}{"2D}
+%
+\DeclareFontFamily{U}{manual}{}
+\DeclareFontShape{U}{manual}{m}{n}{ <-> manfnt }{}
+\newcommand{\manfntsymbol}[1]{%
+ {\fontencoding{U}\fontfamily{manual}\selectfont\symbol{#1}}}
+\newcommand{\dbend}{\manfntsymbol{127}}% Z-shaped
+\newcommand{\danger}{\noindent\hangindent\parindent\hangafter=-2%
+ \hbox to0pt{\hskip-\hangindent\dbend\hfill}}
+%
+\newcommand{\spaceout}{\hskip1emplus2emminus.5em}
+\newif\ifcorrige
+\corrigetrue
+\newenvironment{corrige}%
+{\ifcorrige\relax\else\setbox0=\vbox\bgroup\fi%
+\smallbreak\noindent{\underbar{\textit{Corrigé.}}\quad}}
+{{\hbox{}\nobreak\hfill\checkmark}%
+\ifcorrige\par\smallbreak\else\egroup\par\fi}
+%
+%
+%
+\begin{document}
+\ifcorrige
+\title{MITRO206\\Contrôle de connaissances — Corrigé\\{\normalsize Théories des jeux}}
+\else
+\title{MITRO206\\Contrôle de connaissances\\{\normalsize Théories des jeux}}
+\fi
+\author{}
+\date{12 avril 2021}
+\maketitle
+
+\pretolerance=8000
+\tolerance=50000
+
+\vskip1truein\relax
+
+\noindent\textbf{Consignes.}
+
+Les exercices sont totalement indépendants. Ils pourront être traités
+dans un ordre quelconque, mais on demande de faire apparaître de façon
+très visible dans les copies où commence chaque exercice.
+
+La longueur du sujet ne doit pas effrayer : d'une part, l'énoncé est
+long parce que des rappels ont été faits et que la rédaction des
+questions cherche à éviter toute ambiguïté ; d'autre part, il ne sera
+pas nécessaire de tout traiter pour obtenir la totalité des points.
+
+Les remarques en petits caractères ne font pas partie du sujet et
+peuvent être ignorées.
+
+\medbreak
+
+L'usage de tous les documents (notes de cours manuscrites ou
+imprimées, feuilles d'exercices, livres) est autorisé.
+
+L'usage des appareils électroniques est interdit.
+
+\medbreak
+
+Durée : 2h
+
+Barème \emph{indicatif} : chaque question numérotée aura
+approximativement la même valeur (environ $1$ à $1.5$ points).
+
+\ifcorrige
+Ce corrigé comporte 10 pages (page de garde incluse).
+\else
+Cet énoncé comporte 5 pages (page de garde incluse).
+\fi
+
+\vfill
+{\noindent\tiny
+\immediate\write18{sh ./vc > vcline.tex}
+Git: \input{vcline.tex}
+\immediate\write18{echo ' (stale)' >> vcline.tex}
+\par}
+
+\pagebreak
+
+
+%
+%
+%
+
+\exercice
+
+On considère le jeu en forme normale, à deux joueurs, à somme nulle,
+dont la matrice de gains est la suivante, où $x$ est un réel (la table
+donne les gains d'Alice, qui choisit la ligne, ceux de Bob, qui
+choisit la colonne, sont opposés) :
+
+\begin{center}
+\begin{tabular}{r|rrrr}
+$\downarrow$Alice, Bob$\rightarrow$&$\mathrm{P}$&$\mathrm{Q}$&$\mathrm{R}$&$\mathrm{S}$\\\hline
+$\mathrm{P}$&$0$&$1$&$-1$&$x$\\
+$\mathrm{Q}$&$-1$&$0$&$2$&$-1$\\
+$\mathrm{R}$&$1$&$-2$&$0$&$-1$\\
+$\mathrm{S}$&$-x$&$1$&$1$&$0$\\
+\end{tabular}
+\end{center}
+
+On rappelle qu'une \emph{stratégie optimale} est une stratégie mixte
+qui réalise un gain espéré au moins égal à la valeur du jeu contre
+toute stratégie (pure donc mixte) de l'adversaire.
+
+\textbf{(0)} Quelle est la valeur du jeu dans ce cas ? (Ne pas faire de
+calcul !)
+
+\begin{corrige}
+On a affaire à un jeu à somme nulle \emph{symétrique} (c'est-à-dire
+que sa matrice de gains est antisymétrique), donc la valeur du jeu est
+nulle.
+\end{corrige}
+
+\textbf{(1)} À quelle condition sur $x$ la stratégie $\frac{1}{2}\mathrm{P} +
+\frac{1}{4}\mathrm{Q} + \frac{1}{4}\mathrm{R}$ (consistant à choisir P
+avec probabilité $\frac{1}{2}$, et chacun de Q et R avec
+probabilité $\frac{1}{4}$) est-elle optimale ?
+
+\begin{corrige}
+Ajoutons à la matrice des gains du jeu une ligne correspondant à la
+stratégie considérée :
+\begin{center}
+\begin{tabular}{r|rrrr}
+$\downarrow$Alice, Bob$\rightarrow$&$\mathrm{P}$&$\mathrm{Q}$&$\mathrm{R}$&$\mathrm{S}$\\\hline
+$\mathrm{P}$&$0$&$1$&$-1$&$x$\\
+$\mathrm{Q}$&$-1$&$0$&$2$&$-1$\\
+$\mathrm{R}$&$1$&$-2$&$0$&$-1$\\
+$\mathrm{S}$&$-x$&$1$&$1$&$0$\\\hline
+\hbox{\vrule height12pt depth5pt width0pt}$\frac{1}{2}\mathrm{P} + \frac{1}{4}\mathrm{Q} + \frac{1}{4}\mathrm{R}$&
+$0$&$0$&$0$&$\frac{x-1}{2}$\\
+\end{tabular}
+\end{center}
+Pour qu'elle réalise un gain au moins égal à la valeur du jeu,
+c'est-à-dire $\geq 0$, contre toute stratégie (pure donc mixte) de
+l'adversaire, il faut et il suffit donc que $\frac{x-1}{2} \geq 0$,
+c'est-à-dire $x\geq 1$.
+\end{corrige}
+
+\textbf{(2)} À quelle condition sur $x$ l'expression $\frac{1}{x+2}\,\mathrm{P} +
+\frac{x}{x+2}\,\mathrm{R} + \frac{1}{x+2}\,\mathrm{S}$ (consistant à
+choisir R avec probabilité $\frac{x}{x+2}$, et chacun de P et S avec
+probabilité $\frac{1}{x+2}$) définit-elle une stratégie optimale ?
+
+\begin{corrige}
+L'expression $\frac{1}{x+2}\,\mathrm{P} + \frac{x}{x+2}\,\mathrm{R} +
+\frac{1}{x+2}\,\mathrm{S}$ définit une stratégie (mixte) lorsque ses
+coefficients sont positifs de somme $1$ : le fait qu'ils soient de
+somme $1$ est toujours vrai, et ils sont tous positifs lorsque $x\geq
+0$.
+
+Ajoutons à la matrice des gains du jeu une ligne correspondant à la
+stratégie en question :
+\begin{center}
+\begin{tabular}{r|rrrr}
+$\downarrow$Alice, Bob$\rightarrow$&$\mathrm{P}$&$\mathrm{Q}$&$\mathrm{R}$&$\mathrm{S}$\\\hline
+$\mathrm{P}$&$0$&$1$&$-1$&$x$\\
+$\mathrm{Q}$&$-1$&$0$&$2$&$-1$\\
+$\mathrm{R}$&$1$&$-2$&$0$&$-1$\\
+$\mathrm{S}$&$-x$&$1$&$1$&$0$\\\hline
+\hbox{\vrule height12pt depth5pt width0pt}$\frac{1}{x+2}\,\mathrm{P} +
+\frac{x}{x+2}\,\mathrm{R} + \frac{1}{x+2}\,\mathrm{S}$&
+$0$&$\frac{2(1-x)}{x+2}$&$0$&$0$\\
+\end{tabular}
+\end{center}
+Pour qu'elle réalise un gain au moins égal à la valeur du jeu,
+c'est-à-dire $\geq 0$, contre toute stratégie (pure donc mixte) de
+l'adversaire, il faut et il suffit donc que $\frac{2(1-x)}{x+2} \geq
+0$, c'est-à-dire $x\leq 1$, donc finalement $0\leq x\leq 1$.
+\end{corrige}
+
+\textbf{(3)} Donner une stratégie optimale lorsque $x\leq 0$.
+
+\begin{corrige}
+Lorsque $x\leq 0$, la stratégie pure $\mathrm{S}$ est optimale
+puisqu'elle réalise un gain $\geq 0$ contre toute stratégie (pure donc
+mixte) de l'adversaire.
+\end{corrige}
+
+\textbf{(4)} Dans chacun des deux cas $x=0$ et $x=1$, exhiber une infinité de
+stratégies optimales distinctes.
+
+\begin{corrige}
+Lorsque $x = 0$, on a trouvé deux stratégies optimales, à savoir
+$\frac{1}{2}\mathrm{P} + \frac{1}{2}\mathrm{S}$ (trouvée en (2)) et
+$\mathrm{S}$ (trouvée en (3)). Toute combinaison convexe de ces deux
+stratégies optimales est donc encore optimale, c'est-à-dire
+$\frac{t}{2}\,\mathrm{P} + \frac{2-t}{2}\,\mathrm{S}$
+pour $t\in[0;1]$.
+
+Lorsque $x = 1$, on a trouvé deux stratégies optimales, à savoir
+$\frac{1}{2}\mathrm{P} + \frac{1}{4}\mathrm{Q} +
+\frac{1}{4}\mathrm{R}$ (trouvée en (1)) et $\frac{1}{3}\mathrm{P} +
+\frac{1}{3}\mathrm{R} + \frac{1}{3}\mathrm{S}$ (trouvée en (2)).
+Toute combinaison convexe de ces deux stratégies optimales est donc
+encore optimale, c'est-à-dire $\frac{2+t}{6}\,\mathrm{P} +
+\frac{t}{4}\,\mathrm{Q} + \frac{4-t}{12}\,\mathrm{R} +
+\frac{1-t}{3}\,\mathrm{S}$ pour $t\in[0;1]$.
+\end{corrige}
+
+\textbf{(5)} En supposant que $x$ ne soit pas un réel fixé mais \emph{tiré au
+ hasard} selon une loi uniforme entre $0$ et $1$ une fois que les
+joueurs ont joué (autrement dit, si un joueur choisit P et l'autre S,
+le joueur qui a choisi P reçoit un gain aléatoire uniforme entre
+$0$ et $1$ ; cet aléa est, évidemment, indépendant de tous ceux que
+les joueurs auraient pu utiliser dans la détermination de leur propre
+stratégie !), quelle stratégie adopteriez-vous dans ce jeu ?
+
+\begin{corrige}
+On cherche à maximiser le gain \emph{espéré} minimal contre toute
+stratégie de l'adversaire ; mais comme $x$ est indépendant des choix
+qu'ont pu faire les joueurs, on peut le remplacer par son espérance,
+c'est-à-dire que le jeu considéré revient à prendre $x = \frac{1}{2}$.
+D'après la question (2), une stratégie optimale est donnée par
+$\frac{2}{5}\mathrm{P} + \frac{1}{5}\mathrm{R} +
+\frac{2}{5}\,\mathrm{S}$.
+\end{corrige}
+
+
+%
+%
+%
+
+\exercice
+
+Dans cet exercice, on va d'abord définir les ordinaux
+$\varepsilon_\iota$, puis on va s'intéresser à ceux qui
+sont $<\varepsilon_1$. Les parties de cet exercice sont indépendantes
+à l'exception de ce qui est explicitement rappelé.
+
+\medskip
+
+\underline{Première partie.}
+
+\nobreak
+On définit une fonction $\varphi$ des ordinaux vers les ordinaux par
+$\varphi(\alpha) = \omega^\alpha$. On rappelle que $\varphi$ est
+\emph{strictement croissante} (c'est-à-dire que si $\alpha < \beta$
+alors $\varphi(\alpha) < \varphi(\beta)$).
+
+\textbf{(1)} Rappeler pourquoi $\varphi$ est \emph{continue}, ce qui signifie
+par définition la chose suivante : si $\delta$ est un ordinal limite,
+alors $\lim_{\xi\to\delta} \varphi(\xi) = \varphi(\delta)$, où
+$\lim_{\xi\to\delta} \varphi(\xi)$ est une notation pour
+$\sup\{\varphi(\xi) : \xi < \delta\}$ lorsque $\varphi$ est
+croissante.
+
+\begin{corrige}
+La continuité de la fonction $\varphi\colon \alpha \mapsto
+\omega^\alpha$ fait partie de la définition inductive de
+l'exponentiation ordinale ($\omega ^ \delta = \lim_{\xi\to\delta}
+\omega^\xi$ si $\delta$ est limite).
+\end{corrige}
+
+Pour éviter de partir dans des fausses directions, il est conseillé,
+jusqu'à la question (5) incluse, d'oublier la définition de $\varphi$
+et de retenir seulement que $\varphi$ est strictement croissante et
+continue.
+
+\textbf{(2)} Rappeler pourquoi $\varphi(\alpha) \geq \alpha$ pour
+tout $\alpha$.
+
+\begin{corrige}
+On a vu en cours que si $\varphi$ est une fonction strictement
+croissante d'un ensemble bien-ordonné vers lui-même, alors $\varphi(x)
+\geq x$ pour tout $x$ : il suffit d'appliquer ce résultat à la
+fonction $\varphi$. (Pour être tout à fait exact, on veut plutôt
+appliquer la \emph{démonstration} de ce résultat, vu que le résultat
+ne s'applique pas tel quel faute d'un ensemble bien-ordonné auquel
+l'appliquer vu que les ordinaux ne constituent pas un ensemble ; mais
+la démonstration s'applique exactement sans changement : on montre $x
+\leq \varphi(x)$ par induction, en effet, si par l'absurde on avait
+$\varphi(x) < x$, alors l'hypothèse d'induction appliquée à $y :=
+\varphi(x)$ donnerait $\varphi(x) \leq \varphi(\varphi(x))$, tandis
+que la stricte croissance de $\varphi$ appliquée à $\varphi(x) < x$
+donnerait $\varphi(\varphi(x)) < \varphi(x)$, ce qui est une
+contradiction.)
+\end{corrige}
+
+On dira qu'un ordinal $\gamma$ est un \emph{point fixe} de $\varphi$
+lorsque $\varphi(\gamma) = \gamma$.
+
+\textbf{(3)} Soit dans cette question $\alpha$ un ordinal quelconque :
+considérons la suite $(\gamma_n)$ (indicée par les entiers
+naturels $n$) définie par $\gamma_0 = \alpha$ et $\gamma_{n+1} =
+\varphi(\gamma_n)$. Montrer que $(\gamma_n)$ est croissante
+(c'est-à-dire que $m\leq n$ implique $\gamma_m \leq \gamma_n$).
+Montrer que sa limite $\gamma_\omega := \lim_{n\to\omega} \gamma_n :=
+\sup\{\gamma_n : n \in \mathbb{N}\}$ est un point fixe de $\varphi$.
+Montrer qu'il s'agit du plus petit point fixe de $\varphi$ qui
+soit $\geq\alpha$ (c'est-à-dire que si $\delta$ est un point fixe
+de $\varphi$ et $\delta\geq\alpha$ alors $\delta\geq\gamma_\omega$ :
+on pourra pour cela montrer que $\delta\geq\gamma_n$ pour tout $n$).
+
+\begin{corrige}
+On vient de voir que $\varphi(\alpha)\geq\alpha$ pour tout $\alpha$,
+ce qui montre $\gamma_{n+1}\geq\gamma_n$, donc la suite $(\gamma_n)$
+est croissante. (Si on veut vraiment faire la démonstration jusqu'au
+bout : montrons que $m\leq n$ implique $\gamma_m \leq \gamma_n$ par
+récurrence sur $n\geq m$ : pour $n=m$ c'est évident, et si on a
+$\gamma_m \leq \gamma_n$, alors $\gamma_m \leq \gamma_n \leq
+\varphi(\gamma_n) = \gamma_{n+1}$ ce qui conclut la récurrence.)
+
+Montrons maintenant $\varphi(\gamma_\omega) = \gamma_\omega$. Par
+continuité de $\varphi$, on a $\varphi(\lim_{n\to\omega} \gamma_n) =
+\lim_{n\to\omega} \varphi(\gamma_n)$ (pour être tout à fait complet
+dans la démonstration de cette affirmation : $\gamma_\omega$ est par
+définition le plus petit ordinal supérieur ou égal à tous les
+$\gamma_n$ pour $n<\omega$, donc tout ordinal $\zeta<\gamma_\omega$
+est majoré par un $\gamma_n$ pour un certain $n<\omega$, et par
+croissance de $\varphi$ on a alors $\varphi(\zeta)$ majoré par
+$\varphi(\gamma_n)$, donc la borne supérieure des $\varphi(\gamma_n)$
+pour $n<\omega$ est aussi la borne supérieure des $\varphi(\zeta)$
+pour $\zeta<\gamma_\omega$ : or cette dernière borne supérieure est
+$\varphi(\gamma_omega)$ par continuité de $\varphi$, ce qui montre
+$\varphi(\gamma_\omega) = \lim_{n\to\omega} \varphi(\gamma_n)$),
+c'est-à-dire $\varphi(\gamma_\omega) = \lim_{n\to\omega} \gamma_{1+n}
+= \gamma_\omega$, comme affirmé.
+
+Enfin, si $\delta$ est un point fixe de $\varphi$ et $\delta \geq
+\alpha$, alors par récurrence sur $n$ on a $\delta \geq \gamma_n$ pour
+tout $n<\omega$ (le cas $n=0$ est l'hypothèse, et $\delta \geq
+\gamma_n$ implique $\varphi(\delta) \geq \varphi(\gamma_n)$ par
+croissance de $\varphi$, c'est-à-dire $\delta \geq \gamma_{n+1}$), ce
+qui donne $\delta \geq \gamma_\omega$ puisque $\gamma_\omega$ est la
+borne supérieure des $\gamma_n$ pour $n<\omega$.
+\end{corrige}
+
+La question (3) implique notamment : \emph{pour tout ordinal $\alpha$
+ il existe un point fixe de $\varphi$ qui soit $\geq\alpha$}.
+
+On définit maintenant $\varepsilon_\iota$ pour tout ordinal $\iota$
+par :
+\begin{itemize}
+\item $\varepsilon_0$ est le plus petit point fixe de $\varphi$
+ (c'est-à-dire, si on veut, le plus petit point fixe qui soit $\geq
+ 0$) ;
+\item pour $\iota+1$ ordinal successeur, $\varepsilon_{\iota+1}$ est
+ le plus petit point fixe de $\varphi$ qui soit $>\varepsilon_\iota$
+ (c'est-à-dire, si on veut, le plus petit point fixe qui soit $\geq
+ (\varepsilon_\iota)+1$),
+\item pour $\delta$ ordinal limite, $\varepsilon_\delta$ est
+ $\lim_{\xi\to\delta} \varepsilon_\xi$ (c'est-à-dire
+ $\sup\{\varepsilon_\xi : \xi < \delta\}$).
+\end{itemize}
+
+Cette définition a bien un sens d'après ce qu'on vient de dire.
+
+\textbf{(4)} Montrer que $\iota \mapsto \varepsilon_\iota$ est strictement
+croissante. Montrer que $\varepsilon_\delta$ est un point fixe
+de $\varphi$ aussi pour $\delta$ limite (c'est vrai dans les autres
+cas par la définition) : pour cela, on expliquera pourquoi
+$\varphi(\lim_{\xi\to\delta} \varepsilon_\xi) = \lim_{\xi\to\delta}
+\varphi(\varepsilon_\xi)$.
+
+\begin{corrige}
+Remarquons tout d'abord que $\varepsilon_{\iota+1}$ est le plus petit
+point fixe de $\varphi$ qui soit strictement supérieur à
+$\varepsilon_\iota$, et notamment $\varepsilon_{\iota+1} >
+\varepsilon_\iota$, quel que soit $\iota$.
+
+Il est sans doute plus agréable, ensuite, de montrer que $\iota
+\mapsto \varepsilon_\iota$ est croissante à partir du fait que
+$\varepsilon_{\iota+1} \geq \varepsilon_\iota$ et de la continuité aux
+ordinaux limites (on peut légitimement tenir ce fait pour évident vu
+qu'il est sous-entendu par l'énoncé de la question en ce qu'elle
+définit $\lim_{\xi\to\delta} \varepsilon_\xi$ comme
+$\sup\{\varepsilon_\xi : \xi < \delta\}$). Pour cela, on montre par
+induction transfinie sur $\beta$ que $\alpha \leq \beta$ implique
+$\varepsilon_\alpha \leq \varepsilon_\beta$. Or si $\alpha = \beta$
+il n'y a rien à prouver, donc supposons $\alpha < \beta$. Si $\beta$
+est successeur, disons $\beta = \gamma+1$, alors $\alpha < \beta$,
+signifie $\alpha \leq \gamma$, auquel cas l'hypothèse d'induction
+(comme $\gamma<\beta$) donne $\varepsilon_\alpha \leq
+\varepsilon_\gamma$ et d'après la remarque qu'on a faite,
+$\varepsilon_\alpha \leq \varepsilon_\gamma \leq
+\varepsilon_{\gamma+1} = \varepsilon_\beta$ comme on voulait. Enfin,
+si $\beta$ est limite, alors $\varepsilon_\beta$ est défini comme la
+borne supérieure des $\varepsilon_\xi$ pour $\xi<\beta$, mais en
+particulier $\varepsilon_\alpha$ est dans cet ensemble dont on a pris
+la borne supérieure, donc $\varepsilon_\alpha \leq \varepsilon_\beta$.
+
+Ensuite, pour passer de croissant à strictement croissant, il suffit
+de dire que $\alpha < \beta$ équivaut à $\alpha+1 \leq \beta$, la
+croissance donne $\varepsilon_{\alpha+1} \leq \varepsilon_\beta$, et
+on a déjà remarqué $\varepsilon_\alpha < \varepsilon_{\alpha+1}$, d'où
+$\varepsilon_\alpha < \varepsilon_\beta$.
+
+Montrons enfin que $\varepsilon_\delta$ est point fixe de $\varphi$
+pour $\delta$ limite. Le point crucial est qu'on a
+$\varphi(\lim_{\xi\to\delta} \varepsilon_\xi) = \lim_{\xi\to\delta}
+\varphi(\varepsilon_\xi)$ par continuité de $\varphi$ (pour etre tout
+à fait complet dans la démonstration de cette affirmation :
+$\varepsilon_\delta$ est par définition le plus petit ordinal
+supérieur ou égal à tous les $\varepsilon_\xi$ pour $\xi<\delta$, donc
+tout ordinal $\zeta<\varepsilon_\delta$ est majoré par un
+$\varepsilon_\xi$ pour un certain $\xi<\delta$, et par croissance de
+$\varphi$ on a alors $\varphi(\zeta)$ majoré par
+$\varphi(\varepsilon_\xi)$, donc la borne supérieure des
+$\varphi(\varepsilon_\xi)$ pour $\xi<\delta$ est aussi la borne
+supérieure des $\varphi(\zeta)$ pour $\zeta<\varepsilon_\delta$ : or
+cette dernière borne supérieure est $\varphi(\varepsilon_\delta)$ par
+continuité de $\varphi$, ce qui montre $\varphi(\varepsilon_\delta) =
+\lim_{\xi\to\delta} \varphi(\varepsilon_\xi)$). On montre alors
+$\varphi(\varepsilon_\iota) = \varepsilon_\iota$ par induction
+sur $\xi$ : le cas successeur étant contenu dans la définiton même
+de $\varepsilon$, il n'y a qu'à traiter le cas limite, et on a alors
+$\varphi(\varepsilon_\delta) = \varphi(\lim_{\xi\to\delta}
+\varepsilon_\xi) = \lim_{\xi\to\delta} \varphi(\varepsilon_\xi) =
+\lim_{\xi\to\delta} \varepsilon_\xi = \varepsilon_\delta$.
+\end{corrige}
+
+\textbf{(5)} Montrer que tout ordinal $\gamma$ qui est un point fixe de
+$\varphi$ est de la forme $\varepsilon_\alpha$ pour un certain
+ordinal $\alpha$ (on pourra montrer qu'il existe $\alpha$ tel que
+$\varepsilon_\alpha\geq\gamma$ puis considérer le plus petit
+tel $\alpha$).
+
+\begin{corrige}
+Pour la même raison qu'en (2), on a $\varepsilon_\alpha \geq \alpha$
+pour tout $\alpha$. Donné un point fixe $\gamma$ de $\varphi$, on
+sait donc qu'il existe $\alpha$ tel que $\varepsilon_\alpha \geq
+\gamma$ (on vient de voir que $\alpha=\gamma$ convient). Considérons
+le plus petit tel $\alpha$ : ceci a un sens car les ordinaux sont
+bien-ordonnés. On veut montrer qu'on a $\varepsilon_\alpha = \gamma$.
+Si $\alpha$ est successeur, disons $\alpha = \beta+1$, alors
+$\varepsilon_\beta < \gamma$ par minimalité de $\alpha$, mais comme
+$\gamma$ est un point fixe de $\varphi$ et que $\varepsilon_{\beta+1}$
+est le plus petit point fixe après $\varepsilon_\beta$, on doit avoir
+$\varepsilon_{\beta+1} \leq \gamma$, soit $\varepsilon_\alpha \leq
+\gamma$, et on a bien prouvé $\varepsilon_\alpha = \gamma$. Si
+$\alpha$ est limite, en revanche, par minimalité de $\alpha$, on a
+$\varepsilon_\xi < \gamma$ pour tout $\xi<\alpha$; or
+$\varepsilon_\alpha$ a été défini comme la borne supérieure de
+ces $\varepsilon_\xi$, donc on a $\varepsilon_\alpha \leq \gamma$, et
+on a bien prouvé $\varepsilon_\alpha = \gamma$.
+\end{corrige}
+
+{\footnotesize\textit{Remarque.} On a donc démontré que la fonction
+ $\varphi(1,\tiret) \colon \alpha \mapsto \varepsilon_\alpha$, qui
+ énumère les points fixes de $\varphi(0,\tiret) = \varphi \colon
+ \alpha \mapsto \omega^\alpha$ strictement croissante continue, est
+ elle-même strictement croissante et continue. On pourrait donc
+ continuer le procédé et appeler $\varphi(2,\tiret)$ la fonction
+ énumérant les points fixes de $\varphi(1,\tiret)$ (c'est-à-dire que
+ $\varphi(2,0)$ est le plus petit ordinal $\zeta$ tel que $\zeta =
+ \varepsilon_\zeta$ puis $\varphi(2,1)$ est le suivant, etc.), « et
+ ainsi de suite ». Ce procédé de construction d'ordinaux s'appelle
+ les « fonctions de Veblen » : on peut bien sûr continuer en
+ définissant $\varphi(1,0,0)$ comme le premier ordinal $\delta$ tel
+ que $\delta = \varphi(\delta,0)$ et au-delà.\par}
+
+\medskip
+
+\underline{Deuxième partie.}
+
+\nobreak
+On appelle $\varepsilon_0$ le plus petit ordinal tel que $\varepsilon
+= \omega^\varepsilon$, et $\varepsilon_1$ le suivant, c'est-à-dire le
+plus petit ordinal $>\varepsilon_0$ vérifiant cette même équation
+$\varepsilon = \omega^\varepsilon$ (l'existence de ces ordinaux
+résulte de la première partie de cet exercice).
+
+On a vu en cours que les ordinaux $<\varepsilon_0$ possèdent une
+représentation unique sous forme normale de Cantor itérée, et que
+celle-ci permet de les comparer, de les ajouter et de les multiplier.
+On va s'intéresser ici aux ordinaux $<\varepsilon_1$, et leur donner
+\emph{deux} systèmes différents d'écriture, qu'on appellera
+« écriture 1 » et « écriture 2 ».
+
+\textbf{(6)} Soit $\alpha < \varepsilon_1$ un ordinal différent
+de $\varepsilon_0$ : montrer que dans sa forme normale de Cantor
+$\omega^{\gamma_s} n_s + \cdots + \omega^{\gamma_1} n_1$, tous les
+exposants $\gamma_i$ sont $<\alpha$ (on pourra utiliser le fait,
+démontré en (2), que $\omega^\gamma \geq \gamma$ pour tout
+ordinal $\gamma$).
+
+\begin{corrige}
+Notons pour commencer que $\omega^{\gamma_i} \leq \alpha$ (ceci
+résulte de la comparaison entre les formes normales de Cantor). Si on
+avait $\gamma_i \geq \alpha$ alors on aurait $\alpha \geq
+\omega^{\gamma_i} \geq \gamma_i \geq \alpha$ : donc en fait toutes ces
+inégalités sont des égalités et $\alpha = \gamma_i =
+\omega^{\gamma_i}$ est un point fixe de la fonction $\gamma \mapsto
+\omega^\gamma$, ce qui, pour $\alpha < \varepsilon_1$, n'est possible
+que lorsque $\alpha = \varepsilon_0$, or on a supposé le contraire.
+C'est donc bien que $\gamma_i < \alpha$.
+\end{corrige}
+
+On appellera \emph{écriture 1} d'un ordinal $\alpha < \varepsilon_1$
+l'écriture qui est \underline{ou bien} $\varepsilon_0$ (considéré
+comme un symbole spécial), \underline{ou bien} une forme normale de
+Cantor $\omega^{\gamma_s} n_s + \cdots + \omega^{\gamma_1} n_1$ où les
+exposants $\gamma_s > \cdots > \gamma_1$ sont tous $<\alpha$ et
+eux-mêmes écrits en écriture 1 (ceci a bien un sens par (6)).
+
+À titre d'exemple, $\omega^\omega\,2$ ou $\varepsilon_0$ ou bien
+$\omega^{\varepsilon_0} + 1$ ou encore
+$\omega^{\omega^{\varepsilon_0}+1}\,2$ sont des écritures 1. En
+revanche, $\omega^{\varepsilon_0}$ n'en est pas une (elle ne vérifie
+pas la contrainte sur les exposants), ni $\varepsilon_0 + 1$ (ce n'est
+ni le symbole spécial $\varepsilon_0$ ni une forme normale de Cantor),
+ni $\varepsilon_0\,2$, ni $(\varepsilon_0)^2$.
+
+\textbf{(7)} Expliquer brièvement pourquoi tout ordinal $<\varepsilon_1$
+possède bien une écriture 1 unique. Il est alors facile de voir que
+cette écriture 1 permet algorithmiquement de manipuler les ordinaux
+$<\varepsilon_1$ : c'est-à-dire de les comparer, de les ajouter et de
+les multiplier (on ne demande pas de le justifier, les algorithmes
+étant essentiellement les mêmes que vus en cours pour les ordinaux
+$<\varepsilon_0$ sur la forme normale de Cantor : il faut simplement
+bien se rappeler dans les calculs intermédiaires le fait que
+$\varepsilon_0 = \omega^{\varepsilon_0}$ pour convertir
+$\varepsilon_0$ en forme normale de Cantor dès qu'on en a besoin).
+Calculer notamment $\varepsilon_0\cdot 2$ et $\varepsilon_0\cdot
+\omega$ en écriture 1. Expliquer ensuite comment calculer
+$(\varepsilon_0)^\alpha$ en écriture 1 lorsque $\alpha$ est lui-même
+donné en écriture 1. Notamment, écrire $(\varepsilon_0)^{\omega 2}$
+en écriture 1.
+
+\begin{corrige}
+L'existence et l'unicité de l'écriture 1 résulte du (6) : donné un
+ordinal $<\varepsilon_1$, soit il est égal à $\varepsilon_0$, auquel
+cas il a une écriture 1 par définition (et celle-ci est bien unique
+car on n'autorise pas de forme normale de Cantor pour lui), soit on
+l'écrit sous forme normale de Cantor avec des exposants strictement
+plus petits que lui, cette représentation est unique, et on peut
+recommencer le procédé, ce qui termine au bout d'un nombre fini
+d'étapes puisqu'on a affaire à des ordinaux qui décroissent
+strictement. (Ou, si on préfère, on montre par induction transfinie
+sur $\alpha < \varepsilon_1$ que $\alpha$ possède une écriture 1
+unique par le raisonnement qu'on vient de dire.)
+
+Calculons $\varepsilon_0\cdot 2$ en écriture 1 : il suffit de réécrire
+$\varepsilon_0$ comme $\omega^{\varepsilon_0}$, et alors
+$\omega^{\varepsilon_0}\cdot 2$ est une écriture 1 légitime (c'est
+bien une forme normale de Cantor dont les exposants sont tous écrits
+en écriture 1 et plus petit que l'ordinal donné). De même, calculons
+$\varepsilon_0\cdot \omega$ : pour cela, on écrit $\varepsilon_0\cdot
+\omega = \omega^{\varepsilon_0}\cdot \omega = \omega^{\varepsilon_0+1} =
+\omega^{\omega^{\varepsilon_0}+1}$, ce qui est une écriture 1
+légitime.
+
+Pour calculer $(\varepsilon_0)^\alpha$, on l'écrit comme
+$(\omega^{\varepsilon_0})^\alpha = \omega^{\varepsilon_0\,\alpha}$, or
+on vient de dire qu'on peut calculer algorithmiquement l'écriture 1 de
+$\varepsilon_0\,\alpha$ en fonction de celle de $\alpha$ : en la
+remplaçant dans l'exposant, on obtient alors une écriture 1 de
+$\omega^{\varepsilon_0\,\alpha}$ (sauf pour $\alpha=1$ auquel cas on
+laisse $\varepsilon_0$ comme résultat).
+
+Calculons $(\varepsilon_0)^{\omega 2}$ en écriture 1 : on vient de
+voir qu'il vaut $\omega^{\varepsilon_0\,\omega 2}$, or
+$\varepsilon_0\,\omega 2 = \omega^{\omega^{\varepsilon_0}+1}\,2$ comme
+ci-dessus, donc $(\varepsilon_0)^{\omega 2} =
+\omega^{\omega^{\omega^{\varepsilon_0}+1}\,2}$ (avec le parenthésage :
+$\omega^{((\omega^{((\omega^{\varepsilon_0})+1)})\cdot 2)}$).
+\end{corrige}
+
+\textbf{(8)} Indépendamment des questions précédentes, rappeler pourquoi tout
+ordinal $\alpha$ possède une écriture unique sous la forme
+$(\varepsilon_0)^{\gamma_s}\, \xi_s + \cdots +
+(\varepsilon_0)^{\gamma_1}\, \xi_1$ où $\gamma_s > \cdots > \gamma_1$
+sont des ordinaux et où $\xi_s,\ldots,\xi_1$ sont des ordinaux tous
+non nuls et strictement inférieurs à $\varepsilon_0$.
+
+\begin{corrige}
+Il s'agit de l'écriture en base $\tau$ des ordinaux, dans le cas
+particulier de $\tau = \varepsilon_0$.
+\end{corrige}
+
+\textbf{(9)} Indépendamment des questions précédentes, montrer que
+$\varepsilon_0 + \varepsilon_1 = \varepsilon_1$ (on rappelle que
+$\omega^\gamma + \omega^{\gamma'} = \omega^{\gamma'}$ lorsque $\gamma
+< \gamma'$). En déduire que $\varepsilon_0 \cdot \varepsilon_1 =
+\varepsilon_1$. En déduire que $(\varepsilon_0)^{\varepsilon_1} =
+\varepsilon_1$. Réciproquement, montrer que si $\delta$ est un
+ordinal tel que $(\varepsilon_0)^{\delta} = \delta$ alors il vérifie
+aussi $\omega^\delta = \delta$ (on pourra montrer $\delta \leq
+\omega^\delta \leq \omega^{\varepsilon_0 \delta} = \delta$) et en
+déduire que $\delta \geq \varepsilon_1$. En déduire que
+$\varepsilon_1$ est le plus petit ordinal tel que
+$(\varepsilon_0)^{\delta} = \delta$.
+
+\begin{corrige}
+On a $\varepsilon_0 + \varepsilon_1 = \omega^{\varepsilon_0} +
+\omega^{\varepsilon_1} = \omega^{\varepsilon_1}$ car $\varepsilon_0 <
+\varepsilon_1$. On en déduit $\varepsilon_0 \cdot \varepsilon_1 =
+\omega^{\varepsilon_0} \cdot \omega^{\varepsilon_1} =
+\omega^{\varepsilon_0 + \varepsilon_1} = \omega^{\varepsilon_1} =
+\varepsilon_1$. On en déduit $(\varepsilon_0)^{\varepsilon_1} =
+((\omega^{\varepsilon_0})^{\varepsilon_1} =
+\omega^{\varepsilon_0\cdot\varepsilon_1} = \omega^{\varepsilon_1} =
+\varepsilon_1$.
+
+Si $(\varepsilon_0)^{\delta} = \delta$ alors $\delta \leq
+\omega^\delta \leq \omega^{\varepsilon_0 \delta} =
+(\omega^{\varepsilon_0})^\delta = (\varepsilon_0)^{\delta} = \delta$
+où les deux inégalités résultent de (2) et de la croissance de
+$\gamma\mapsto\omega^\gamma$, donc toutes ces inégalités sont des
+égalités et notamment $\omega^\delta = \delta$. Comme $\varepsilon_1$
+est le deuxième point fixe de $\gamma \mapsto \omega^\gamma$, on en
+déduit que soit $\delta = \varepsilon_0$ soit $\delta \geq
+\varepsilon_1$ : la première possibilité est exclue car
+$\varepsilon_0^{\varepsilon_0} > \varepsilon_0$ (par stricte
+croissance de l'exponentiation ordinale en l'exposant lorsque la base
+est $\geq 2$), et on a donc $\delta \geq \varepsilon_1$. On a bien
+prouvé que $\varepsilon_1$ est le plus petit ordinal tel que
+$(\varepsilon_0)^{\delta} = \delta$.
+\end{corrige}
+
+On appellera \emph{écriture 2} d'un ordinal $\alpha < \varepsilon_1$
+une écriture $(\varepsilon_0)^{\gamma_s}\, \xi_s + \cdots +
+(\varepsilon_0)^{\gamma_1}\, \xi_1$ comme en (7), où les exposants
+$\gamma_s > \cdots > \gamma_1$ sont tous $<\alpha$ et eux-mêmes écrits
+en écriture 2, et où les $\xi_s,\ldots,\xi_1$ (qui
+sont $<\varepsilon_0$) sont écrits en forme normale de Cantor itérée.
+
+À titre d'exemple, $\omega^\omega\,2$ (il faut sous-entendre
+$(\varepsilon_0)^0$ devant, qui vaut $1$) ou $(\varepsilon_0)^2 + 1$
+ou bien $\varepsilon_0\,2 + \omega^\omega$ ou encore
+$(\varepsilon_0)^{\varepsilon_0}\,\omega^\omega\,3$ sont des
+écritures 2. En revanche, $\omega^{\varepsilon_0+1}$ n'en est pas une
+(les puissances de $\omega$ ne peuvent apparaître qu'au sein d'une
+forme normale de Cantor itérée, dont l'exposant ne fait donc jamais
+intervenir $\varepsilon_0$).
+
+\textbf{(10)} Expliquer brièvement pourquoi tout ordinal $<\varepsilon_1$
+possède bien une écriture 2 unique. Esquisser un algorithme
+permettant de convertire l'écriture 2 d'un ordinal $<\varepsilon_1$ en
+écriture 1 (on utilisera la question (7)).
+
+\begin{corrige}
+L'existence et l'unicité de l'écriture 2 résulte de (8) et de
+l'existence et unicité de la forme normale de Cantor itérée pour les
+ordinaux $<\varepsilon_0$.
+
+Pour convertir de l'écriture 2 en écriture 1, on part de l'écriture 2,
+on convertit chaque exposant de $(\varepsilon_0)^\gamma$ de
+l'écriture 1 en écriture 2 en utilisant l'algorithme récursivement
+(ceci termine bien car les exposants sont strictement plus simples, ou
+plus petits comme on voudra dire). On calcule alors la valeur de
+$(\varepsilon_0)^\gamma$ en partant de l'écriture 1 de $\gamma$ en
+utilisant la question (7). Il ne reste alors plus qu'à distribuer à
+droite les produits $(\varepsilon_0)^\gamma\cdot\xi$ avec $\xi$ écrit
+comme forme normale de Cantor itérée, et enfin calculer l'expression
+globale en écriture 1 (ce qui ne fait intervenir que des sommes et des
+produits, qu'on sait calculer).
+\end{corrige}
+
+{\footnotesize\textit{Remarque.} Il est aussi possible de convertir
+ algorithmiquement de l'écriture 1 vers l'écriture 2 : ceci passe par
+ calculer les $\omega^\alpha$ pour $\alpha$ donné en écriture 2.\par}
+
+\medskip
+
+\underline{Troisième partie.}
+
+\nobreak
+On aura besoin ici des définitions des ordinaux $\varepsilon_0$ et
+$\varepsilon_1$ données en tête de la première partie, mais pas plus.
+
+On s'intéresse à un jeu de Hercule et de l'hydre qui est analogue au
+jeu considéré en cours mais avec une extension : comme en cours,
+l'hydre est un arbre fini enraciné, mais l'hydre a maintenant deux
+types de têtes (= feuilles de l'arbre) : des têtes normales, et des
+\emph{œufs} (pouvant donner naissance à de nouvelles hydres). Quand
+Hercule coupe une tête $x$ normale, l'hydre se reproduit exactement
+comme on l'a vu en cours, c'est-à-dire qu'elle reproduit autant
+d'exemplaires qu'elle le veut, œufs compris, de tout le sous-arbre
+partant du nœud $y$ parent de $x$ dans l'arbre (ces copies étant
+ajoutées comme filles du nœud $z$ parent de $y$), à condition que $y$
+ne soit pas la racine (sinon, l'hydre ne joue pas). En revanche, si
+Hercule coupe un œuf, cet œuf éclot est remplacé par une nouvelle
+hydre, c'est-à-dire par un sous-arbre, arbitrairement complexe (choisi
+par le joueur qui contrôle l'hydre), mais ne comportant lui-même pas
+d'œuf, qui prend la place de la tête où était situé l'œuf.
+
+A titre d'exemple, sur le dessin suivant, où les œufs ont été
+représentés par des ovales gris, selon la tête coupée par Hercule :
+\begin{center}
+\begin{tikzpicture}[baseline=0]
+\draw[very thin] (-1.5,0) -- (1.5,0);
+\begin{scope}[every node/.style={circle,fill,inner sep=0.5mm}]
+\node (P0) at (0,0) {};
+\node (P1) at (0,1) {};
+\node (P2) at (0,2) {};
+\node (P3) at (-1,3) {};
+\node (P4) at (1,3) {};
+\end{scope}
+\begin{scope}[line width=1.5pt]
+\draw (P0) -- (P1);
+\draw (P1) -- (P2);
+\draw (P2) -- (P3);
+\draw (P2) -- (P4);
+\fill[fill=gray] (P3) to[out=0,in=270] ($(P3) + (0.3,0.3)$) to[out=90,in=0] ($(P3) + (0,1.0)$) to[out=180,in=90] ($(P3) + (-0.3,0.3)$) to[out=270,in=180] (P3);
+\end{scope}
+\begin{scope}[every node/.style={circle,fill,inner sep=0.5mm}]
+\node at (P3) {};
+\end{scope}
+\end{tikzpicture}
+peut devenir
+\begin{tikzpicture}[baseline=0]
+\draw[very thin] (-1.5,0) -- (1.5,0);
+\begin{scope}[every node/.style={circle,fill,inner sep=0.5mm}]
+\node (P0) at (0,0) {};
+\node (P1) at (0,1) {};
+\node (P2) at (0,2) {};
+\node (P3) at (-1,3) {};
+\node (P4) at (1,3) {};
+\node (P3a) at (-1.5,4) {};
+\node (P3b) at (-1,4) {};
+\node (P3c) at (-0.5,4) {};
+\node (P3aa) at (-1.75,4.5) {};
+\node (P3ab) at (-1.25,4.5) {};
+\end{scope}
+\begin{scope}[line width=1.5pt]
+\draw (P0) -- (P1);
+\draw (P1) -- (P2);
+\draw (P2) -- (P3);
+\draw (P2) -- (P4);
+\draw (P3) -- (P3a);
+\draw (P3) -- (P3b);
+\draw (P3) -- (P3c);
+\draw (P3a) -- (P3aa);
+\draw (P3a) -- (P3ab);
+\end{scope}
+\end{tikzpicture}
+ou
+\begin{tikzpicture}[baseline=0]
+\draw[very thin] (-1.5,0) -- (1.5,0);
+\begin{scope}[every node/.style={circle,fill,inner sep=0.5mm}]
+\node (P0) at (0,0) {};
+\node (P1) at (0,1) {};
+\node (P2) at (0,2) {};
+\node (P3a) at (-1,3) {};
+\node (P2b) at (0.5,2) {};
+\node (P3b) at (-0.25,3) {};
+\node (P2c) at (1,2) {};
+\node (P3c) at (0.5,3) {};
+\node (P2d) at (1.5,2) {};
+\node (P3d) at (1.25,3) {};
+\end{scope}
+\begin{scope}[line width=1.5pt]
+\draw (P0) -- (P1);
+\draw (P1) -- (P2);
+\draw (P1) -- (P2b);
+\draw (P1) -- (P2c);
+\draw (P1) -- (P2d);
+\draw (P2) -- (P3a);
+\draw (P2b) -- (P3b);
+\draw (P2c) -- (P3c);
+\draw (P2d) -- (P3d);
+\fill[fill=gray] (P3a) to[out=0,in=270] ($(P3a) + (0.3,0.3)$) to[out=90,in=0] ($(P3a) + (0,1.0)$) to[out=180,in=90] ($(P3a) + (-0.3,0.3)$) to[out=270,in=180] (P3a);
+\fill[fill=gray] (P3b) to[out=0,in=270] ($(P3b) + (0.3,0.3)$) to[out=90,in=0] ($(P3b) + (0,1.0)$) to[out=180,in=90] ($(P3b) + (-0.3,0.3)$) to[out=270,in=180] (P3b);
+\fill[fill=gray] (P3c) to[out=0,in=270] ($(P3c) + (0.3,0.3)$) to[out=90,in=0] ($(P3c) + (0,1.0)$) to[out=180,in=90] ($(P3c) + (-0.3,0.3)$) to[out=270,in=180] (P3c);
+\fill[fill=gray] (P3d) to[out=0,in=270] ($(P3d) + (0.3,0.3)$) to[out=90,in=0] ($(P3d) + (0,1.0)$) to[out=180,in=90] ($(P3d) + (-0.3,0.3)$) to[out=270,in=180] (P3d);
+\end{scope}
+\begin{scope}[every node/.style={circle,fill,inner sep=0.5mm}]
+\node at (P3a) {};
+\node at (P3b) {};
+\node at (P3c) {};
+\node at (P3d) {};
+\end{scope}
+\end{tikzpicture}
+\end{center}
+
+\textbf{(11)} En associant à toute position du jeu (= tout arbre enraciné dont
+certaines feuilles sont qualifiées d'œufs) un
+ordinal $<\varepsilon_1$, montrer que Hercule gagne toujours,
+c'est-à-dire qu'il va toujours réduire l'hydre à sa seule racine en
+temps fini (quoi qu'il fasse et quoi que fasse l'hydre).
+
+\begin{corrige}
+À toute hydre $T$ on associe un ordinal $o(T) <\varepsilon_1$ par
+récurrence sur la profondeur de l'arbre, de la façon suivante : si $T$
+est un œuf, alors on pose $o(T) = \varepsilon_0$ ; sinon, si
+$T_1,\ldots,T_r$ sont les sous-arbres ayant pour racine les fils de la
+racine, triés de façon que $o(T_1) \geq \cdots \geq o(T_r)$, alors on
+pose $o(T) = \omega^{o(T_1)} + \cdots + \omega^{o(T_r)}$. Exactement
+les mêmes démonstrations que dans le cours tiennent, il faut
+simplement ajouter la clause suivante : si $T$ est un œuf et $T'$ est
+une hydre sans œuf, alors $o(T') < \varepsilon_0 = o(T)$, donc en
+remplaçant l'œuf par une hydre sans œuf quelconque, on fait
+strictement décroître l'ordinal.
+
+(Remarquons que, comme $\varepsilon_0 = \omega^{\varepsilon_0}$, une
+tige de longueur arbitraire se finissant par un œuf a toujours la même
+valeur $\varepsilon_0$ avec ce système : ce n'est pas un problème, et
+ce n'est pas surprenant puisque de telles tiges offrent
+essentiellement les mêmes possibilités à l'hydre.)
+\end{corrige}
+
+\textbf{(12)} Donner un exemple de position du jeu associé à l'ordinal
+$(\varepsilon_0)^{\varepsilon_0}$ par le système proposé en (11).
+
+\begin{corrige}
+L'hydre suivante (dans laquelle les œufs ont été représentés par des
+ovales gris) :
+\begin{center}
+\begin{tikzpicture}
+\draw[very thin] (-1.5,0) -- (1.5,0);
+\begin{scope}[every node/.style={circle,fill,inner sep=0.5mm}]
+\node (P0) at (0,0) {};
+\node (P1) at (0,1) {};
+\node (P2) at (0,2) {};
+\node (P3) at (-1,3) {};
+\node (P4) at (1,3) {};
+\end{scope}
+\begin{scope}[line width=1.5pt]
+\draw (P0) -- (P1);
+\draw (P1) -- (P2);
+\draw (P2) -- (P3);
+\draw (P2) -- (P4);
+\fill[fill=gray] (P3) to[out=0,in=270] ($(P3) + (0.3,0.3)$) to[out=90,in=0] ($(P3) + (0,1.0)$) to[out=180,in=90] ($(P3) + (-0.3,0.3)$) to[out=270,in=180] (P3);
+\fill[fill=gray] (P4) to[out=0,in=270] ($(P4) + (0.3,0.3)$) to[out=90,in=0] ($(P4) + (0,1.0)$) to[out=180,in=90] ($(P4) + (-0.3,0.3)$) to[out=270,in=180] (P4);
+\end{scope}
+\begin{scope}[every node/.style={circle,fill,inner sep=0.5mm}]
+\node at (P3) {};
+\node at (P4) {};
+\end{scope}
+\end{tikzpicture}
+\end{center}
+a la valeur $\omega^{\omega^{\varepsilon_0\,2}} =
+\omega^{(\omega^{\varepsilon_0})^2} = \omega^{\varepsilon_0^2} =
+(\omega^{\varepsilon_0})^{\varepsilon_0} =
+(\varepsilon_0)^{\varepsilon_0}$, comme demandé.
+\end{corrige}
+
+{\footnotesize\textit{Remarque.} Pour rendre ce jeu plus intéressant,
+ il faudrait sans doute ajouter une règle selon laquelle Hercule ne
+ peut couper un œuf que s'il ne reste aucune tête non-œuf à couper,
+ sinon il est assez clair qu'il a intérêt à commencer par éliminer
+ tous les œufs. Mais cette contrainte, puisqu'elle ne concerne
+ qu'Hercule n'a aucune influence sur ce qu'on vient de prouver.\par}
+
+
+%
+%
+%
+\end{document}
diff --git a/controle-20220413.tex b/controle-20220413.tex
new file mode 100644
index 0000000..5c2448d
--- /dev/null
+++ b/controle-20220413.tex
@@ -0,0 +1,441 @@
+%% This is a LaTeX document. Hey, Emacs, -*- latex -*- , get it?
+\documentclass[12pt,a4paper]{article}
+\usepackage[francais]{babel}
+\usepackage[utf8]{inputenc}
+\usepackage[T1]{fontenc}
+%\usepackage{ucs}
+\usepackage{times}
+% A tribute to the worthy AMS:
+\usepackage{amsmath}
+\usepackage{amsfonts}
+\usepackage{amssymb}
+\usepackage{amsthm}
+%
+\usepackage{mathrsfs}
+\usepackage{wasysym}
+\usepackage{url}
+%
+\usepackage{graphics}
+\usepackage[usenames,dvipsnames]{xcolor}
+\usepackage{tikz}
+\usetikzlibrary{matrix,calc}
+\usepackage{hyperref}
+%
+%\externaldocument{notes-mitro206}[notes-mitro206.pdf]
+%
+\theoremstyle{definition}
+\newtheorem{comcnt}{Tout}
+\newcommand\thingy{%
+\refstepcounter{comcnt}\smallskip\noindent\textbf{\thecomcnt.} }
+\newcommand\exercice{%
+\refstepcounter{comcnt}\bigskip\noindent\textbf{Exercice~\thecomcnt.}\par\nobreak}
+\renewcommand{\qedsymbol}{\smiley}
+%
+\newcommand{\outnb}{\operatorname{outnb}}
+\newcommand{\downstr}{\operatorname{downstr}}
+\newcommand{\precs}{\operatorname{precs}}
+\newcommand{\mex}{\operatorname{mex}}
+\newcommand{\id}{\operatorname{id}}
+\newcommand{\limp}{\Longrightarrow}
+\newcommand{\gr}{\operatorname{gr}}
+\newcommand{\rk}{\operatorname{rk}}
+\newcommand{\fuzzy}{\mathrel{\|}}
+%
+\DeclareUnicodeCharacter{00A0}{~}
+%
+\DeclareMathSymbol{\tiret}{\mathord}{operators}{"7C}
+\DeclareMathSymbol{\traitdunion}{\mathord}{operators}{"2D}
+%
+\DeclareFontFamily{U}{manual}{}
+\DeclareFontShape{U}{manual}{m}{n}{ <-> manfnt }{}
+\newcommand{\manfntsymbol}[1]{%
+ {\fontencoding{U}\fontfamily{manual}\selectfont\symbol{#1}}}
+\newcommand{\dbend}{\manfntsymbol{127}}% Z-shaped
+\newcommand{\danger}{\noindent\hangindent\parindent\hangafter=-2%
+ \hbox to0pt{\hskip-\hangindent\dbend\hfill}}
+%
+\newcommand{\spaceout}{\hskip1emplus2emminus.5em}
+\newif\ifcorrige
+\corrigetrue
+\newenvironment{corrige}%
+{\ifcorrige\relax\else\setbox0=\vbox\bgroup\fi%
+\smallbreak\noindent{\underbar{\textit{Corrigé.}}\quad}}
+{{\hbox{}\nobreak\hfill\checkmark}%
+\ifcorrige\par\smallbreak\else\egroup\par\fi}
+%
+%
+%
+\begin{document}
+\ifcorrige
+\title{MITRO206\\Contrôle de connaissances — Corrigé\\{\normalsize Théories des jeux}}
+\else
+\title{MITRO206\\Contrôle de connaissances\\{\normalsize Théories des jeux}}
+\fi
+\author{}
+\date{13 avril 2022}
+\maketitle
+
+\pretolerance=8000
+\tolerance=50000
+
+\vskip1truein\relax
+
+\noindent\textbf{Consignes.}
+
+Les exercices sont totalement indépendants. Ils pourront être traités
+dans un ordre quelconque, mais on demande de faire apparaître de façon
+très visible dans les copies où commence chaque exercice.
+
+La longueur du sujet ne doit pas effrayer : l'énoncé est long parce
+que des rappels ont été faits et que la rédaction des questions
+cherche à éviter toute ambiguïté. Les réponses attendues sont
+généralement beaucoup plus courtes que les questions elles-mêmes
+(notamment dans le dernier exercice).
+
+\medbreak
+
+L'usage de tous les documents (notes de cours manuscrites ou
+imprimées, feuilles d'exercices, livres) est autorisé.
+
+L'usage des appareils électroniques est interdit.
+
+\medbreak
+
+Durée : 2h
+
+Barème \emph{indicatif} : $8+6+6$.
+
+\ifcorrige
+Ce corrigé comporte 10 pages (page de garde incluse).
+\else
+Cet énoncé comporte 6 pages (page de garde incluse).
+\fi
+
+\vfill
+{\noindent\tiny
+\immediate\write18{sh ./vc > vcline.tex}
+Git: \input{vcline.tex}
+\immediate\write18{echo ' (stale)' >> vcline.tex}
+\par}
+
+\pagebreak
+
+
+%
+%
+%
+
+\exercice
+
+Le but de cet exercice est de tenter une classification des jeux en
+forme normale, à deux joueurs, \emph{symétriques} (c'est-à-dire que
+les deux joueurs ont les mêmes options et les mêmes gains sous l'effet
+de la permutation qui les échange) et avec deux options.
+
+On considère donc le jeu dont la matrice de gains est la suivante, où
+$u,v,x,y$ sont des réels sur lesquels on va discuter (les options sont
+étiquetées $C$ et $D$ ; le gain d'Alice est listé en premier, celui de
+Bob en second) :
+
+\begin{center}
+\begin{tabular}{r|c|c|}
+$\downarrow$Alice, Bob$\rightarrow$&$C$&$D$\\\hline
+$C$&$u$, $u$&$v$, $x$\\\hline
+$D$&$x$, $v$&$y$, $y$\\\hline
+\end{tabular}
+\end{center}
+
+On se limitera à l'étude du cas où $u>v$, ce qu'on supposera
+désormais.
+
+(1) Expliquer brièvement pourquoi il ne change rien à l'analyse du jeu
+(p.ex., au calcul des équilibres de Nash) de remplacer tous les gains
+$t$ d'un joueur donné par $at+b$ où $a>0$ et $b$ est quelconque. En
+déduire qu'on peut supposer, dans le jeu ci-dessus, que $u=1$ et
+$v=0$, ce qu'on fera désormais :
+
+\begin{center}
+\begin{tabular}{r|c|c|}
+$\downarrow$Alice, Bob$\rightarrow$&$C$&$D$\\\hline
+$C$&$1$, $1$&$0$, $x$\\\hline
+$D$&$x$, $0$&$y$, $y$\\\hline
+\end{tabular}
+\end{center}
+
+(2) À quelle condition le profil $(C,C)$ (c'est-à-dire : Alice
+joue $C$ et Bob joue $C$) est-il un équilibre de Nash ? À quelle
+condition $(D,D)$ est-il un équilibre de Nash ? À quelle condition
+$(C,D)$ est-il un équilibre de Nash ? Qu'en est-il de $(D,C)$ ? (À
+chaque fois, on demande des conditions sous forme d'inégalités portant
+sur $x$ et $y$.)
+
+On suppose dans la suite (pour écarter des cas limites pénibles) que
+$x$ n'est pas exactement égal à $1$ et que $y$ n'est pas exactement
+égal à $0$.
+
+(3) Expliquer pourquoi il n'y a pas d'équilibre de Nash dans lequel un
+joueur joue une stratégie pure (i.e., soit $C$ soit $D$) et l'autre
+une stratégie strictement mixte (i.e., $pC + (1-p)D$ avec $0<p<1$).
+
+(4) Analyser la possibilité que $(pC + (1-p)D, \; qC + (1-q)D)$, où
+$0<p<1$ et $0<q<1$ soit un équilibre de Nash : que doivent valoir
+$p$ et $q$ si c'est le cas ? et à quelle condition nécessaire et
+suffisante obtient-on effectivement un équilibre de Nash de cette
+forme ? (On pourra tracer un graphique, par ailleurs demandé à la
+question suivante, pour visualiser le signe de $1-x+y$.)
+
+(5) Dans le plan de coordonnées $(x,y)$, représenter graphiquement les
+domaines des paramètres dans lesquels existent les différents
+équilibres de Nash trouvés dans l'analyse. (On rappelle qu'il doit
+toujours y en avoir au moins un, ce qui peut permettre de détecter
+d'éventuelles erreurs.) Dans quelle partie du diagramme se situent le
+dilemme du prisonnier et le dilemme du trouillard respectivement ?
+
+(La question (6) peut être traitée indépendamment des questions
+précédentes.)
+
+(6) On introduit le nouveau concept suivant : pour un jeu symétrique,
+une \textbf{stratégie rationnelle commun} est une stratégie mixte,
+donc ici, $rC + (1-r)D$ pour $0\leq 1\leq r$, qui quand elle est jouée
+par l'ensemble des joueurs, conduit au gain (forcément le même pour
+tous) le plus élevé sous cette contrainte\footnote{Autrement dit, une
+ stratégie rationnelle commune est une stratégie mixte $s$ telle que
+ si tous les joueurs jouent $s$, leur espérance de gain commune sera
+ supérieure ou égale à celle s'ils jouent tous $s'$, quelle que
+ soit $s'$.}. Dans le jeu considéré ici, calculer l'espérance de
+gain commune des joueurs s'ils jouent tous $rC + (1-r)D$ , et en
+déduire pour quelle(s) valeur(s) de $r$ cette fonction est maximale,
+en discutant éventuellement selon les valeurs de $x,y$. Tracer un
+nouveau graphique pour représenter, dans le plan de coordonnés
+$(x,y)$, les domaines de paramètres dans lequels existent les
+différentes stratégies rationnelles communes (on distinguera : $C$,
+$D$ et $rC + (1-r)D$ avec $0<r<1$).
+
+
+%
+%
+%
+
+\exercice
+
+On s'intéresse ici à la variation suivante du jeu de nim (fini) :
+après avoir retiré des bâtonnets d'une ligne, un joueur peut en outre,
+s'il le souhaite, \emph{couper} la ligne en deux, ce qui crée deux
+lignes au lieu d'une, en répartissant comme il le veut les bâtonnets
+de la ligne initiale (après en avoir retiré au moins un) entre ces
+deux lignes.
+
+De façon plus formelle, l'état du jeu est donné par la liste des
+nombres $n_1,\ldots,n_r \in \mathbb{N}$ de bâtonnets des différentes
+lignes du jeu (on peut ignorer ceux pour lesquels $n_i=0$) ; et un
+coup du jeu consiste à choisir un $1\leq i\leq r$ et à remplacer $n_i$
+dans la liste soit par un entier neturels $n' < n_i$, soit par
+\emph{deux} entiers naturels $n',n''$ tels que $n' + n'' < n_i$ (on
+peut d'ailleurs ne considérer que ce deuxième type de coup puisque
+prendre $n''=0$ revient à n'avoir que $n'$). Comme d'habitude, le
+joueur qui ne peut pas jouer a perdu (i.e., le gagnant est celui qui
+retire le dernier bâtonnet) ; et la disposition des lignes ou des
+bâtonnets au sein d'une ligne n'a pas d'importance, seul compte leur
+nombre (et tout est fini).
+
+(1) Expliquer pourquoi la valeur de Grundy de la position
+$(n_1,\ldots,n_r)$ du jeu est la somme de nim $f(n_1) \oplus \cdots
+\oplus f(n_r)$ des valeurs de Grundy $f(n_i)$ des positions ayant une
+seule ligne avec $n_i$ bâtonnets (où $f$ est une fonction qui reste à
+calculer, avec évidemment $f(0)=0$).
+
+(2) Expliquer pourquoi $f(n) = \mex\{f(n') \oplus f(n'') \; | \;
+n'+n'' < n\}$ est le plus petit entier naturel qui n'est pas de la
+forme $f(n') \oplus f(n'')$ où $n',n''$ parcourent les couples
+d'entiers naturels tels que $n'+n'' < n$ (et comme d'habitude, $\mex
+S$ est le plus petit entier naturel qui n'est pas dans $S$).
+
+(3) Indépendamment de ce qui précède, expliquer pourquoi $k \oplus
+\ell \leq k + \ell$ quels que soient $k,\ell \in \mathbb{N}$.
+
+(4) Montrer que $f(n) = n$ pour tout $n$.
+
+(5) Que conclure quant à la stratégie gagnante à la variante proposée
+ici du jeu de nim par rapport au jeu de nim standard ?
+
+
+%
+%
+%
+
+\exercice
+
+On s'intéresse dans cet exercice au jeu de \emph{Hackenbush bicolore
+ en arbre}, défini comme suit. L'état du jeu est représenté par un
+arbre (fini, enraciné\footnote{C'est-à-dire que la racine fait partie
+ de la donnée de l'arbre, ce qui est la convention la plus
+ courante.}) dont chaque arête est soit coloriée \emph{bleue} soit
+\emph{rouge}, mais jamais les deux à la fois (il y a exactement deux
+types d'arêtes). Deux joueurs, Blaise et Roxane, alternent et chacun
+à son tour choisit une arête de l'arbre, bleue pour Blaise ou rouge
+pour Roxane, et l'efface, ce qui fait automatiquement disparaître du
+même coup tout le sous-arbre qui descendait de cette arête (voir
+figure). L'arête choisie doit avoir la couleur associée au joueur,
+c'est-à-dire bleue pour Blaise ou rouge pour Roxane, mais toutes les
+arêtes qui en descendent sont effacées quelle que soit leur couleur.
+Le jeu se termine lorsque plus aucun coup n'est possible (c'est-à-dire
+que le joueur qui doit jouer n'a plus d'arête de sa couleur), auquel
+cas, selon la convention habituelle, le joueur qui ne peut plus jouer
+a perdu.
+
+À titre d'exemple, ceci illustre un coup possible de Roxane
+(effacement d'une arête rouge et de tout le sous-arbre qui en
+descend) :
+\begin{center}
+\begin{tikzpicture}[baseline=0]
+\fill [gray!50!white] (-3.0,0) rectangle (2.0,-0.2);
+\begin{scope}[every node/.style={circle,fill,inner sep=0.5mm}]
+\node (P0) at (0,0) {};
+\node (P1) at (-0.75,1) {};
+\node (P2) at (-2.25,2) {};
+\node (P3) at (-2.75,3) {};
+\node (P4) at (-1.75,3) {};
+\node (P5) at (0.75,2) {};
+\node (P6) at (0.00,3) {};
+\node (P7) at (0.75,3) {};
+\node (P8) at (1.50,3) {};
+\node (P9) at (1.75,1) {};
+\node (P10) at (1.75,2) {};
+\end{scope}
+\begin{scope}[line width=1.5pt]
+\draw[blue] (P0) -- (P1);
+\draw[red] (P1) -- (P2);
+\draw[red] (P1) -- (P5);
+\draw[blue] (P2) -- (P3);
+\draw[blue] (P2) -- (P4);
+\draw[blue] (P5) -- (P6);
+\draw[blue] (P5) -- (P7);
+\draw[red] (P5) -- (P8);
+\draw[red] (P0) -- (P9);
+\draw[blue] (P9) -- (P10);
+\end{scope}
+\begin{scope}[line width=3pt,gray!50!black]
+\draw ($0.5*(P1) + 0.5*(P5) + (-0.2,-0.2)$) -- ($0.5*(P1) + 0.5*(P5) + (0.2,0.2)$);
+\draw ($0.5*(P1) + 0.5*(P5) + (-0.2,0.2)$) -- ($0.5*(P1) + 0.5*(P5) + (0.2,-0.2)$);
+\end{scope}
+\end{tikzpicture}
+~devient~
+\begin{tikzpicture}[baseline=0]
+\fill [gray!50!white] (-3.0,0) rectangle (2.0,-0.2);
+\begin{scope}[every node/.style={circle,fill,inner sep=0.5mm}]
+\node (P0) at (0,0) {};
+\node (P1) at (-0.75,1) {};
+\node (P2) at (-2.25,2) {};
+\node (P3) at (-2.75,3) {};
+\node (P4) at (-1.75,3) {};
+\node (P9) at (1.75,1) {};
+\node (P10) at (1.75,2) {};
+\end{scope}
+\begin{scope}[line width=1.5pt]
+\draw[blue] (P0) -- (P1);
+\draw[red] (P1) -- (P2);
+\draw[blue] (P2) -- (P3);
+\draw[blue] (P2) -- (P4);
+\draw[red] (P0) -- (P9);
+\draw[blue] (P9) -- (P10);
+\end{scope}
+\end{tikzpicture}
+\end{center}
+
+(1) Expliquer brièvement pourquoi une position de ce jeu peut être
+considérée comme une somme disjonctive de différents jeux du même
+type. Plus exactement, soit $T$ un arbre bicolore de racine $x$,
+soient $y_1,\ldots,y_r$ les fils de $x$, soient $T_1,\ldots,T_r$ les
+sous-arbres ayant pour racines $y_1,\ldots,y_r$ et soient
+$T'_1,\ldots,T'_r$ les arbres de racine $x$ où $T'_i$ est formé de $x$
+et de $T_i$ (y comprise l'arête entre $x$ et $y_i$) : expliquer
+brièvement pourquoi la position représentée par l'arbre $T$ est la
+somme disjonctive de celles représentées par $T'_1,\ldots,T'_r$.
+
+\smallskip
+
+Si $T$ est un arbre de Hackenbush bicolore, notons $1{:}T$ l'arbre
+$T'$ correspondant à placer $T$ au sommet d'une unique arête bleue
+reliée à la racine (autrement dit, $T'$ est formé en ajoutant un
+nouveau sommet, qui sera la racine, et en ajoutant une arête bleue
+entre cette nouvelle racine et l'ancienne racine de $T$).
+
+On \underline{admet} les résultats suivants : à tout arbre de
+Hackenbush bicolore $T$ on peut associer un nombre réel $v(T)$ (sa
+\emph{valeur}), de façon que :
+\begin{itemize}
+\item[(a)] Si $T$ et $T'_1, \ldots, T'_r$ sont comme dans l'énoncé de
+ la question (1) alors $v(T) = v(T'_1) + \cdots + v(T'_r)$.
+\item[(b)] On a $v(-T) = -v(T)$ où $-T$ est l'arbre obtenu en échangeant
+ les couleurs rouge et bleue dans $T$.
+\item[(c)] On a $v(1{:}T) = \varphi_+(v(T))$ où $\varphi_+$ est la
+ fonction définie par\footnote{Les cas dans la définition de
+ $\varphi_+$ se chevauchent un peu, et c'est normal : les
+ définitions sont compatibles aux chevauchements.}
+\[
+\varphi_+(v) = \left\{
+\begin{array}{ll}
+v+1&\hbox{~si $v\geq 0$}\\
+2^{-n}\,(v+n+1)&\hbox{~si $-n\leq v \leq -n+1$ où $n\in\mathbb{N}$}\\
+\end{array}
+\right.
+\]
+\item[(d)] On a $v(T) \geq 0$ si et seulement si Blaise possède une
+ stratégie gagnante en jouant en second à partir de la position $T$,
+ et $v(T) \leq 0$ lorsque Roxane en possède une.
+\end{itemize}
+
+(2) Utiliser ces règles admises pour calculer la valeur de l'arbre
+tracé à gauche dans la figure ci-dessus (avant effacement). Pour
+éviter de se tromper, on recommande de reproduire l'arbre et
+d'indiquer à côté de chaque sommet la valeur du sous-arbre qui en
+descend, et à côté de chaque arête la valeur du sous-arbre avec
+l'arête en question. En déduire qui a une stratégie gagnante dans
+cette position selon le joueur qui commence.
+
+\smallbreak
+
+\centerline{* * *}
+
+Indépendamment de ce qui précède, on va considérer une opération sur
+les jeux partisans : si $G$ est un jeu combinatoire partisan, vu comme
+un graphe orienté (bien-fondé), on définit un jeu noté $1{:}G$ en
+ajoutant une unique position $0$ à $G$ comme on va l'expliquer. Pour
+chaque position $z$ de $G$ il y a une position notée $1{:}z$ de
+$1{:}G$, et il y a une unique autre position, notée $0$,
+dans $1{:}G$ ; pour chaque arête $z \to z'$ de $G$, il y a une arête
+$1{:}z\, \to \, 1{:}z'$ dans $1{:}G$, coloriée de la même manière que
+dans $G$, et il y a de plus une arête $1{:}z\, \to 0$ dans $1{:}G$
+pour chaque $z$, coloriée en bleu (en revanche, $0$ est un puits,
+c'est-à-dire qu'aucune arête n'en part) ; la position initiale de
+$1{:}G$ est $1{:}z_0$ où $z_0$ est celle de $G$. De façon plus
+informelle, pour jouer au jeu $1{:}G$, chaque joueur peut faire un
+coup normal ($1{:}z\, \to \, 1{:}z'$) de $G$, mais par ailleurs,
+Blaise peut à tout moment appliquer un coup « destruction totale »
+$1{:}z\, \to 0$ qui fait terminer immédiatement le jeu (et il a alors
+gagné\footnote{Ce jeu considéré tout seul n'est donc pas très amusant
+ puisque Blaise a toujours la possibilité de gagner
+ instantanément.}).
+
+(3) Montrer que si $G \geq H$ on a $1{:}G \geq 1{:}H$. (On rappelle
+que $G \geq H$ signifie : « Blaise a une stratégie gagnante s'il joue
+en second au jeu $G - H$ défini comme la somme disjonctive du jeu $G$
+et du jeu $-H$ obtenu en échangeant les deux joueurs au jeu $H$ ».
+Pour cela, on expliquera comment Blaise peut gagner à $(1{:}G) -
+(1{:}H)$ en jouant en second, en supposant qu'il sait gagner à $G - H$
+en jouant en second.) En déduire que si $G \doteq H$ alors $1{:}G
+\doteq 1{:}H$.
+
+{\footnotesize\textit{Remarque.} Ceci justifie partiellement
+ l'affirmation (c) des règles admises ci-dessus en ce sens que cela
+ explique que $v(1{:}G)$ ne dépende que de $v(G)$ et pas du détail
+ de $G$, et aussi que la fonction $\varphi_+$ est croissante.\par}
+
+
+
+
+
+%
+%
+%
+\end{document}
diff --git a/controle-20230417.tex b/controle-20230417.tex
new file mode 100644
index 0000000..fce95a1
--- /dev/null
+++ b/controle-20230417.tex
@@ -0,0 +1,793 @@
+%% This is a LaTeX document. Hey, Emacs, -*- latex -*- , get it?
+\documentclass[12pt,a4paper]{article}
+\usepackage[a4paper,margin=2.5cm]{geometry}
+\usepackage[francais]{babel}
+\usepackage[utf8]{inputenc}
+\usepackage[T1]{fontenc}
+%\usepackage{ucs}
+\usepackage{times}
+% A tribute to the worthy AMS:
+\usepackage{amsmath}
+\usepackage{amsfonts}
+\usepackage{amssymb}
+\usepackage{amsthm}
+%
+\usepackage{mathrsfs}
+\usepackage{wasysym}
+\usepackage{url}
+%
+\usepackage{graphics}
+\usepackage[usenames,dvipsnames]{xcolor}
+\usepackage{tikz}
+\usetikzlibrary{matrix,calc}
+\usepackage{hyperref}
+%
+%\externaldocument{notes-mitro206}[notes-mitro206.pdf]
+%
+\theoremstyle{definition}
+\newtheorem{comcnt}{Tout}
+\newcommand\thingy{%
+\refstepcounter{comcnt}\smallskip\noindent\textbf{\thecomcnt.} }
+\newcommand\exercice{%
+\refstepcounter{comcnt}\bigskip\noindent\textbf{Exercice~\thecomcnt.}\par\nobreak}
+\renewcommand{\qedsymbol}{\smiley}
+%
+\newcommand{\outnb}{\operatorname{outnb}}
+\newcommand{\downstr}{\operatorname{downstr}}
+\newcommand{\precs}{\operatorname{precs}}
+\newcommand{\mex}{\operatorname{mex}}
+\newcommand{\id}{\operatorname{id}}
+\newcommand{\limp}{\Longrightarrow}
+\newcommand{\gr}{\operatorname{gr}}
+\newcommand{\rk}{\operatorname{rk}}
+\newcommand{\dur}{\operatorname{dur}}
+\newcommand{\fuzzy}{\mathrel{\|}}
+%
+\newcommand{\dblunderline}[1]{\underline{\underline{#1}}}
+%
+\DeclareUnicodeCharacter{00A0}{~}
+%
+\DeclareMathSymbol{\tiret}{\mathord}{operators}{"7C}
+\DeclareMathSymbol{\traitdunion}{\mathord}{operators}{"2D}
+%
+\DeclareFontFamily{U}{manual}{}
+\DeclareFontShape{U}{manual}{m}{n}{ <-> manfnt }{}
+\newcommand{\manfntsymbol}[1]{%
+ {\fontencoding{U}\fontfamily{manual}\selectfont\symbol{#1}}}
+\newcommand{\dbend}{\manfntsymbol{127}}% Z-shaped
+\newcommand{\danger}{\noindent\hangindent\parindent\hangafter=-2%
+ \hbox to0pt{\hskip-\hangindent\dbend\hfill}}
+%
+\newcommand{\spaceout}{\hskip1emplus2emminus.5em}
+\newif\ifcorrige
+\corrigetrue
+\newenvironment{corrige}%
+{\ifcorrige\relax\else\setbox0=\vbox\bgroup\fi%
+\smallbreak\noindent{\underbar{\textit{Corrigé.}}\quad}}
+{{\hbox{}\nobreak\hfill\checkmark}%
+\ifcorrige\par\smallbreak\else\egroup\par\fi}
+%
+%
+%
+\begin{document}
+\ifcorrige
+\title{MITRO206\\Contrôle de connaissances — Corrigé\\{\normalsize Théories des jeux}}
+\else
+\title{MITRO206\\Contrôle de connaissances\\{\normalsize Théories des jeux}}
+\fi
+\author{}
+\date{17 avril 2023}
+\maketitle
+
+\pretolerance=8000
+\tolerance=50000
+
+\vskip1truein\relax
+
+\noindent\textbf{Consignes.}
+
+Les exercices sont totalement indépendants. Ils pourront être traités
+dans un ordre quelconque, mais on demande de faire apparaître de façon
+très visible dans les copies où commence chaque exercice.
+
+La longueur du sujet ne doit pas effrayer : l'énoncé est long parce
+que des rappels ont été faits et que la rédaction des questions
+cherche à éviter toute ambiguïté. De plus, il ne sera pas nécessaire
+de traiter la totalité pour avoir la note maximale.
+
+\medbreak
+
+L'usage de tous les documents (notes de cours manuscrites ou
+imprimées, feuilles d'exercices, livres) est autorisé.
+
+L'usage des appareils électroniques est interdit.
+
+\medbreak
+
+Durée : 2h
+
+Barème \emph{indicatif} : $5+5+5+7$ (sur $20$)
+
+\ifcorrige
+Ce corrigé comporte 8 pages (page de garde incluse).
+\else
+Cet énoncé comporte 4 pages (page de garde incluse).
+\fi
+
+\vfill
+{\noindent\tiny
+\immediate\write18{sh ./vc > vcline.tex}
+Git: \input{vcline.tex}
+\immediate\write18{echo ' (stale)' >> vcline.tex}
+\par}
+
+\pagebreak
+
+
+%
+%
+%
+
+\exercice
+
+On considère le jeu de nim éventuellement transfini. (On rappelle
+qu'il est défini de la manière suivante : une position du jeu est un
+tuple $(\alpha_1,\ldots,\alpha_r)$ d'ordinaux, on dit qu'il y a
+« $\alpha_i$ bâtonnets sur la ligne $i$ », et un coup consiste à
+décroître strictement \emph{un et un seul} des $\alpha_i$, autrement
+dit il existe un coup de $(\alpha_1,\ldots,\alpha_r)$ vers
+$(\alpha_1,\ldots,\beta,\ldots,\alpha_r)$ où $\beta<\alpha_i$ est mis
+à la place de $\alpha_i$.)
+
+Pour chacune des positions suivantes, dire si c'est une position P
+(c'est-à-dire gagnante pour le joueur qui vient de jouer) ou N
+(c'est-à-dire gagnante pour le joueur qui doit jouer), et, dans ce
+dernier cas, donner un coup gagnant possible pour le joueur en
+question.
+
+(a) $(1,2,3)$ (autrement dit, une ligne avec $1$ bâtonnet, une ligne
+avec $2$, et une ligne avec $3$)
+
+\begin{corrige}
+On a $1 = 2^0$ et $2 = 2^1$ et $3 = 2^1 + 2^0 = 2^1 \oplus 2^0$ si
+bien que $1 \oplus 2 \oplus 3 = 0$ : la valeur de Grundy de la
+position est $0$, et c'est donc une position P.
+\end{corrige}
+
+(b) $(3,6,9)$
+
+\begin{corrige}
+On a $3 = 2^1 + 2^0 = 2^1 \oplus 2^0$ et $6 = 2^2 + 2^1 = 2^2 \oplus
+2^1$ et $9 = 2^3 + 2^0 = 2^3 \oplus 2^0$ si bien que $3 \oplus 6
+\oplus 9 = 2^3 \oplus 2^2 = 2^3 + 2^2 = 12$ : la valeur de Grundy de
+la position est $12$, et c'est donc une position N.
+
+Pour trouver un coup gagnant, c'est-à-dire un coup vers une
+position P, on cherche à annuler la valeur de Grundy : autrement dit,
+on cherche à remplacer le nombre $n$ de bâtonnets d'une ligne par $n
+\oplus 12$, et il s'agit donc de trouver une ligne telle que $n \oplus
+12 < n$. On vérifie facilement que la seule possibilité est de
+réduire la ligne ayant $9$ bâtonnets à $9 \oplus 12 = 2^2 + 2^0 = 5$
+bâtonnets. Bref, le seul coup gagnant est $(3,6,9) \to (3,6,5)$.
+\end{corrige}
+
+(c) $(\omega,\omega2,\omega3)$
+
+\begin{corrige}
+En se rappelant que $\omega = 2^{\omega}$, on a $\omega2 =
+2^{\omega+1}$ et $\omega3 = 2^{\omega+1} + 2^{\omega}$ en binaire : on
+a donc $\omega \oplus (\omega2) \oplus (\omega3) = 0$ : la valeur de
+Grundy de la position est $0$, et c'est donc une position P.
+\end{corrige}
+
+(d) $(\omega,\omega^2,\omega^3)$
+
+\begin{corrige}
+En se rappelant de nouveau que $\omega = 2^{\omega}$, on a $\omega^2 =
+2^{\omega2}$ et $\omega^3 = 2^{\omega3}$ en binaire : on a donc
+$\omega \oplus \omega^2 \oplus \omega^3 = 2^{\omega3} + 2^{\omega2} +
+2^\omega = \omega^3 + \omega^2 + \omega =: \gamma$ : la valeur de
+Grundy de la position est $\gamma \neq 0$, et c'est cdonc une
+position N.
+
+Comme dans la question (b), on cherche à annuler la valeur de Grundy,
+autrement dit remplacer le nombre $\alpha$ de bâtonnets d'une ligne
+par $\alpha \oplus \gamma$ (où $\gamma = \omega^3 + \omega^2 +
+\omega$, qu'il vaut mieux penser comme $2^{\omega3} \oplus 2^{\omega2}
+\oplus 2^\omega$) d'une manière à ce que le résultat soit plus petit.
+On vérifie facilement que la seule possibilité est de réduire la ligne
+ayant $\omega^3$ bâtonnets à $\omega^3 \oplus \gamma = \omega^2 +
+\omega$ bâtonnets. Bref, le seul coup gagnant est
+$(\omega,\omega^2,\omega^3) \to (\omega,\omega^2,\omega^2+\omega)$.
+\end{corrige}
+
+(e) $(\omega,\omega^\omega,\omega^{\omega^\omega})$
+
+\begin{corrige}
+En se rappelant une fois de plus que $\omega = 2^{\omega}$, on a
+$\omega^\omega = (2^\omega)^\omega = 2^{\omega\times\omega} =
+2^{\omega^2}$, et $\omega^{\omega^\omega} = (2^\omega)^{\omega^\omega}
+= 2^{\omega\times \omega^{\omega}} = 2^{\omega^{1+\omega}} =
+2^{\omega^\omega}$. Le raisonnement est alors exactement analogue à
+la question (d) (car la seule chose qui importe dans cette question
+ait qu'on ait affaire à trois puissances de $2$ distinctes) : la
+valeur de Grundy de la position est $\gamma := 2^{\omega^\omega} +
+2^{\omega^2} + 2^\omega = \omega^{\omega^\omega} + \omega^\omega +
+\omega \neq 0$ donc c'est une position N, et le seul coup gagnant est
+$(\omega,\omega^\omega,\omega^{\omega^\omega}) \to
+(\omega,\omega^\omega,\omega^\omega + \omega)$.
+\end{corrige}
+
+(f) $(\varepsilon_1, \varepsilon_2, \varepsilon_3)$ où $\varepsilon_0
+< \varepsilon_1 < \varepsilon_2 < \varepsilon_3$ sont les quatre
+premiers ordinaux\footnote{On peut définir $\varepsilon_{n+1}$ comme
+ la limite, c'est-à-dire la borne supérieure, de la suite
+ $(u_k)_{k<\omega}$ strictement croissante définie par $u_0 =
+ (\varepsilon_n) + 1$ et $u_{k+1} = \omega^{u_k}$, c'est-à-dire $u_1
+ = \omega^{(\varepsilon_n) + 1}$ et $u_2 =
+ \omega^{\omega^{(\varepsilon_n) + 1}}$, etc., mais on n'aura pas
+ besoin de ce fait.} vérifiant $\varepsilon = \omega^\varepsilon$.
+
+\begin{corrige}
+Pour $i \in \{1,2,3\}$ (ou n'importe quel ordinal, en fait), on a
+$\varepsilon_i = \omega^{\varepsilon_i} = (2^\omega)^{\varepsilon_i} =
+2^{\omega\cdot\varepsilon_i} = 2^{\omega\cdot\omega^{\varepsilon_i}} =
+2^{\omega^{1+\varepsilon_i}} = 2^{\omega^{\varepsilon_i}} =
+2^{\varepsilon_i}$ (en utilisant au passage le fait, facilement
+vérifié, que $1 + \rho = \rho$ quel que soit l'ordinal infini $\rho$).
+Le raisonnement est alors exactement analogue aux questions (d) et (e)
+(car la seule chose qui importe dans ces questions ait qu'on ait
+affaire à trois puissances de $2$ distinctes) : la valeur de Grundy de
+la position est $\gamma := 2^{\varepsilon_3} + 2^{\varepsilon_2} +
+2^{\varepsilon_1} = \varepsilon_3 + \varepsilon_2 + \varepsilon_1 \neq
+0$ donc c'est une position N, et le seul coup gagnant est
+$(\varepsilon_1, \varepsilon_2, \varepsilon_3) \to (\varepsilon_1,
+\varepsilon_2, \varepsilon_2 + \varepsilon_1)$.
+\end{corrige}
+
+(g) Donner un exemple de position N du jeu de nim (de préférence fini)
+avec un nombre distinct de bâtonnets sur chaque ligne (i.e., les
+$\alpha_i$ sont deux à deux distincts), où il existe strictement plus
+qu'un coup gagnant pour le joueur qui doit jouer. (Pour indication,
+ceci est possible à partir de trois lignes de bâtonnets.)
+
+\begin{corrige}
+On cherche donc une position $(n_1,n_2,n_3)$ avec trois lignes de
+bâtonnets, de valeur de Grundy $g := n_1 \oplus n_2 \oplus n_3$, telle
+qu'au moins deux des trois quantités $n_i \oplus g$ soit strictement
+inférieure au $n_i$ correspondant (le raisonnement étant expliqué à la
+question (b)), en prenant note du fait que $n_1 \oplus g = n_2 \oplus
+n_3$ et de façon analogue pour les trois autres. On cherche donc, par
+exemple, à avoir $n_1 \oplus n_3 < n_2$ et $n_1 \oplus n_2 < n_3$.
+Ceci n'est pas difficile en prenant par exemple pour $n_1$ une
+puissance de $2$ qui existe dans l'écriture binaire de $n_2$ et
+de $n_3$ mais telle qu'en l'enlevant on obtienne des nombres
+strictement plus petits. Par exemple, la position $(2,6,7)$ (de
+valeur de Grundy $2\oplus 6\oplus 7 = 3 =: g$) admet des coups
+gagnants vers $(2,5,7)$ ou $(2,6,4)$ ou même $(1,6,7)$.
+\end{corrige}
+
+
+%
+%
+%
+
+\exercice
+
+Si $G$ est un graphe orienté bien-fondé (qu'on peut considérer comme
+l'ensemble des positions d'un jeu combinatoire auquel il ne manque que
+l'information, sans pertinence ici, de la position initiale), on
+rappelle qu'on a défini la fonction rang
+\[
+\rk(x) := \sup\nolimits^+\{\rk(y) : y\in\outnb(x)\} = \sup\,\{\rk(y)+1 :
+y\in\outnb(x)\}
+\]
+(en notant $\sup^+ S$ le plus petit ordinal strictement supérieur à
+tous les ordinaux de $S$ et $\sup S$ le plus petit ordinal supérieur
+ou égal à tous les ordinaux de $S$, et $\outnb(x)$ l'ensemble des
+voisins sortants de $x$), et la fonction de Grundy
+\[
+\gr(x) := \mex\,\{\gr(y) : y\in\outnb(x)\}
+\]
+(où $\mex S$ désigne le plus petit ordinal qui n'est pas dans $S$),
+— ces définitions ayant bien un sens par induction bien-fondée. La
+première mesure en quelque sorte la durée du jeu si les deux joueurs
+coopèrent pour le faire durer aussi longtemps que possible, et la
+seconde nous donne notamment l'information de quel joueur a une
+stratégie gagnante.
+
+On va maintenant définir une fonction qui mesure en quelque sorte la
+durée du jeu si le joueur perdant cherche à perdre aussi lentement que
+possible tandis que le joueur gagnant cherche à gagner aussi vite que
+possible. Précisément, on pose
+\[
+\left\{
+\begin{array}{ll}
+\dur(x) := \sup\,\{\dur(y)+1 : y\in\outnb(x)\}
+&\;\;\text{si $\gr(x) = 0$}\\
+\dur(x) := \min\,\{\dur(y)+1 : y\in\outnb(x)\text{~et~}\gr(y)=0\}
+&\;\;\text{si $\gr(x) \neq 0$}\\
+\end{array}
+\right.
+\]
+(où $\min S$ désigne le plus petit ordinal de $S$, si $S$ est un
+ensemble non-vide d'ordinaux).
+
+(1) Expliquer pourquoi cette définition a bien un sens (on
+prendra garde au fait que $\min\varnothing$ n'est pas défini).
+
+\begin{corrige}
+Lorsque $\gr(x) \neq 0$, il existe au moins un voisin sortant $y \in
+\outnb(x)$ tel que $\gr(y) = 0$ (car le $\mex$ est la plus petite
+valeur exclue: si un tel $y$ n'existait pas, le $\mex$ serait $0$) :
+ceci assure que le $\min$ dans la deuxième ligne de la définition
+porte toujours sur un ensemble non-vide.
+
+En-dehors de ce fait, la définition ne pose pas de problème : le
+$\sup$ d'un ensemble d'ordinaux existe toujours, et la récursivité de
+la définition est légitime par induction bien-fondée, c'est-à-dire que
+$\dur(x)$ peut faire appel aux valeurs de $\dur(y)$ pour des voisins
+sortants $y$ de $x$. (Le fait de faire appel à $\gr(y)$ n'est pas un
+problème car on sait que $\gr$ est bien défini, et la distinction en
+cas est légitime de toute manière.)
+\end{corrige}
+
+(2) Expliquer rapidement et informellement pourquoi $\dur(x)$
+correspond bien à l'explication intuitive qu'on a donnée.
+
+\begin{corrige}
+Quand $x$ est une position avec $\gr(x) = 0$, c'est-à-dire une
+position P, le joueur qui doit jouer est le joueur perdant (n'ayant
+pas de stratégie gagnante) : il joue donc vers une position $y$ le
+faisant perdre le plus lentement possible, c'est-à-dire avec $\dur(y)$
+aussi grand que possible, d'où la définition $\dur(x) =
+\sup\,\{\dur(y)+1 : y\in\outnb(x)\}$ dans ce cas (le $+1$ sert à
+compter le coup joué par ce joueur).
+
+Au contraire, lorsque $\gr(x) \neq 0$, c'est-à-dire que $x$ est une
+position N, le joueur qui doit jouer est le joueur gagnant : il joue
+donc vers une position $y$ le faisant gagner le plus rapidement
+possible, c'est-à-dire avec $\dur(y)$ aussi petit que possible parmi
+les coups $x\to y$ gagnants, autrement dit ceux pour lesquels $\gr(y)
+= 0$, d'où la définition $\dur(x) = \min\,\{\dur(y)+1 :
+y\in\outnb(x)\;\text{~et~}\;\gr(y)=0\}$ dans ce cas (le $+1$ sert à
+compter le coup joué par ce joueur).
+\end{corrige}
+
+(3) Sur le graphe $G$ représenté ci-dessous, calculer chacune des
+fonctions $\rk$, $\gr$ et $\dur$ (les lettres servent simplement à
+étiqueter les sommets) :
+
+\begin{center}\footnotesize
+\begin{tikzpicture}[>=stealth,thick,text width=5bp,text height=5bp,text depth=0bp]
+\node (nA) at (0bp,0bp) [draw,circle] {A};
+\node (nB) at (0bp,-40bp) [draw,circle] {B};
+\node (nC) at (40bp,0bp) [draw,circle] {C};
+\node (nD) at (40bp,-40bp) [draw,circle] {D};
+\node (nE) at (80bp,0bp) [draw,circle] {E};
+\node (nF) at (80bp,-40bp) [draw,circle] {F};
+\node (nG) at (120bp,-40bp) [draw,circle] {G};
+\draw[->] (nA) -- (nB);
+\draw[->] (nA) -- (nC);
+\draw[->] (nB) -- (nD);
+\draw[->] (nB) to[out=315,in=225] (nG);
+\draw[->] (nC) -- (nD);
+\draw[->] (nC) -- (nE);
+\draw[->] (nD) -- (nF);
+\draw[->] (nE) -- (nF);
+\draw[->] (nF) -- (nG);
+\draw[->] (nE) to[out=0,in=90] (nG);
+\end{tikzpicture}
+\end{center}
+\vskip-\baselineskip
+Si on joue à partir du sommet A comme position initiale et que, comme
+suggéré dans la définition de $\dur$, le joueur perdant cherche à
+perdre aussi lentement que possible tandis que le joueur gagnant
+cherche à gagner aussi vite que possible, quelle sera le déroulement
+du jeu ?
+
+\begin{corrige}
+Les sommets ont été fort sympathiquement étiquetés dans un ordre
+compatible avec le rang (tri topologique), c'est-à-dire que chaque
+arête pointe vers un sommet situé strictement après dans l'ordre
+alphabétique. On effectue donc les calculs dans l'ordre alphabétique
+inversé. On trouve, pour le rang :
+
+\begin{center}\footnotesize
+\begin{tikzpicture}[>=stealth,thick,text width=5bp,text height=5bp,text depth=0bp]
+\node (nA) at (0bp,0bp) [draw,circle] {$4$};
+\node (nB) at (0bp,-40bp) [draw,circle] {$3$};
+\node (nC) at (40bp,0bp) [draw,circle] {$3$};
+\node (nD) at (40bp,-40bp) [draw,circle] {$2$};
+\node (nE) at (80bp,0bp) [draw,circle] {$2$};
+\node (nF) at (80bp,-40bp) [draw,circle] {$1$};
+\node (nG) at (120bp,-40bp) [draw,circle] {$0$};
+\draw[->] (nA) -- (nB);
+\draw[->] (nA) -- (nC);
+\draw[->] (nB) -- (nD);
+\draw[->] (nB) to[out=315,in=225] (nG);
+\draw[->] (nC) -- (nD);
+\draw[->] (nC) -- (nE);
+\draw[->] (nD) -- (nF);
+\draw[->] (nE) -- (nF);
+\draw[->] (nF) -- (nG);
+\draw[->] (nE) to[out=0,in=90] (nG);
+\end{tikzpicture}
+\end{center}
+\vskip-\baselineskip
+
+Pour la valeur de Grundy :
+
+\begin{center}\footnotesize
+\begin{tikzpicture}[>=stealth,thick,text width=5bp,text height=5bp,text depth=0bp]
+\node (nA) at (0bp,0bp) [draw,circle] {$0$};
+\node (nB) at (0bp,-40bp) [draw,circle] {$1$};
+\node (nC) at (40bp,0bp) [draw,circle] {$1$};
+\node (nD) at (40bp,-40bp) [draw,circle] {$0$};
+\node (nE) at (80bp,0bp) [draw,circle] {$2$};
+\node (nF) at (80bp,-40bp) [draw,circle] {$1$};
+\node (nG) at (120bp,-40bp) [draw,circle] {$0$};
+\draw[->] (nA) -- (nB);
+\draw[->] (nA) -- (nC);
+\draw[->] (nB) -- (nD);
+\draw[->] (nB) to[out=315,in=225] (nG);
+\draw[->] (nC) -- (nD);
+\draw[->] (nC) -- (nE);
+\draw[->] (nD) -- (nF);
+\draw[->] (nE) -- (nF);
+\draw[->] (nF) -- (nG);
+\draw[->] (nE) to[out=0,in=90] (nG);
+\end{tikzpicture}
+\end{center}
+\vskip-\baselineskip
+
+Et pour la fonction $\dur$ (afin de rendre le calcul plus facile à
+suivre, on a entouré en double les sommets de valeur de Grundy $0$) :
+
+\begin{center}\footnotesize
+\begin{tikzpicture}[>=stealth,thick,text width=5bp,text height=5bp,text depth=0bp]
+\node (nA) at (0bp,0bp) [draw,double,circle] {$4$};
+\node (nB) at (0bp,-40bp) [draw,circle] {$1$};
+\node (nC) at (40bp,0bp) [draw,circle] {$3$};
+\node (nD) at (40bp,-40bp) [draw,double,circle] {$2$};
+\node (nE) at (80bp,0bp) [draw,circle] {$1$};
+\node (nF) at (80bp,-40bp) [draw,circle] {$1$};
+\node (nG) at (120bp,-40bp) [draw,double,circle] {$0$};
+\draw[->] (nA) -- (nB);
+\draw[->] (nA) -- (nC);
+\draw[->] (nB) -- (nD);
+\draw[->] (nB) to[out=315,in=225] (nG);
+\draw[->] (nC) -- (nD);
+\draw[->] (nC) -- (nE);
+\draw[->] (nD) -- (nF);
+\draw[->] (nE) -- (nF);
+\draw[->] (nF) -- (nG);
+\draw[->] (nE) to[out=0,in=90] (nG);
+\end{tikzpicture}
+\end{center}
+\vskip-\baselineskip
+
+Si on joue à partir de la position A, le premier joueur, appelons-la
+Alice, est en position perdante car la valeur de Grundy est nulle, et
+va jouer vers C car c'est ce qui maximise la fonction $\dur$ ; son
+adversaire, appelons-le Bob, joue alors vers D car c'est le seul coup
+gagnant, après quoi Alice joue vers F (seul coup possible) et Bob joue
+vers G.
+\end{corrige}
+
+(4) Montrer que, dans n'importe quel graphe $G$ bien-fondé, et pour
+n'importe quel sommet $x$ de $G$, on a $\dur(x) \leq \rk(x)$.
+
+\begin{corrige}
+On montre par induction bien-fondée que $\dur(x) \leq \rk(x)$. On
+peut donc faire l'hypothèse qu'on sait que $\dur(y) \leq \rk(y)$ pour
+tout voisin sortant $y \in \outnb(x)$ (hypothèse d'induction). Dans
+le cas où $\gr(x) = 0$, on a $\dur(x) = \sup\,\{\dur(y)+1 :
+y\in\outnb(x)\}$, qui est $\leq \sup\,\{\rk(y)+1 : y\in\outnb(x)\}$
+par hypothèse d'induction (il est clair qu'augmenter au sens large
+tous les éléments d'un ensemble d'ordinaux augmente son $\sup$ au sens
+large), et ceci vaut $\rk(x)$ par définition. Dans le cas où $\gr(x)
+\neq 0$, on a $\dur(x) = \min\,\{\dur(y)+1 :
+y\in\outnb(x)\;\text{~et~}\;\gr(y)=0\} \leq \sup\,\{\dur(y)+1 :
+y\in\outnb(x)\;\text{~et~}\;\gr(y)=0\} \leq \sup\,\{\dur(y)+1 :
+y\in\outnb(x)\}$ de façon évidente ($\min S \leq \sup S$ est clair),
+et pour la même raison que précédemment, ceci est $\leq
+\sup\,\{\rk(y)+1 : y\in\outnb(x)\} = \rk(x)$.
+\end{corrige}
+
+
+%
+%
+%
+
+\exercice
+
+Soit $G$ un graphe orienté dont l'ensemble des sommets est (au plus)
+dénombrable, et soit $x_0$ un sommet de $G$. (Il n'y a pas d'autre
+hypothèse sur $G$, par exemple on ne suppose pas que $G$ est
+bien-fondé.)
+
+On considère le jeu suivant. Deux joueurs s'affrontent, qu'on
+appellera \emph{le Fugueur} et \emph{le Borneur}. Le Fugueur
+commence, après quoi ils jouent tour à tour. En partant de $x_0$,
+chaque joueur, quand vient son tour, choisit un voisin sortant de la
+position actuelle $x$ \emph{ou bien} choisit de conserver $x$ ;
+autrement dit, il choisit un élément de $\outnb(x) \cup \{x\}$. (Pour
+être parfaitement clair : au premier tour, le Fugueur choisit $x_1 \in
+\outnb(x_0) \cup \{x_0\}$, puis le Borneur choisit $x_2 \in
+\outnb(x_1) \cup \{x_1\}$, et ainsi de suite.) Le jeu dure infiniment
+longtemps (manifestement, les règles permettent toujours à chaque
+joueur de faire un coup). Au bout d'un nombre infini de coups, on
+considère la suite $(x_n)_{n\in\mathbb{N}}$ de toutes les positions
+traversées :
+\begin{itemize}
+\item si cette suite est d'image finie (c'est-à-dire que l'ensemble
+ $\{x_n : n\in\mathbb{N}\}$ de toutes les positions traversées est
+ fini), alors le Borneur a gagné ;
+\item sinon, le Fugueur a gagné.
+\end{itemize}
+
+(1) Montrer, en appliquant un des résultats du cours, que l'un des
+joueurs a nécessairement une stratégie gagnante (on ne demande pas de
+préciser lequel). On pourra préalablement montrer que pour toute
+partie $F \subseteq G$, la partie $F^{\mathbb{N}} \subseteq
+G^{\mathbb{N}}$ est \emph{fermée} (pour la topologie sur
+$G^{\mathbb{N}}$ produit de la topologie
+discrète\footnote{C'est-à-dire celle qui a été étudiée en cours.}), et
+en déduire une propriété de l'ensemble
+$\bigcup_{F\text{~fini~}\subseteq G} F^{\mathbb{N}}$ réunion des
+$F^{\mathbb{N}}$ où $F$ parcourt toutes les parties \emph{finies}
+de $G$.
+
+\begin{corrige}
+Pour commencer, montrons que si $F$ est une partie de $G$ alors
+$F^{\mathbb{N}} \subseteq G^{\mathbb{N}}$ est fermée. Ceci revient à
+montrer que son complémentaire est ouvert, autrement dit, que si
+$\dblunderline{x} \in G^{\mathbb{N}}$ n'est pas dans $F^{\mathbb{N}}$,
+alors il existe un voisinage fondamental de $\dblunderline{x}$ qui ne
+rencontre pas $F^{\mathbb{N}}$. Or si $\dblunderline{x} \in
+G^{\mathbb{N}}$ n'est pas dans $F^{\mathbb{N}}$, c'est qu'elle a une
+valeur $x_\ell$ qui n'est pas dans $F$, et toute suite commençant par
+$x_0,\ldots,x_\ell$ n'est pas dans $F^{\mathbb{N}}$, c'est-à-dire que
+le voisinage fondamental $V_{\ell+1}(\dblunderline{x})$ est inclus
+dans le complémentaire de $F^{\mathbb{N}}$. Ceci achève de montrer
+que $F^{\mathbb{N}} \subseteq G^{\mathbb{N}}$ est fermée.
+
+Maintenant, l'ensemble $\mathscr{F}$ des parties finies d'un ensemble
+(au plus) dénombrable, en l'occurrence, $G$, est (au plus)
+dénombrable. Ce fait peut être tenu pour acquis, mais rappelons
+pourquoi il est vrai : en effet, si $G = \{g_i : i\in\mathbb{N}\}$,
+alors on peut par exemple énumérer $\mathscr{F}$ à travers les
+écritures binaires, ou plus précisément, $\mathscr{F} = \{F_n :
+n\in\mathbb{N}\}$ où $F_n$ désigne la partie finie
+$\{g_{i_0},\ldots,g_{i_r}\}$ lorsque $n = 2^{i_0} + \cdots + 2^{i_r}$
+avec $i_0,\ldots,i_r$ entiers naturels distincts (autrement dit, $F_0
+= \varnothing$, $F_1 = \{g_0\}$, $F_2 = \{g_1\}$, $F_3 = \{g_0,g_1\}$,
+etc.). Peu importe le fait qu'il y ait des répétitions dans
+l'énumération $F_n$ (un ensemble surjecté par $\mathbb{N}$ est encore
+dénombrable).
+
+Ceci nous permet de dire que $\bigcup_{F\text{~fini~}\subseteq G}
+F^{\mathbb{N}}$, autrement dit $\bigcup_{F \in \mathscr{F}}
+F^{\mathbb{N}}$, est une réunion dénombrable de fermés
+dans $G^{\mathbb{N}}$. Comme un fermé est borélien, c'est une réunion
+dénombrable de boréliens, donc c'est encore un borélien.
+
+Enfin, remarquons que dire que l'ensemble $\{x_n : n\in\mathbb{N}\}$
+est fini revient à dire qu'il est inclus un ensemble fini $F$,
+autrement dit, que la suite $\dblunderline{x} = (x_n)$ appartient à
+$F^{\mathbb{N}}$ pour un certain ensemble fini $F$, ou, ce qui revient
+au même, qu'elle appartient à $\bigcup_{F \in \mathscr{F}}
+F^{\mathbb{N}}$.
+
+On a donc affaire à un jeu de Gale-Stewart défini par l'ensemble de
+suites $B := \bigcup_{F \in \mathscr{F}} F^{\mathbb{N}}$ borélien (ou
+par son complémentaire $A := G^{\mathbb{N}} \setminus B$ si on prend
+le point de vue du Fugueur). Le théorème de détermination borélienne
+de Martin assure que l'un des joueurs a forcément une stratégie
+gagnante.
+\end{corrige}
+
+(2) Indépendamment de la question précédente, donner un exemple de
+couple $(G,x_0)$ pour lequel le Borneur possède une stratégie gagnante
+à ce jeu. Donner un exemple pour lequel le Fugueur en possède une.
+(On cherchera à donner des exemples aussi simples que possibles.)
+
+\begin{corrige}
+Si $G$ est le graphe formé du seul sommet $x_0$, alors le Borneur
+gagne trivialement (la suite sera la suite constante).
+
+Si $G$ est le graphe formé des entiers naturels avec une arête $i \to
+j$ lorsque $i<j$ (autrement dit des petits entiers naturels vers les
+grands), ou même seulement $i \to i+1$, alors le Fugueur a une
+stratégie gagnante consistant à jouer de $i$ vers $i+1$, ce qui assure
+que la suite $(x_{2i+1})$ des positions choisies par le Fugueur est
+strictement croissante quoi que fasse le Borneur (il ne peut pas
+revenir en arrière), et notamment, elle n'est pas d'image finie.
+\end{corrige}
+
+
+%
+%
+%
+
+\exercice
+
+On considère une variante \emph{à somme (possiblement) non-nulle} de
+Pierre-Papier-Ciseaux, à savoir le jeu en forme normale défini par la
+matrice de gain suivante :
+\begin{center}
+\begin{tabular}{r|ccc}
+$\downarrow$Alice, Bob$\rightarrow$&$U$&$V$&$W$\\\hline
+$U$&$x,x$&$-1,+1$&$+1,-1$\\
+$V$&$+1,-1$&$x,x$&$-1,+1$\\
+$W$&$-1,+1$&$+1,-1$&$x,x$\\
+\end{tabular}
+\end{center}
+où $x$ est un réel et, pour plus de commodité, on a écrit $U$ pour
+« Pierre », $V$ pour « Papier » et $W$ pour « Ciseaux ». (La ligne
+correspond à l'option choisie par Alice, la colonne à l'option de Bob,
+et chaque case indique le gain d'Alice suivi du gain de Bob.)
+
+Le but de l'exercice est d'étudier les équilibres de Nash de ce jeu.
+
+(On prendra bien note, pour simplifier les raisonnements en cas, du
+fait que les options ont une symétrie cyclique\footnote{C'est-à-dire
+ que remplacer $U$ par $V$ et $V$ par $W$ et $W$ par $U$ ne change
+ rien au jeu.}, et que les joueurs ont eux aussi des rôles
+symétriques.)
+
+(1) Considérons le profil de stratégies mixtes dans lequel les deux
+joueurs choisissent chacun chaque option avec probabilité
+$\frac{1}{3}$ (c'est-à-dire la stratégie qui est optimale dans le cas
+à somme nulle). Pour quelle(s) valeur(s) de $x$ ce profil est-il un
+équilibre de Nash ?
+
+\begin{corrige}
+Pour des raisons de symétrie, si l'un des joueurs joue cette stratégie
+mixte $\frac{1}{3}U + \frac{1}{3}V + \frac{1}{3}W$, le gain espéré de
+chacun des deux joueurs est le même quelle que soit la stratégie pure,
+donc aussi mixte, de l'autre joueur. Cette valeur se calcule
+d'ailleurs aisément (comme somme des trois colonnes, ou des trois
+lignes, de la matrice de gains, affectées des
+coefficients $\frac{1}{3}$) : c'est $\frac{1}{3}x$ ; mais la seule
+chose qui importe est que l'adversaire ait le même gain espéré quelle
+que soit la stratégie pure, donc aussi mixte, qu'il choisit : il n'a
+donc pas intérêt à changer unilatéralement sa stratégie. Il s'agit
+donc \emph{toujours} d'un équilibre de Nash, quelle que soit la valeur
+de $x$.
+\end{corrige}
+
+\emph{On suppose dorénavant que $x<-1$.}
+
+(2) Existe-t-il un équilibre de Nash dans lequel Alice joue purement
+$U$ ? (On raisonnera les options pouvant être dans le support de la
+stratégie de Bob en réponse.) En déduire tous les équilibres de Nash
+dans lesquels au moins un joueur joue une stratégie pure.
+
+\begin{corrige}
+Si Alice joue purement $U$, les gains de Bob pour les différentes
+stratégies pures de sa réponse sont $x$ pour $U$, $+1$ pour $V$ et
+$-1$ pour $W$ d'après la matrice de gains. Comme $+1 > -1 > x$, la
+seule option qui peut faire partie du support d'une meilleure réponse
+de Bob est $V$, autrement dit, si Alice joue purement $U$ dans un
+équilibre de Nash, Bob répond forcément purement $V$. Mais par le
+même raisonnement (compte tenu de la symétrie cyclique des options et
+de la symétrie des joueurs), si Bob joue purement $V$, Alice joue $W$
+(et pas $U$ comme supposé). Il ne peut donc pas y avoir d'équilibre
+de Nash dans lequel Alice joue purement $U$. Et de nouveau par
+symétrie cyclique des options et symétrie des joueurs, il ne peut y
+avoir aucun équilibre de Nash dans lequel un joueur jouerait une
+stratégie pure.
+\end{corrige}
+
+(3) Dans cette question et la suivante, envisageons un équilibre de
+Nash dans lequel Alice joue la stratégie mixte $pU + (1-p)V$ avec
+$0<p<1$. Supposons dans cette question que Bob réponde avec un une
+stratégie mixte ayant elle aussi $\{U,V\}$ comme support. Montrer que
+$p = \frac{x+1}{2x}$ et que le gain de Bob est $\frac{x^2+1}{2x}$. En
+utilisant le fait que $\frac{x^2+1}{2x} < -\frac{1}{x}$ lorsque $x<-1$
+(qu'on admettra pour ne pas perdre son temps en calculs inutiles), en
+déduire qu'un tel équilibre de Nash n'existe pas.
+
+\begin{corrige}
+Si Alice joue $pU + (1-p)V$, les gains espérés de Bob pour les
+différences stratégies pures de sa réponse sont $px-(1-p) =
+-1+(x+1)p$ pour $U$, $p + (1-p)x = x-(x-1)p$ pour $V$ et $-p + (1-p) =
+1-2p$ pour $W$. Si une meilleure réponse de Bob a $\{U,V\}$ comme
+support, ces deux options doivent apporter le même gain espéré,
+c'est-à-dire qu'on doit avoir $-1+(x+1)p = x-(x-1)p$, ce qui équivaut
+à $p = \frac{x+1}{2x}$, et le gain en question est $\frac{x^2+1}{2x}$,
+tandis que le gain espéré pour $W$ est alors $1-2p = -\frac{1}{x}$.
+D'après l'inégalité $\frac{x^2+1}{2x} < -\frac{1}{x}$, l'option $W$
+fournit un meilleur gain espéré pour Bob, donc $\{U,V\}$ ne peut pas
+être le support d'une meilleure réponse de Bob à $pU + (1-p)V$
+d'Alice.
+\end{corrige}
+
+(4) On considère toujours un équilibre de Nash dans lequel Alice joue
+la stratégie mixte $pU + (1-p)V$ avec $0<p<1$. Supposons maintenant
+que Bob réponde avec une stratégie mixte ayant (au moins) $U$ et $W$
+dans son support support. Montrer que $p = \frac{2}{x+3}$ (et
+que $x\neq -3$) ; en utilisant le fait que $\frac{2}{x+3} > 1$ lorsque
+$-3<x<-1$ et que $\frac{2}{x+3} < 0$ lorsque $x < -3$ (de nouveau, on
+admettra ces faits pour ne pas perdre de temps en calculs), en déduire
+qu'un tel équilibre de Nash n'existe pas.
+
+\begin{corrige}
+On a dit dans la question (3) que si Alice joue $pU + (1-p)V$, les
+gains espérés de Bob pour les différences stratégies pures de sa
+réponse sont $px-(1-p) = -1+(x+1)p$ pour $U$, $p + (1-p)x =
+x-(x-1)p$ pour $V$ et $-p + (1-p) = 1-2p$ pour $W$. Si une meilleure
+réponse de Bob a $U$ et $W$ dans son support, ces deux options doivent
+apporter le même gain espéré, c'est-à-dire qu'on doit avoir $-1+(x+1)p
+= 1-2p$, ce qui équivaut à $(x+3)p = 3$, donc $x\neq 3$ et $p =
+\frac{2}{x+3}$. D'après les inégalités admises, $p$, qui devrait être
+entre $0$ et $1$, ne l'est jamais si $x<-1$, donc un tel équilibre de
+Nash n'existe pas.
+\end{corrige}
+
+(5) Expliquer soigneusement pourquoi les questions (2) à (4) montrent
+que dans tout équilibre de Nash du jeu considéré, les deux joueurs
+jouent une stratégie mixte ayant $\{U,V,W\}$ comme support (i.e.,
+aucun ensemble strictement plus petit n'est possible).
+
+\begin{corrige}
+On a vu en (2) qu'il n'existe aucun équilibre de Nash dans lequel un
+joueur joue une stratégie pure. Supposons maintenant un équilibre de
+Nash dans lequel un joueur a deux options dans son support. Par
+symétrie, sans perte de généralité, on peut supposer que c'est Alice
+et que ces deux options sont $U$ et $V$. Comme les stratégies pures
+sont exclues, les supports possibles de la réponse de Bob sont :
+$\{U,V\}$, $\{U,W\}$, $\{V,W\}$ et $\{U,V,W\}$. Dans la question (3)
+on a exclu $\{U,V\}$ ; dans la question (4), on a exclu $\{U,W\}$ et
+$\{U,V,W\}$. Reste le cas où le support de la stratégie de Bob est
+$\{V,W\}$ (tandis que celui d'Alice est, on le rappelle, $\{U,V\}$).
+Mais quitte à effectuer une symétrie cyclique des options ($U\to W\to
+V\to U$) et échanger les joueurs, cela revient au cas où le support de
+la stratégie d'Alice est $\{U,V\}$ et celui de Bob est $\{U,W\}$ : or
+on a déjà exclu ce cas. Il ne reste donc aucune possibilité.
+\end{corrige}
+
+(6) Envisageons maintenant un équilibre de Nash dans lequel Alice joue
+une stratégie mixte $pU + p'V + (1-p-p')W$ avec $p>0$, $p'>0$ et
+$1-p-p'>0$ et Bob répond par une stratégie ayant elle aussi
+$\{U,V,W\}$ comme support. Écrire un système de deux équations
+linéaires vérifié par $p,p'$, justifier que ce système est
+non-dégénéré et conclure. Quels sont tous les équilibres de Nash du
+jeu ?
+
+\begin{corrige}
+Si Alice joue $pU + p'V + (1-p-p')W$, les gains espérés de Bob pour
+les différences stratégies pures de sa réponse sont $px - p' +
+(1-p-p') = 1 + (x-1)p - 2p'$ pour $U$, $p + p' x - (1-p-p') = -1 + 2p
++ (x+1)p'$ pour $V$ et $-p + p' + (1-p-p')x = x -(x+1)p -
+(x-1)p'$ pour $W$. Si une meilleure réponse de Bob a $\{U,V,W\}$
+comme support, ces trois options doivent apporter le même gain espéré,
+c'est-à-dire que $1 + (x-1)p - 2p' = -1 + 2p + (x+1)p' = x -(x+1)p -
+(x-1)p'$, ou (en soustrayant, disons, le premier membre aux deux
+autres) :
+\[
+\begin{aligned}
+-(x-3)p + (x+3)p' &= 2\\
+- 2xp - (x-3)p' &= -(x-1)
+\end{aligned}
+\]
+Le déterminant de ce système est $(x-3)^2 + 2x(x+3) = 3(x^2+3)$ qui
+est non nul quel que soit $x$, donc le système est non-dégénéré : la
+solution $p=p'=\frac{1}{3}$ trouvée en (1) est donc la seule solution.
+
+Bref, on a montré que le seul équilibre de Nash dans lequel les
+supports des stratégies d'Alice et Bob sont $\{U,V,W\}$ est celui
+décrit en (1) ; comme on a vu en (5) que ceci est la seule possibilité
+de support, il s'agit du seul équilibre de Nash du jeu.
+\end{corrige}
+
+
+
+
+
+%
+%
+%
+\end{document}
diff --git a/misc/randomize-test.pl b/misc/randomize-test.pl
new file mode 100755
index 0000000..42ff144
--- /dev/null
+++ b/misc/randomize-test.pl
@@ -0,0 +1,183 @@
+#! /usr/local/bin/perl -w
+
+use strict;
+use warnings;
+
+use Digest::SHA qw(sha256);
+
+use Getopt::Std;
+
+my %opts;
+
+getopts("c:N:n:", \%opts);
+
+my @preamble = (); # Preamble lines
+my @postamble = (); # Postamble lines
+my @questions = (); # Array of question hashrefs
+# Each question hash has keys:
+# {question}: arrayref with question lines
+# {answers}: arrayref of arrayrefs with answer lines, correct answer first
+# {varid}: idnex in qvars array
+my @qvars = (); # Array of question variants
+# Each entry is an arrayref of indexes in the questions array
+
+
+my $commonseed = $opts{N} // "";
+my $seed = $opts{n} // "";
+
+### READ INPUT FILE
+
+if ( 1 ) { ## Keep following variables local
+
+my $in_preamble = 1;
+my $in_postamble = 0;
+my $in_qvar = 0;
+my $curqn;
+my $listref = \@preamble;
+my $varid;
+LINELOOP:
+while ($_ = <>) {
+ if ( $_ =~ m/^\\def\\seedval\{.*\}$/ ) {
+ $_ = "\\def\\seedval\{$seed\}\n";
+ }
+ if ( $_ =~ m/\\begin\{qcm\}/ ) {
+ die "wrong placement" unless $in_preamble;
+ die "bad format" unless $_ eq "\\begin\{qcm\}\n";
+ $in_preamble = 0;
+ $listref = undef;
+ next LINELOOP;
+ } elsif ( $_ =~ m/\\end\{qcm\}/ ) {
+ die "wrong placement" if $in_preamble;
+ die "bad format" unless $_ eq "\\end\{qcm\}\n";
+ $in_postamble = 0;
+ $listref = \@postamble;
+ next LINELOOP;
+ } elsif ( $_ =~ m/\\begin\{qvar\}/ ) {
+ die "wrong placement" if $in_preamble || $in_postamble || $in_qvar || defined($curqn);
+ die "bad format" unless $_ eq "\\begin\{qvar\}\n";
+ $in_qvar = 1;
+ push @qvars, [];
+ next LINELOOP;
+ } elsif ( $_ =~ m/\\end\{qvar\}/ ) {
+ die "wrong placement" if $in_preamble || $in_postamble || (!$in_qvar) || defined($curqn);
+ die "bad format" unless $_ eq "\\end\{qvar\}\n";
+ $in_qvar = 0;
+ next LINELOOP;
+ } elsif ( $_ =~ m/\\begin\{question\}/ ) {
+ die "wrong placement" if $in_preamble || $in_postamble || defined($curqn);
+ die "bad format" unless $_ eq "\\begin\{question\}\n";
+ my %qn = ();
+ push @qvars, [] unless $in_qvar;
+ $qn{varid} = $#qvars;
+ $qn{question} = [];
+ $qn{answers} = [];
+ push @questions, \%qn;
+ push @{$qvars[$#qvars]}, $#questions;
+ $listref = $qn{question};
+ $curqn = \%qn;
+ next LINELOOP;
+ } elsif ( $_ =~ m/\\end\{question\}/ ) {
+ die "wrong placement" if $in_preamble || $in_postamble || !defined($curqn);
+ die "bad format" unless $_ eq "\\end\{question\}\n";
+ $listref = undef;
+ $curqn = undef;
+ next LINELOOP;
+ } elsif ( $_ =~ m/\\(right)?answer/
+ && $_ !~ /\\newcommand/ && $_ !~ /\\let\\rightanswer/ ) {
+ die "wrong placement" if $in_preamble || $in_postamble || !defined($curqn);
+ die "bad format" unless $_ eq "\\answer\n" || $_ eq "\\rightanswer\n";
+ die "this is impossible" unless ref($curqn) eq "HASH" && defined $curqn->{answers};
+ die "right answer should come first" unless ($_ eq "\\rightanswer\n") == (scalar(@{$curqn->{answers}}) == 0);
+ my @ans = ();
+ push @{$curqn->{answers}}, \@ans;
+ $listref = \@ans;
+ ## no next LINELOOP here: include \answer in answer itself!
+ }
+ die "this is impossible" if $in_preamble && $in_postamble;
+ die "this is impossible" if $in_preamble && ($listref ne \@preamble);
+ die "this is impossible" if $in_postamble && ($listref ne \@postamble);
+ push @{$listref}, $_ if defined($listref);
+}
+
+}
+
+
+### RANDOMIZE
+
+my $nbqn = $opts{c} // int((scalar(@qvars)+1)/2);
+
+my @questab = ();
+my @quesanstab = ();
+
+if ( 1 ) { ## Keep following variables local
+
+my @hashlist;
+
+for ( my $u=0 ; $u<scalar(@qvars) ; $u++ ) {
+ push @hashlist, sha256("${commonseed}\n${seed}\nQV\n${u}\n");
+}
+my @qvartab = sort { $hashlist[$a] cmp $hashlist[$b] } (0..(scalar(@qvars)-1));
+
+for ( my $k=0 ; $k<scalar(@qvartab) && $k<$nbqn ; $k++ ) {
+ my $u = $qvartab[$k];
+ @hashlist = ();
+ for ( my $kv=0 ; $kv<scalar(@{$qvars[$u]}) ; $kv++ ) {
+ my $i = $qvars[$u]->[$kv];
+ push @hashlist, sha256("${commonseed}\n${seed}\nQ\n${i}\n");
+ }
+ my $kv = (sort { $hashlist[$a] cmp $hashlist[$b] } (0..(scalar(@{$qvars[$u]})-1)))[0];
+ my $i = $qvars[$u]->[$kv];
+ die "this is impossible" unless $questions[$i]->{varid} == $u;
+ push @questab, $i;
+}
+
+for ( my $i=0 ; $i<scalar(@questions) ; $i++ ) {
+ @hashlist = ();
+ my $r = $questions[$i]->{answers};
+ for ( my $j=0 ; $j<scalar(@$r) ; $j++ ) {
+ push @hashlist, sha256("${commonseed}\n${seed}\nQ\n${i}\nA\n${j}\n");
+ }
+ my @anstab;
+ @anstab = sort { $hashlist[$a] cmp $hashlist[$b] } (0..(scalar(@$r)-1));
+ push @quesanstab, \@anstab;
+}
+
+}
+
+
+### WRITE OUTPUT FILE
+
+my @correct;
+
+foreach my $l ( @preamble ) {
+ print $l;
+}
+
+print "\\begin\{qcm\}\n\n";
+
+for ( my $k=0 ; $k<scalar(@questab) ; $k++ ) {
+ my $i = $questab[$k];
+ my $qn = $questions[$i];
+ print "\\begin\{question\}\n";
+ foreach my $l ( @{$qn->{question}} ) {
+ print $l;
+ }
+ for ( my $kj=0 ; $kj<scalar(@{$quesanstab[$i]}) ; $kj++ ) {
+ my $j = $quesanstab[$i]->[$kj];
+ my $a = $qn->{answers}->[$j];
+ foreach my $l ( @{$a} ) {
+ print $l;
+ }
+ push @correct, sprintf("%d%s", $k+1, chr(ord("A")+$kj)) if $j==0;
+ }
+ print "\\end\{question\}\n\n";
+}
+
+print "\\end\{qcm\}\n\n";
+
+printf "\%\% === %s ===\n", join(" ", @correct);
+printf "\\ifcorrige\\bigskip\\noindent\\textbf{Corrigé.} %s\\fi\n", join(" ", @correct);
+
+foreach my $l ( @postamble ) {
+ print $l;
+}
diff --git a/notes-mitro206.tex b/notes-mitro206.tex
index 6bd8a82..d5fb37f 100644
--- a/notes-mitro206.tex
+++ b/notes-mitro206.tex
@@ -1022,7 +1022,9 @@ surréels de Conway.
\section{Jeux en forme normale}\label{section-games-in-normal-form}
-\setcounter{comcnt}{0}
+\setcounter{subsection}{-1}
+
+\subsection{Rappels sur les convexes}
\thingy Pour cette section, on rappelle les définitions suivants : une
\index{affine (combinaison)|see{combinaison affine}}\defin{combinaison
@@ -1035,9 +1037,19 @@ $\lambda_1,\ldots,\lambda_m$ vérifient $\sum_{i=1}^m \lambda_i = 1$
forme $\frac{\sum_{i=1}^m \lambda_i x_i}{\sum_{i=1}^m \lambda_i}$ où
$\lambda_1,\ldots,\lambda_m$ vérifient $\sum_{i=1}^m \lambda_i \neq
0$ : on parle alors de \defin{barycentre} de $x_1,\ldots,x_m$ affecté
-des coefficients $\lambda_1,\ldots,\lambda_m$).
-
-Une \index{convexe (combinaison)|see{combinaison
+des poids $\lambda_1,\ldots,\lambda_m$).
+
+Lorsque $x_1,\ldots,x_m$ sont tels que la combinaison affine est
+déterminée par ses coefficients, autrement dit lorsque l'application
+$(\lambda_1,\ldots,\lambda_m) \mapsto \sum_{i=1}^m \lambda_i x_i$
+définie sur l'ensemble des $(\lambda_1,\ldots,\lambda_m)$ tels que
+$\sum_{i=1}^m \lambda_i = 1$, est injective, on dit que
+$x_1,\ldots,x_m$ sont \index{affinement indépendants
+ (points)}\defin{affinement indépendants} (ou affinement libres). Il
+revient au même de dire que $x_2-x_1,\ldots,x_m-x_1$ sont linéairement
+indépendants.
+
+\thingy Une \index{convexe (combinaison)|see{combinaison
convexe}}\defin{combinaison convexe} est une combinaison affine
$\sum_{i=1}^m \lambda_i x_i$ où $\lambda_1,\ldots,\lambda_m$ sont
\emph{positifs} ($\lambda_1,\ldots,\lambda_m \geq 0$) et vérifient
@@ -1048,16 +1060,88 @@ Un \defin{convexe} de $\mathbb{R}^m$ est une partie stable par
combinaisons convexes (c'est-à-dire que $C$ est dit convexe lorsque si
$x_1,\ldots,x_m \in C$ et $\lambda_1,\ldots\lambda_m\geq 0$ vérifient
$\sum_{i=1}^m \lambda_i = 1$ alors $\sum_{i=1}^m \lambda_i x_i \in
-C$).
-
-Une \index{affine (application)}\textbf{application affine} $u\colon
-\mathbb{R}^p \to \mathbb{R}^q$ est une fonction qui préserve les
-combinaisons affines (autrement dit, si $\sum_{i=1}^m \lambda_i = 1$
-alors $u\Big(\sum_{i=1}^m \lambda_i x_i\Big) = \sum_{i=1}^m \lambda_i
-u(x_i)$). Il revient au même de dire que $u$ est la somme d'une
-constante (dans $\mathbb{R}^q$) et d'une application linéaire
+C$). Par associativité du barycentre, il revient au même d'imposer
+cette condition pour $m=2$, c'est-à-dire que $C \subseteq
+\mathbb{R}^m$ est convexe ssi on a $[x,y] \subseteq C$ pour tous
+$x,y\in C$, où $[x,y] := \{\lambda x + (1-\lambda) y : \lambda \in
+[0;1]\}$.
+
+Une intersection quelconque de convexes étant manifestement un
+convexe, on peut parler du plus petit convexe contenant une partie $A
+\subseteq \mathbb{R}^m$, appelé \index{convexe
+ (enveloppe)|see{enveloppe convexe}}\defin{enveloppe convexe}
+de $A$ : il s'agit simplement de l'ensemble de toutes les combinaisons
+convexes des points de $A$ (i.e., des ensembles \emph{finis} de points
+de $A$). L'enveloppe convexe d'une partie finie s'appelle un
+\defin{polytope} (convexe). L'enveloppe convexe d'une partie $A$
+affinement libre et finie\footnote{Une partie affinement libre de
+ $\mathbb{R}^m$ a au plus $m+1$ points, donc le mot « fini » est
+ redondant dans ce cadre, mais la définition peut s'étendre en
+ dimension infinie auquel cas il n'est plus inutile.} s'appelle
+\defin{simplexe} de sommets $A$.
+
+\thingy Une \index{affine (application)}\textbf{application affine}
+$u\colon \mathbb{R}^p \to \mathbb{R}^q$ est une fonction qui préserve
+les combinaisons affines (autrement dit, si $\sum_{i=1}^m \lambda_i =
+1$ alors $u\Big(\sum_{i=1}^m \lambda_i x_i\Big) = \sum_{i=1}^m
+\lambda_i\, u(x_i)$). Il revient au même de dire que $u$ est la somme
+d'une constante (dans $\mathbb{R}^q$) et d'une application linéaire
$\mathbb{R}^p \to \mathbb{R}^q$.
+On aura fréquemment besoin du fait évident suivant :
+\begin{prop}\label{affine-functions-take-extrema-at-boundary}
+Soit $A \subseteq \mathbb{R}^m$ un ensemble fini et $C$ son enveloppe
+convexe, et soit $u \colon \mathbb{R}^m \to \mathbb{R}$ une
+application affine. Alors $u(a) \leq M$ pour tout $a \in A$ implique
+$u(s) \leq M$ pour tout $s \in C$. Ou, si on préfère, $\max\{u(s) :
+s\in C\}$ existe et vaut $\max\{u(a) : a\in A\}$ (et la même
+affirmation en remplaçant $\max$ par $\min$ vaut aussi).
+\end{prop}
+
+En français : « une fonction affine sur un polytope atteint ses bornes
+sur un sommet de ce dernier ».
+
+\begin{proof}
+Tout point $s$ de $C$ est combinaison convexe de points de $A$,
+c'est-à-dire $s = \sum_{i=1}^n \lambda_i x_i$ avec $x_i \in A$ et
+$\lambda_i \geq 0$ vérifiant $\sum_{i=1}^n \lambda_i = 1$. Comme $u$
+est affine, on a alors $u(s) = \sum_{i=1}^n \lambda_i\, u(x_i)$. Si
+$u(a) \leq M$ pour tout $a\in A$, ceci implique $u(s) \leq
+\sum_{i=1}^n \lambda_i\, M = M$, comme annoncé.
+
+Pour en déduire l'affirmation sur le $\max$, il suffit d'appliquer ce
+qu'on vient de démontrer pour $M = \max\{u(a) : a\in A\}$ (qui existe
+puisque $A$ est fini) : on a $u(s) \leq M$ pour tout $s \in C$, et
+comme les éléments de $A$ sont dans $C$, ceci montre bien $\max\{u(s)
+: s\in C\} = \max\{u(a) : a\in A\}$.
+\end{proof}
+
+\begin{prop}\label{affine-functions-take-no-strict-extrema-inside}
+Reprenant le contexte de la proposition précédente ($A \subseteq
+\mathbb{R}^m$ un ensemble fini, $C$ son enveloppe convexe, et $u
+\colon \mathbb{R}^m \to \mathbb{R}$ affine), soit $M := \max_{a \in A}
+u$ (dont on vient de voir que c'est aussi $\max_{s\in C} u$) : alors
+une combinaison convexe $s$ à coefficients \emph{strictement positifs}
+de points de $A$ (i.e., $s = \sum_{i=1}^n \lambda_i x_i$ avec $x_i \in
+A$ et $\lambda_i > 0$ vérifiant $\sum_{i=1}^n \lambda_i = 1$) vérifie
+$u(s) = M$, si et seulement si chacun des points en question vérifie
+aussi cette propriété (i.e., on a $u(x_i) = M$ pour tout $i$).
+\end{prop}
+
+\begin{proof}
+On vient de voir que $u(x_i) \leq M$ pour tout $i$ implique $u(s) \leq
+M$. Si on a $u(x_j) < M$ pour un certain $j$, alors on a $\lambda_j\,
+u(x_j) < \lambda_j\, M$, donc en sommant avec les autres $\lambda_i\,
+u(x_i) \leq \lambda_i M$ on trouve $u(s) = \sum_{i=1}^n \lambda_i\,
+u(x_i) < \sum_{i=1}^n \lambda_i\, M = M$. Réciproquement, si $u(x_i)
+= M$ pour tout $i$, il est évident que $u(s) = M$.
+\end{proof}
+
+On peut aussi reformuler ce résultat en affirmant que la partie de $C$
+où $u$ prend sa valeur maximale $M$ est l'enveloppe convexe de la
+partie de $A$ où elle prend cette valeur.
+
+
\subsection{Généralités}
\begin{defn}\label{definition-game-in-normal-form}
@@ -1086,36 +1170,51 @@ fait que les joueurs peuvent également jouer de façon probabiliste, ce
qui amène à introduire la notion de stratégie mixte :
\begin{defn}\label{definition-mixed-strategy-abst}
-Donné un ensemble $B$ fini d'« options », on appelle \index{mixte (stratégie)}\defin{stratégie
- mixte} sur $B$ une fonction $s\colon B\to\mathbb{R}$ telle que
-$s(b)\geq 0$ pour tout $b\in B$ et $\sum_{b\in B} s(b) = 1$ :
-autrement dit, il s'agit d'une distribution de probabilités sur $B$.
-
-Le \defin[support (d'une stratégie mixte)]{support} de $s$ est l'ensemble des options $b\in B$ pour
-lesquelles $s(b) > 0$.
-
-Parfois, on préférera considérer la stratégie comme la combinaison
-formelle $\sum_{b\in B} s(b)\cdot b$ (« formelle » signifiant que le
-produit $t\cdot b$ utilisé ici n'a pas de sens intrinsèque : il est
-défini par son écriture ; l'écriture $\sum_{b\in B} s(b)\cdot b$ est
-donc une simple notation pour $s$). Autrement dit, ceci correspond à
-voir une stratégie mixte comme une combinaison convexe d'éléments
-de $B$, i.e., un point du simplexe affine dont les sommets sont les
-éléments de $B$. En particulier, un élément $b$ de $B$ (stratégie
-pure) sera identifié à l'élément de $S_B$ qui affecte le poids $1$
-à $b$ et $0$ à tout autre élément.
+Donné un ensemble $B$ fini d'« options », on appelle \index{mixte
+ (stratégie)}\defin{stratégie mixte} sur $B$ une fonction $s\colon
+B\to\mathbb{R}$ telle que $s(b)\geq 0$ pour tout $b\in B$ et
+$\sum_{b\in B} s(b) = 1$ : autrement dit, il s'agit d'une distribution
+de probabilités sur $B$.
+
+Le \defin[support (d'une stratégie mixte)]{support} de $s$ est
+l'ensemble des options $b\in B$ pour lesquelles $s(b) > 0$.
+\end{defn}
+
+On identifiera un élément $b$ de $B$ à la fonction (stratégie pure)
+$\delta_b \colon B\to\mathbb{R}$ qui prend la valeur $1$ en $b$ et $0$
+ailleurs (distribution de probabilités de Dirac en $b$). Ceci permet
+de considérer $B$ comme une partie de l'ensemble $S_B$ des stratégies
+mixtes, et d'identifier une stratégie mixte $s\colon B\to\mathbb{R}$
+quelconque sur $B$ à la combinaison convexe $\sum_{b\in B} s(b)\cdot
+b$ des stratégies pures : i.e., on peut considérer les stratégies
+mixtes comme des combinaison convexes formelles\footnote{Le mot
+ « formel » signifie ici que la combinaison n'a pas d'autre sens que
+ comme l'expression par laquelle elle est définie, par exemple
+ $\frac{1}{3}\text{Pierre} + \frac{1}{3}\text{Papier} +
+ \frac{1}{3}\text{Ciseaux}$ est une combinaison convexe formelle de
+ $\{\text{Pierre}, \text{Papier}, \text{Ciseaux}\}$ représentant la
+ probabilité uniforme sur cet ensemble.} des éléments de $B$.
+
+On passera indifféremment entre ces trois points de vue sur les
+stratégies mixtes : fonctions $s\colon B\to\mathbb{R}$ (positives de
+somme $1$), mesures de probabilités sur $B$, ou bien combinaisons
+convexes formelles d'éléments de $B$.
En tout état de cause, l'ensemble $S_B$ des stratégies mixtes sur $B$
-sera vu (notamment comme espace topologique) comme le fermé de
-$\mathbb{R}^B$ défini par l'intersection des demi-espaces de
-coordonnées positives et de l'hyperplan défini par la somme des
-coordonnées égale à $1$.
-\end{defn}
+sera vu comme la partie de $\mathbb{R}^B$ définie par l'intersection
+des demi-espaces de coordonnées positives et de l'hyperplan défini par
+la somme des coordonnées égale à $1$ : il s'agit d'un \emph{convexe
+ fermé}, qui est l'enveloppe convexe des points de $B$ identifiés
+ainsi qu'on vient de le dire aux $\delta_b =
+(0,\ldots,0,1,0,\ldots,0)$, c'est-à-dire que $S_B$ est le
+\defin{simplexe} d'ensemble de sommets $B$ (identifié à l'ensemble
+des $\delta_b$).
\begin{defn}\label{definition-mixed-strategy-game}
Pour un jeu comme défini en \ref{definition-game-in-normal-form}, une
stratégie mixte pour le joueur $i$ est donc une fonction $s\colon A_i
-\to\mathbb{R}$ comme on vient de le dire. On notera parfois $S_i$
+\to\mathbb{R}$ comme on vient de le dire
+en \ref{definition-mixed-strategy-abst}. On notera parfois $S_i$
l'ensemble des stratégies mixtes du joueur $i$. Un \defin{profil de
stratégies mixtes} est un élément du produit cartésien $S := S_1
\times \cdots \times S_N$.
@@ -1131,55 +1230,86 @@ parler de profil de stratégies pures.
\end{defn}
\thingy\label{remark-mixed-stragy-profile-versus-correlated-profile}
-Il va de soi qu'un profil de stratégies mixtes, i.e., un
-élément de $S := S_1 \times \cdots \times S_N$, i.e., la donnée d'une
-distribution de probabilité sur chaque $A_i$, n'est pas la même chose
-qu'une distribution de probabilités sur $A := A_1 \times \cdots \times
-A_N$. Néanmoins, on peut voir les profils de stratégies mixtes comme
-des distributions particulières sur $A$, à savoir celles pour
-lesquelles les marginales (i.e., les projections sur un des $A_i$)
-sont indépendantes. Concrètement, ceci signifie que donné
-$(s_1,\ldots,s_N) \in S$, on en déduit un $s\colon A\to\mathbb{R}$,
-aussi une distribution de probabilité, par la définition suivante :
-$s(a_1,\ldots,a_N) = s_1(a_1)\cdots s_N(a_N)$ (produit des
-$s_i(a_i)$). On identifiera parfois abusivement l'élément
-$(s_1,\ldots,s_N) \in S$ à la distribution $s\colon A\to\mathbb{R}$
-qu'on vient de décrire (ce n'est pas un problème car $s_i$ se déduit
-de $s$ : précisément, $s_i(b) = \sum_{a\,:\, a_i = b} s(a)$ où la somme
-est prise sur les $a \in A$ tels que $a_i = b$).
-
-\danger (Il faudra prendre garde au fait qu'on peut voir $S$ soit
-comme une partie $S_1\times\cdots\times S_N$ de
+Il va de soi qu'un profil de stratégies mixtes, i.e., un élément de $S
+:= S_1 \times \cdots \times S_N$, i.e., la donnée d'une distribution
+de probabilité sur chaque $A_i$, \emph{n'est pas la même chose} qu'une
+distribution de probabilités sur $A := A_1 \times \cdots \times A_N$.
+
+Néanmoins, si $(s_1,\ldots,s_N) \in S$, on définit une distribution de
+probabilités $s\colon A\to\mathbb{R}$ (un élément de $S_A$, si on
+veut) de la façon suivante :
+\[
+s(a_1,\ldots,a_N) = s_1(a_1)\cdots s_N(a_N)
+\tag{$*$}
+\]
+(produit des $s_i(a_i)$).
+
+Probabilistiquement, cette formule revient à dire qu'on tire un
+élément $a_i$ de $A_i$ selon la distribution $s_i$ \emph{de façon
+ indépendante} pour chaque $i$ de manière à former un $N$-uplet
+$(a_1,\ldots,a_N)$. La formule ($*$) servira donc à représenter
+l'hypothèse que les joueurs ont accès à des sources de hasard qui sont
+indépendantes (si Alice tire son coup à pile ou face et que Bob fait
+de même, les pièces tirées par Alice et Bob sont des variables
+aléatoires indépendantes). Les distributions de probabilités $s$
+sur $A$ définies par la formule ($*$) sont précisément celles dont
+composantes sont indépendantes, et qui sont alors complètement
+déterminées par leurs \emph{distributions
+ marginales}\footnote{\label{footnote-marginals}La $i$-ième
+ \defin{marginale} d'une variable aléatoire sur $A_1\times \cdots
+ \times A_N$ est simplement sa $i$-ième composante (= projection
+ sur $A_i$). La $i$-ième \textbf{distribution marginale} de la
+ distribution de probabilités $s \colon A \to \mathbb{R}$ est donc la
+ distribution de probabilités $s_i \colon A_i \to \mathbb{R}$ qui à
+ $b\in A_i$ associe la somme des $s(a_1,\ldots,a_N)$ prise sur tous
+ les $N$-uplets $(a_1,\ldots,a_N)$ tels que $a_i =
+ b$.} $s_1,\ldots,s_N$.
+
+On identifiera parfois abusivement l'élément $(s_1,\ldots,s_N) \in S$
+à la distribution $s\colon A\to\mathbb{R}$ qu'on vient de décrire (ce
+n'est pas un problème car $s_i$ se déduit de $s$ : précisément,
+$s_i(b) = \sum_{a\,:\, a_i = b} s(a)$ où la somme est prise sur les $a
+\in A$ tels que $a_i = b$, cf. note \ref{footnote-marginals}).
+
+\danger Il faudra prendre garde au fait que, du coup, $S$ peut se voir
+\emph{soit} comme la partie $S_1\times\cdots\times S_N$ de
$\mathbb{R}^{A_1}\times\cdots\times \mathbb{R}^{A_N}$ formé des
-$N$-uplets $(s_1,\ldots,s_N)$, soit comme la partie de $\mathbb{R}^A =
-\mathbb{R}^{A_1\times\cdots\times A_N}$ formé des fonctions de la
-forme $s\colon (a_1,\ldots,a_N) = s_1(a_1)\cdots s_N(a_N)$ comme on
-l'a expliqué au paragraphe précédent. Ces deux points de vue ont un
-sens et ont parfois un intérêt, mais ils ne partagent pas les mêmes
-propriétés. Par exemple, $S$ est convexe en tant que partie
-$S_1\times\cdots\times S_N$ de $\mathbb{R}^{A_1}\times\cdots\times
-\mathbb{R}^{A_N}$, mais pas en tant que partie de $\mathbb{R}^A$.)
+$N$-uplets $(s_1,\ldots,s_N)$, \emph{soit} comme la partie de
+$\mathbb{R}^A = \mathbb{R}^{A_1\times\cdots\times A_N}$ formé des
+fonctions de la forme $s\colon (a_1,\ldots,a_N) \mapsto s_1(a_1)\cdots
+s_N(a_N)$ comme on l'a expliqué aux paragraphes précédents. Ces deux
+points de vue ont un sens et ont parfois un intérêt, mais ils ne
+partagent pas les mêmes propriétés. Par exemple, $S$ est convexe en
+tant que partie $S_1\times\cdots\times S_N$ de
+$\mathbb{R}^{A_1}\times\cdots\times \mathbb{R}^{A_N}$, mais pas en
+tant que partie de $\mathbb{R}^A$. Néanmoins, ce double point de vue
+ne devrait pas causer de confusion.
Ceci conduit à faire la définition suivante :
-\begin{defn}
+\begin{defn}\label{definition-mixed-strategy-gain}
Donné un jeu en forme normale comme
en \ref{definition-game-in-normal-form}, si $s := (s_1,\ldots,s_N) \in
S_1 \times \cdots \times S_N$ est un profil de stratégies mixtes, on
-appelle \defin[gain espéré]{gain [espéré]} du joueur $i$ selon ce profil la
-quantité
+appelle \defin[gain espéré]{gain [espéré]} du joueur $i$ selon ce
+profil la quantité
\[
u_i(s) := \sum_{a\in A} s_1(a_1)\cdots s_N(a_N)\,u_i(a)
\]
-(ceci définit $u_i$ comme fonction de $S_1\times\cdots \times S_N$
-vers $\mathbb{R}$).
+c'est-à-dire selon la distribution de probabilités définie par la
+formule ($*$)
+de \ref{remark-mixed-stragy-profile-versus-correlated-profile}. Ceci
+étend $u_i$ en une function de $S := S_1\times\cdots \times S_N$
+vers $\mathbb{R}$, qu'on dénotera par le même symbole car il n'en
+résultera pas de confusion.
\end{defn}
Selon l'approche qu'on veut avoir, on peut dire qu'on a défini
-$u_i(s)$ comme l'espérance de $u_i(a)$ si chaque $a_j$ est tiré selon
-la distribution de probabilité $s_i$ ; ou bien qu'on a utilisé
-l'unique prolongement de $u_i$ au produit des simplexes $S_i$ qui soit
-affine en chaque variable $s_i$.
+$u_i(s)$ comme l'espérance de $u_i(a)$ si chaque $a_j$ est tiré
+indépendamment selon la distribution de probabilité $s_i$ ; ou bien
+qu'on a utilisé l'unique prolongement de $u_i$ au produit des
+simplexes $S_i$ qui soit affine en chaque variable $s_i$
+(« multi-affine »).
@@ -1188,26 +1318,37 @@ affine en chaque variable $s_i$.
\begin{defn}\label{definition-best-response-and-nash-equilibrium}
Donné un jeu en forme normale comme
en \ref{definition-game-in-normal-form}, si $1 \leq i \leq N$ et si
-$s_? := (s_1,\ldots,s_{i-1},s_{i+1},\ldots,s_N) \in S_1 \times \cdots
-\times S_{i-1} \times S_{i+1} \times \cdots \times S_N$ est un profil
-de stratégies mixtes pour tous les joueurs autres que le joueur $i$,
-on dit que la stratégie mixte $s_! \in S_i$ est une \defin{meilleure
- réponse} (resp. la meilleure réponse \textbf{stricte}) contre $s_?$
-lorsque pour tout $t \in S_i$ on a $u_i(s_?,s_!) \geq u_i(s_?,t)$
-(resp. lorsque pour tout $t \in S_i$ différent de $s_!$ on a
-$u_i(s_?,s_!) > u_i(s_?,t)$), où $(s_?,t)$ désigne l'élément de
-$S_1\times \cdots \times S_N$ obtenu en insérant $t \in S_i$ comme
-$i$-ième composante entre $s_{i-1}$ et $s_{i+1}$, c'est-à-dire le gain
-[espéré] obtenu en jouant $t$ contre $s_?$.
+$s_? := (s_1,\ldots,s_{i-1},s_{i+1},\ldots,s_N) \in S_{?i} := S_1
+\times \cdots \times S_{i-1} \times S_{i+1} \times \cdots \times S_N$
+est un profil de stratégies mixtes pour tous les joueurs autres que le
+joueur $i$, on dit que la stratégie mixte $s_! \in S_i$ est une
+\defin{meilleure réponse} (resp. la meilleure réponse
+\textbf{stricte}) contre $s_?$ lorsque pour tout $t \in S_i$ on a
+$u_i(s_?,s_!) \geq u_i(s_?,t)$ (resp. lorsque pour tout $t \in S_i$
+différent de $s_!$ on a $u_i(s_?,s_!) > u_i(s_?,t)$), où $(s_?,t)$
+désigne l'élément de $S_1\times \cdots \times S_N$ obtenu en insérant
+$t \in S_i$ comme $i$-ième composante entre $s_{i-1}$ et $s_{i+1}$,
+c'est-à-dire le gain [espéré] obtenu en jouant $t$ contre $s_?$.
Un profil de stratégies mixtes $s = (s_1,\ldots,s_N)$ (pour l'ensemble
-des joueurs) est dit être un \index{Nash (équilibre de)}\defin{équilibre de Nash} (resp., un
-équilibre de Nash \defin[strict (équilibre de Nash)]{strict}) lorsque pour tout $1\leq i \leq N$,
-la stratégie $s_i$ pour le joueur $i$ est une meilleure réponse
-(resp. la meilleure réponse stricte) contre le profil $s_{?i}$ pour
-les autres joueurs obtenu en supprimant la composante $s_i$ de $s$.
+des joueurs) est dit être un \index{Nash (équilibre
+ de)}\defin{équilibre de Nash} (resp., un équilibre de Nash
+\defin[strict (équilibre de Nash)]{strict}) lorsque pour tout $1\leq i
+\leq N$, la stratégie $s_i$ pour le joueur $i$ est une meilleure
+réponse (resp. la meilleure réponse stricte) contre le profil $s_{?i}
+:= (s_1,\ldots,s_{i-1},s_{i+1},\ldots,s_N)$ pour les autres joueurs
+obtenu en supprimant la composante $s_i$ de $s$.
\end{defn}
+Concrètement, donc, un équilibre de Nash est donc un profil de
+stratégies mixtes de l'ensemble des joueurs dans lequel \emph{aucun
+ joueur n'a intérêt à changer unilatéralement sa stratégie} (au sens
+où faire un tel changement lui apporterait une espérance de gain
+strictement supérieure). Un équilibre de Nash \emph{strict}
+correspond à la situation où tout changement unilatéral de stratégie
+d'un joueur lui apporterait une espérance de gain strictement
+inférieure.
+
\begin{prop}\label{stupid-remark-best-mixed-strategies}
Donné un jeu en forme normale comme
en \ref{definition-game-in-normal-form}, si $1 \leq i \leq N$ et si
@@ -1223,28 +1364,58 @@ En particulier, une meilleure réponse stricte est nécessairement une
stratégie pure.
\end{prop}
\begin{proof}
-Tout ceci résulte du fait que le gain espéré $u_i(s_?,t)$ est une
-fonction affine de $t \in S_i$ (et une fonction affine sur un simplexe
-prend son maximum — ou son minimum — sur un des sommets de ce
-simplexe, et ne peut le prendre à l'intérieur que si elle prend aussi
-cette valeur sur les sommets).
-
-Plus précisément : $u_i(s_?,t)$ (pour $t \in S_i$) est combinaison
-convexe avec pour coefficients $t(a)$ pour $a\in A_i$, des
-$u_i(s_?,a)$. Si $v$ est le maximum des $u_i(s_?,a)$ (qui sont en
-nombre fini donc ce maximum existe), alors $v$ est aussi le maximum de
-toute combinaison convexe $u_i(s_?,t)$ des $u_i(s_?,a)$ : c'est-à-dire
-que $t\in S_i$ est une meilleure réponse possible contre $s_?$ si et
-seulement si $u_i(s_?,t) = v$. En particulier, tout $a \in A_i$ qui
-réalise ce maximum $v$ est une meilleure réponse possible
-(contre $s_?$) qui est une stratégie pure. D'autre part, une
-combinaison convexe $u_i(s_?,t)$ de valeurs $\leq v$ ne peut être
-égale à $v$ que si toutes les valeurs $u_i(s_?,a)$ entrant
-effectivement (c'est-à-dire avec un coefficient $>0$) dans la
-combinaison sont égales à $v$ (s'il y en avait une $<v$, elle
-entraînerait forcément une moyenne pondérée $<v$), et réciproquement.
+Tout ceci résulte essentiellement des propositions
+\ref{affine-functions-take-extrema-at-boundary} et \ref{affine-functions-take-no-strict-extrema-inside} :
+le gain espéré $u_i(s_?,t)$ du joueur $i$ (une fois fixé le
+profil $s_?$ de tous les autres joueurs) est une fonction affine de la
+stratégie $t \in S_i$ du joueur $i$, et une fonction affine sur un
+simplexe prend son maximum sur un sommet du simplexe, et ne peut la
+prendre à l'intérieur d'une face (= enveloppe convexe d'un
+sous-ensemble des sommets) que si elle la prend également en chaque
+sommet de cette face.
+
+Plus précisément, soit $v = \max_{t \in S_i} u_i(s_?,t)$, dont la
+proposition \ref{affine-functions-take-extrema-at-boundary} garantit
+l'existence ; une meilleure réponse possible contre $s_?$ est
+précisément un $t$ réalisant $u_i(s_?,t) = v$ : il résulte de la
+proposition citée que $v$ est aussi $\max_{a \in A_i} u_i(s_?,a)$,
+c'est-à-dire qu'il existe une stratégie \emph{pure} $a \in A_i$ qui
+est une meilleure réponse possible contre $s_?$. Par ailleurs, si
+$s_!$ est une meilleure réponse, i.e., $u_i(s_?,s_!) = v$, alors la
+proposition \ref{affine-functions-take-no-strict-extrema-inside}
+(appliquée avec pour $x_1,\ldots,x_n$ les points du support de $s_!$)
+montre que chaque stratégie pure $a$ appartenant au support de $s_!$
+vérifie aussi $u_i(s_?,a) = v$.
+
+Comme une meilleure réponse stricte est unique, elle est forcément
+pure d'après ce qu'on vient de voir.
\end{proof}
+\textbf{Reformulation.} Un profil de stratégies mixtes
+$(s_1,\ldots,s_N) \in S_1\times \cdots\times S_N$ est un équilibre de
+Nash si et seulement si, pour chaque $i$,
+\begin{itemize}
+\item pour tout $a$ dans le support de $s_i$, le gain espéré
+ $u_i(s_{?i},a)$ (rappelons que ceci est une notation pour
+ $u_i(s_1,\ldots,s_{i-1},a,s_{i+1},\ldots,s_N$) prend la \emph{même}
+ valeur, et
+\item cette valeur est supérieure ou égale à $u_i(s_{?i},b)$ pour
+ tout $b\in A_i$.
+\end{itemize}
+
+\smallskip
+
+On vient de voir que \emph{lorsque les stratégies de tous les autres
+ joueurs sont fixées} (à $s_?$ dans la proposition ci-dessus), le
+joueur $i$ a une meilleure réponse en stratégie pure : on pourrait
+être tenté d'en conclure, mais ce serait une erreur, que les
+stratégies mixtes n'apportent donc rien à l'histoire, et qu'un
+équilibre de Nash existe forcément en stratégies pures : \emph{ce
+ n'est pas le cas}. Néanmoins, les équilibres de Nash existent bien
+en stratégies mixtes, et c'est le résultat central du domaine (ayant
+essentiellement valu à son auteur le prix d'économie de la banque de
+Suède à la mémoire d'Alfred Nobel en 1994) :
+
\begin{thm}[John Nash, 1951]\label{theorem-nash-equilibria}
Pour un jeu en forme normale comme
en \ref{definition-game-in-normal-form}, il existe un équilibre de
@@ -1265,7 +1436,7 @@ profil $s$ de stratégies, on peut définir continûment un nouveau
profil $s^\sharp$ en donnant plus de poids aux options qui donnent un
meilleur gain au joueur correspondant — si bien que $s^\sharp$ sera
différent de $s$ dès que $s^\sharp$ n'est pas un équilibre de Nash. Comme
-la fonction $T \colon s \to s^\sharp$ doit avoir un point fixe, ce point
+la fonction $T \colon s \mapsto s^\sharp$ doit avoir un point fixe, ce point
fixe sera un équilibre de Nash.
\begin{proof}[Démonstration de \ref{theorem-nash-equilibria}]
@@ -1304,9 +1475,10 @@ mais elle peut donner l'impression qu'on commet une « erreur
D'après la première expression donnée, il est clair qu'on a bien
$s^\sharp_i \in S_i$, et qu'on a donc bien défini une fonction
-$T\colon S\to S$. Cette fonction est continue, donc admet un point
-fixe $s$ d'après \ref{brouwer-fixed-point-theorem} (notons qu'ici, $S$
-est vu comme le convexe $S_1\times\cdots\times S_N$ dans
+$T\colon S\to S$. Cette fonction est continue (comme composée de
+fonctions continues), donc admet un point fixe $s$
+d'après \ref{brouwer-fixed-point-theorem} (notons qu'ici, $S$ est vu
+comme le convexe $S_1\times\cdots\times S_N$ dans
$\mathbb{R}^{A_1}\times\cdots\times \mathbb{R}^{A_N}$). On va montrer
que $s$ est un équilibre de Nash.
@@ -1340,17 +1512,18 @@ Pour $N=2$, une méthode qui peut fonctionner dans des cas suffisamment
petits consiste à énumérer tous les supports
(cf. \ref{definition-mixed-strategy-abst}) possibles des stratégies
mixtes des joueurs dans un équilibre de Nash, c'est-à-dire toutes les
-$(2^{\#A_1}-1)\times(2^{\#A_2}-1)$ données de parties non vides de
-$A_1$ et $A_2$, et, pour chacune, appliquer le raisonnement suivant :
-si $s_i$ est une meilleure réponse possible pour le joueur $i$ (contre
-la stratégie $s_{?i}$ de l'autre joueur) alors \emph{toutes les
- options du support de $s_i$ ont la même espérance de gain}
-(contre $s_{?i}$ ; cf. \ref{stupid-remark-best-mixed-strategies}), ce
-qui fournit un jeu d'égalités linéaires sur les valeurs de $s_{?i}$.
-En rassemblant ces inégalités (ainsi que celles qui affirment que la
-somme des valeurs de $s_i$ et de $s_{?i}$ valent $1$), on arrive
-« normalement » à trouver tous les équilibres de Nash possibles : voir
-les exercices
+$(2^{\#A_1}-1)\penalty0\times\penalty0(2^{\#A_2}-1)$ données de
+parties non vides de $A_1$ et $A_2$, et, pour chacune, appliquer le
+raisonnement suivant : si $s_i$ est une meilleure réponse possible
+pour le joueur $i$ (contre la stratégie $s_{?i}$ de l'autre joueur)
+alors \emph{toutes les options du support de $s_i$ ont la même
+ espérance de gain} (contre $s_{?i}$ ;
+cf. \ref{stupid-remark-best-mixed-strategies}), ce qui fournit un jeu
+d'égalités linéaires sur les valeurs de $s_{?i}$. En rassemblant ces
+inégalités (ainsi que celles qui affirment que la somme des valeurs de
+$s_i$ et de $s_{?i}$ valent $1$), on arrive « normalement » à trouver
+tous les équilibres de Nash possibles : voir \ref{dove-or-hawk}
+ci-dessous, ainsi que les exercices
\ref{normal-form-game-exercise-two-by-two} et \ref{normal-form-game-exercise-three-by-three}
(dernières questions) pour des exemples.
@@ -1360,6 +1533,67 @@ algorithmiquement possible en théorie en vertu d'un théorème de Tarski
et Seidenberg sur la décidabilité des systèmes d'équations algébriques
réels, mais possiblement inextricable dans la pratique.)
+\thingy\label{dove-or-hawk-redux} Pour donner un exemple, revenons sur
+le \index{trouillard (jeu du)}jeu du trouillard défini
+en \ref{dove-or-hawk} et dressons un tableau de l'espérance de gain
+pour quelques stratégies mixtes ainsi que la stratégie mixte
+« générique » :
+
+{\footnotesize
+\begin{center}
+\begin{tabular}{r|c|c|c|c|c|}
+$\downarrow$Alice, Bob$\rightarrow$&Colombe&Faucon&$\frac{2}{3}C+\frac{1}{3}F$&$\frac{1}{2}C+\frac{1}{2}F$&$qC+(1-q)F$\\\hline
+Colombe&$2,2$&\textcolor{blue}{$0,4$}&$\frac{4}{3},\frac{8}{3}$&$1,3$&{\tiny $2q, 4-2q$}\\\hline
+Faucon&\textcolor{blue}{$4,0$}&$-4,-4$&$\frac{4}{3},-\frac{4}{3}$&$0,-2$&{\tiny $-4+8q, -4+4q$}\\\hline
+$\frac{2}{3}C+\frac{1}{3}F$&$\frac{8}{3},\frac{4}{3}$&$-\frac{4}{3},\frac{4}{3}$&\textcolor{blue}{$\frac{4}{3},\frac{4}{3}$}&$\frac{2}{3},\frac{4}{3}$&{\tiny $-\frac{4}{3}+4q, \frac{4}{3}$}\\\hline
+$\frac{1}{2}C+\frac{1}{2}F$&$3,1$&$-2,0$&$\frac{4}{3},\frac{2}{3}$&$\frac{1}{2},\frac{1}{2}$&$-2+5q,q$\\\hline
+$pC+(1-p)F$&{\tiny $4-2p,2p$}&{\tiny $-4+2p,-4+8p$}&{\tiny $\frac{4}{3},-\frac{4}{3}+4p$}&{\tiny $p,-2+5p$}&$(\dagger)$\\\hline
+\end{tabular}
+\end{center}
+\par\noindent
+où $(\dagger) = (-4 + 4p + 8q - 6pq, - 4 + 8p + 4q - 6pq)$.
+\par}
+
+\smallskip
+
+Les trois profils $(C,F)$ (i.e., Alice joue Colombe et Bob joue
+Faucon), $(F,C)$ et $(\frac{2}{3}C+\frac{1}{3}F,
+\frac{2}{3}C+\frac{1}{3}F)$ sont des équilibres de Nash : cela résulte
+du fait que, quand on regarde la case correspondante du tableau, le
+premier nombre est maximum sur la colonne (c'est-à-dire qu'Alice n'a
+pas intérêt à changer unilatéralement sa stratégie) et le second est
+maximum sur la ligne (c'est-à-dire que Bob n'a pas intérêt à changer
+unilatéralement sa stratégie) : chacun de ces maxima peut se tester
+simplement contre les stratégies pures de l'adversaire (i.e., les deux
+premières lignes ou colonnes). Pour trouver ces équilibres et
+vérifier qu'il n'y en a pas d'autre, on peut :
+\begin{itemize}
+\item commencer par rechercher les équilibres de Nash où chacun des
+ deux joueurs joue une stratégie pure (ceci revient à regarder
+ chacune des quatre cases du tableau initial et chercher si le
+ premier nombre est maximal sur la colonne et le second maximal sur
+ la ligne) ;
+\item considérer la possibilité d'un équilibre de Nash où l'un des
+ joueurs joue une stratégie pure et l'autre une stratégie mixte
+ supportée par les deux options : si par exemple Alice joue
+ $pC+(1-p)F$ avec $p>0$ et Bob joue $a \in \{C,F\}$,
+ d'après \ref{stupid-remark-best-mixed-strategies}, le gain d'Alice
+ dans les cases $(C,a)$ et $(F,a)$ doit être le même, ce qui n'est
+ pas le cas, donc il n'y a pas de tel équilibre (pas plus que dans le
+ cas correspondant pour Bob) ;
+\item enfin, rechercher les équilibres de Nash où chacun des deux
+ joueurs joue une stratégie supportée par les deux options, soit ici
+ $pC+(1-p)F$ pour Alice et $qC+(1-q)F$ pour Bob avec $0<p<1$ et
+ $0<q<1$ : d'après \ref{stupid-remark-best-mixed-strategies} le gain
+ espéré d'Alice doit être le même contre $qC+(1-q)F$ indifféremment
+ qu'elle joue Colombe ou Faucon, ce qui implique $2q = -4+8q$ donc
+ $q=\frac{2}{3}$, et symétriquement, le gain espéré de Bob doit être
+ le même contre $pC+(1-p)F$ indifféremment qu'il joue Colombe ou
+ Faucon, ce qui implique $2p = -4+8p$ donc $p=\frac{2}{3}$, si bien
+ que le seul équilibre de Nash de cette nature est celui qu'on a
+ décrit (et il faut vérifier qu'il en est bien un).
+\end{itemize}
+
\thingy Mentionnons en complément une notion plus générale que celle
d'équilibre de Nash : si $s\colon A \to \mathbb{R}$ (où $A :=
A_1\times\cdots\times A_N$) est cette fois une distribution de
@@ -1418,179 +1652,270 @@ dire que $(s_1,\ldots,s_N)$ est un équilibre de Nash. Autrement dit :
\subsection{Jeux à somme nulle : le théorème du minimax}\label{zero-sum-games}
+Plaçons nous maintenant dans le cadre d'un jeu à somme nulle,
+c'est-à-dire qu'il y a $N=2$ joueurs, et qu'ils ont des gains
+opposés : disons qu'on appelle $u = u_1$ le gain du joueur $1$
+(« Alice »), le gain du joueur $2$ (« Bob ») étant alors $u_2 = -u$ et
+n'a pas besoin d'être écrit dans le tableau. Ainsi, Alice cherche à
+\emph{maximiser} $u$ et Bob cherche à le \emph{minimiser} (puisque
+maximiser $-u$ revient à minimiser $u$).
+
+En considérant le jeu du point de vue de sa matrice de gains (où, de
+nouveau, on n'a écrit que le gain d'Alice), Alice cherche à choisir
+une ligne qui maximiser la valeur écrite dans le tableau et Bob
+cherche à choisir une colonne qui mimimise la valeur écrite. Mais en
+général, si $u\colon A_1 \times A_2 \to \mathbb{R}$ est un tableau
+fini de nombres réels, $\max_{a \in A_1} \min_{b\in A_2} u(a,b)$ ne
+coïncide pas avec $\min_{b\in A_2} \max_{a \in A_1} u(a,b)$ (on a
+seulement l'inégalité $\max_{a \in A_1} \min_{b\in A_2} u(a,b) \leq
+\min_{b\in A_2} \max_{a \in A_1} u(a,b)$, qui sera démontrée ci-dessus
+au début de la démonstration de \ref{theorem-minimax}). L'observation
+cruciale est que, quand on passe des ensembles finis $A_i$ de
+stratégies pures aux simplexes $S_i$ de stratégies mixtes, on a alors
+une égalité :
+
\begin{thm}[« du minimax », J. von Neumann, 1928]\label{theorem-minimax}
-Soient $C$ et $C'$ deux convexes compacts dans des espaces affines
-réels de dimension finie, et $u\colon C\times C' \to \mathbb{R}$
-une application bi-affine (c'est-à-dire, affine en chaque variable
-séparément). Alors
+Soient $A_1,A_2$ deux ensembles finis et $S_1,S_2$ les simplexes de
+stratégies mixtes (cf. \ref{definition-mixed-strategy-abst})
+sur $A_1,A_2$ ; soit $u\colon A_1\times A_2 \to \mathbb{R}$ une
+fonction quelconque, et on notera encore $u$ son unique extension
+(explicitée en \ref{definition-mixed-strategy-gain}) en une fonction
+$S_1\times S_2 \to \mathbb{R}$ affine en chaque variable. On a
+alors :
\[
-\max_{x\in C} \min_{y\in C'} u(x,y) =
-\min_{y\in C'} \max_{x\in C} u(x,y)
+\max_{x\in S_1} \min_{y\in S_2} u(x,y) =
+\min_{y\in S_2} \max_{x\in S_1} u(x,y)
\]
+(Ceci sous-entend notamment ces $\min$ et $\max$ existent bien.)
+
+Plus généralement, si $S_1$ et $S_2$ sont deux convexes compacts dans
+des espaces affines réels de dimension finie, et $u\colon S_1\times
+S_2 \to \mathbb{R}$ une application affine en chaque variable
+séparément, alors on a l'égalité ci-dessus.
\end{thm}
\begin{proof}
-Tout d'abord, l'inégalité dans un sens est évidente : on a
+Plaçons-nous d'abord dans le cas (qui intéresse la théorie des jeux)
+où $A_1,A_2$ sont deux ensembles finis et $S_1,S_2$ les simplexes de
+stratégies mixtes.
+
+Tout d'abord, montrons l'existence des $\min$ et $\max$ annoncés ainsi
+que l'inégalité dans le sens $\max\min \leq \min\max$. On a vu
+en \ref{affine-functions-take-extrema-at-boundary} que $\min_{y\in
+ S_2} u(x,y)$ existe et vaut $\min_{b\in A_2} u(x,b)$ pour tout $x\in
+S_1$ : ceci montre que la fonction $x \mapsto \min_{y\in S_2} u(x,y)$
+s'écrit encore $x \mapsto \min_{b\in A_2} u(x,b)$, donc il s'agit du
+minimum d'un nombre \emph{fini} de fonctions continues (affines)
+sur $S_1$, donc elle est encore continue et atteint ses bornes sur
+l'ensemble compact $S_1$ : il existe donc $x_* \in S_1$ où cette
+fonction atteint son maximum. Soit de même $y_* \in S_2$ un point où
+$y \mapsto \max_{x\in S_1} u(x,y)$ atteint son minimum. On a alors :
\[
-\max_{x\in C} \min_{y\in C'} u(x,y)
-= \min_{y\in C'} u(x_*,y)
-\leq u(x_*,y_*) \leq \max_{x\in C} u(x,y_*) =
-\min_{y\in C'} \max_{x\in C} u(x,y)
+\max_{x\in S_1} \min_{y\in S_2} u(x,y)
+= \min_{y\in S_2} u(x_*,y)
+\leq u(x_*,y_*) \leq \max_{x\in S_1} u(x,y_*) =
+\min_{y\in S_2} \max_{x\in S_1} u(x,y)
\]
-où $x_* \in C$ est un point où $\max_{x\in C} \min_{y\in C'}
-u(x,y)$ est atteint et $y_* \in C'$ un point où $\min_{y\in C'}
-\max_{x\in C} u(x,y)$ l'est. Il s'agit donc de prouver
-l'inégalité de sens contraire.
-
-Commençons par supposer que $C$ est l'enveloppe convexe d'un nombre
-fini de points $(x_i)_{i\in I}$ et $C'$ de $(y_j)_{j\in J}$, et on
-expliquera plus loin comment se ramener à ce cas (même si c'est le
-seul qui servira dans le cadre de la théorie des jeux). Lorsque cette
-hypothèse est vérifiée, on va définir une fonction $T\colon C\times C'
-\to C\times C'$ de la façon suivante. Donnons-nous $(x,y) \in C\times
-C'$. Pour chaque $i\in I$, on définit $\varphi_i(x,y) = \max (0,\;
-u(x_i,y)-u(x,y))$, et de même on pose $\psi_j(x,y) = \max (0,\;
-u(x,y)-u(x,y_j))$. Posons enfin $T(x,y) = (x^\sharp,y^\sharp)$ où
-$x^\sharp$ et $y^\sharp$ (qui dépendent tous les deux de $x$ et $y$ à
-la fois, malgré la notation) sont définis comme suit. On appelle
-$x^\sharp$ le barycentre de $x$ affecté du coefficient $1$ et des
-$x_i$ (pour $i\in I$) affectés des coefficients respectifs
-$\varphi_i(x,y)$, c'est-à-dire $x^\sharp = \frac{x + \sum_{i\in I}
- \varphi_i(x,y)\,x_i}{1 + \sum_{i\in I} \varphi_i(x,y)}$ ; et soit de
-même $y^\sharp$ le barycentre de $y$ avec coefficient $1$ et des $y_i$
-avec les coefficients $\psi_i(x,y)$. Clairement, $x^\sharp$ et
-$y^\sharp$ sont dans $C$ et $C'$ respectivement (il s'agit de
-barycentres à coefficients positifs, c'est-à-dire de combinaisons
-convexes). La fonction $T\colon C\times C' \to C\times C'$ définie
-par $T(x,y) = (x^\sharp,y^\sharp)$ est continue. Par ailleurs, on a
-$x^\sharp = x$ si et seulement si $x$ réalise $\max_{\tilde x\in C}
-u(\tilde x,y)$ (un sens est évident, et pour l'autre il suffit de se
-convaincre que s'il existe $\tilde x$ tel que $u(\tilde x,y) > u(x,y)$
-alors il y a un $i$ tel que ceci soit vrai en remplaçant $\tilde x$
-par $x_i$, et on a alors $\varphi_i(x,y)>0$ donc $u(x^\sharp,y) >
-u(x,y)$) ; et on a un résultat analogue pour $y$. La fonction $T$
-continue du compact convexe $C\times C'$ vers lui-même y admet
-d'après \ref{brouwer-fixed-point-theorem} un
-point fixe $(x_0,y_0)$, vérifiant donc $(x_0^\sharp, y_0^\sharp) =
-(x_0,y_0)$, c'est-à-dire que $u (x_0,y_0) = \max_{x\in C} u(x,y_0) =
-\min_{y\in C'} u(x_0, y)$. On a donc maintenant
+La première relation est la définition de $x_*$, la seconde est la
+définition du $\min$, la troisième la définition du $\max$, et la
+quatrième est la définition de $y_*$. (Notons que tout ceci vaut dans
+n'importe quel contexte où les $\min$ et $\max$ existent, notamment
+ceci prouve aussi $\max_{a \in A_1} \min_{b\in A_2} u(a,b) \leq
+\min_{b\in A_2} \max_{a \in A_1} u(a,b)$ affirmé ci-dessus.)
+
+Il s'agit maintenant de prouver l'inégalité de sens contraire.
+
+Considérons $(x_0,y_0)$ un équilibre de Nash du jeu à somme nulle dont
+la matrice de gains est donnée par $u$ pour le joueur $1$ (et $-u$
+pour le joueur $2$) : on sait qu'un tel équilibre existe
+d'après \ref{theorem-nash-equilibria}. On a alors
\[
-\max_{x\in C} \min_{y\in C'} u(x,y)
-\geq \min_{y\in C'} u(x_0,y) = u(x_0,y_0)
-= \max_{x\in C} u(x,y_0) \geq
-\min_{y\in C'} \max_{x\in C} u(x,y)
+\max_{x\in S_1} \min_{y\in S_2} u(x,y)
+\geq \min_{y\in S_2} u(x_0,y) = u(x_0,y_0)
+= \max_{x\in S_1} u(x,y_0) \geq
+\min_{y\in S_2} \max_{x\in S_1} u(x,y)
\]
-ce qu'on voulait.
-
-Pour se ramener au cas où $C$ et $C'$ sont enveloppes convexes d'un
-nombre fini de points, on observe que pour tout $\varepsilon>0$ il
-existe $\Sigma$ et $\Sigma'$ des enveloppes convexes d'un nombre fini
-de points (= polytopes) contenues dans $C$ et $C'$ respectivement et
-telles que pour tout $x\in C$ on ait $\min_{y\in C'} u(x,y) >
-\min_{y\in\Sigma'} u(x,y)-\varepsilon$ et $\max_{x\in C} u(x,y) <
-\max_{x\in\Sigma} u(x,y)+\varepsilon$ (explication : il est trivial
-que pour chaque $x$ il existe un $\Sigma'$ vérifiant la condition
-demandée, le point intéressant est qu'un unique $\Sigma'$ peut
-convenir pour tous les $x$ ; mais pour chaque $\Sigma'$ donné,
-l'ensemble des $x$ pour lesquels il convient est un ouvert de $C$, qui
-est compact, donc un nombre fini de ces ouverts recouvrent $C$, et on
-prend l'enveloppe convexe de la réunion des $\Sigma'$ en question ; on
-procède de même pour $\Sigma$). On a alors $\max_{x\in C} \min_{y\in
- C'} u(x,y) > \max_{x\in \Sigma} \min_{y\in \Sigma'} u(x,y) -
-\varepsilon$ et une inégalité analogue pour l'autre membre : on en
-déduit l'inégalité recherchée à $2\varepsilon$ près, mais comme on
-peut prendre $\varepsilon$ arbitrairement petit, on a ce qu'on
-voulait.
+Ici, la première relation est la définition du $\max$, la seconde
+traduit le fait que (dans la définition de l'équilibre de Nash) Bob
+fait une meilleure réponse possible à $x_0$, la troisième traduit le
+fait qu'Alice fait une meilleure réponse possible à $y_0$, et la
+quatrième est la définition du $\min$.
+
+Ceci achève de montrer $\max_{x\in S_1} \min_{y\in S_2} u(x,y) =
+\min_{y\in S_2} \max_{x\in S_1} u(x,y)$ lorsque $S_1$ et $S_2$ sont
+deux simplexes dans $\mathbb{R}^n$, et $u\colon S_1\times S_2 \to
+\mathbb{R}$ une application affine en chaque variable séparément. Si
+maintenant $S_1$ et $S_2$ sont deux polytopes, c'est-à-dire chacun
+l'enveloppe d'un nombre fini de points $A_1$ et $A_2$
+dans $\mathbb{R}^n$, mais plus forcément des simplexes (c'est-à-dire
+qu'on ne suppose plus les points de $A_i$ affinement indépendants), la
+même conclusion vaut encore : en effet, si on appelle $S'_i$ le
+simplexe construit abstraitement sur $A_i$ (i.e., l'ensemble des
+fonctions $s\colon A_i \to \mathbb{R}$ positives de somme $1$), et
+$\varpi_i \colon S'_i \to S_i$ la fonction (affine et surjective)
+envoyant $s\colon A_i \to \mathbb{R}$ positive de somme $1$ sur la
+somme des $s(a)\cdot a$ (dans le $\mathbb{R}^n$ où vivent $A_i$
+et $S_i$), alors la fonction $u\colon S_1\times S_2 \to \mathbb{R}$
+donnée définit une fonction $u'\colon S'_1\times S'_2 \to \mathbb{R}$,
+elle aussi affine en chaque variable, par $u'(x,y) = u(\varpi_1(x),
+\varpi_2(y))$, et il est évident que les extrema de $u'$ sur $S'_1
+\times S'_2$ coïncident avec ceux de $u$ sur $S_1 \times S_2$, donc la
+conclusion qui précède, appliquée à $u'$, donne la conclusion désirée
+sur $u$.
+
+Enfin, si $S_1$ et $S_2$ sont seulement supposés être des convexes
+compacts dans $\mathbb{R}^n$, on observe que pour tout $\varepsilon>0$
+il existe $\Sigma_1$ et $\Sigma_2$ des polytopes contenus dans $S_1$
+et $S_2$ respectivement et tels que pour tout $x\in S_1$ on ait
+$\min_{y\in S_2} u(x,y) > \min_{y\in\Sigma_2} u(x,y)-\varepsilon$ et
+$\max_{x\in S_1} u(x,y) < \max_{x\in\Sigma_1} u(x,y)+\varepsilon$
+(explication : il est trivial que pour chaque $x$ il existe un
+$\Sigma_2$ vérifiant la condition demandée, le point intéressant est
+qu'un unique $\Sigma_2$ peut convenir pour tous les $x$ ; mais pour
+chaque $\Sigma_2$ donné, l'ensemble des $x$ pour lesquels il convient
+est un ouvert de $S_1$, qui est compact, donc un nombre fini de ces
+ouverts recouvrent $S_1$, et on prend l'enveloppe convexe de la
+réunion des $\Sigma_2$ en question ; on procède de même pour
+$\Sigma_1$). On a alors $\max_{x\in S_1} \min_{y\in S_2} u(x,y) >
+\max_{x\in \Sigma_1} \min_{y\in \Sigma_2} u(x,y) - \varepsilon$ et une
+inégalité analogue pour l'autre membre : on en déduit l'inégalité
+recherchée à $2\varepsilon$ près, mais comme on peut prendre
+$\varepsilon$ arbitrairement petit, on a ce qu'on voulait.
\end{proof}
+Le corollaire suivant explicite la situation particulière où, en
+outre, le jeu est symétrique entre les deux joueurs (c'est-à-dire que
+sa matrice de gains écrite pour un seul joueur est, elle,
+\emph{anti}symétrique) :
+
\begin{cor}\label{symmetric-zero-sum-game}
-Soit $C$ un convexe compact dans un espace affine réel de dimension
-finie, et $u\colon C^2 \to \mathbb{R}$ une application bi-affine
-antisymétrique (i.e., $u(y,x) = -u(x,y)$). Alors il
-existe $x\in C$ tel que pour tout $y\in C$ on ait $u(x,y)\geq 0$
-(et la valeur commune des deux membres de l'égalité du
-théorème \ref{theorem-minimax} est $0$).
+Soient $A$ un ensemble fini et $S$ le simplexe de stratégies mixtes
+sur $A$ ; soit $u\colon A^2 \to \mathbb{R}$ une application
+antisymétrique (i.e., $u(b,a) = -u(a,b)$), et on notera encore $u$ son
+unique extension affine en chaque variable en une fonction $S^2 \to
+\mathbb{R}$ affine en chaque variable. Alors la valeur commune des
+deux membres de l'égalité du théorème \ref{theorem-minimax} est $0$ :
+il existe $x\in S$ tel que pour tout $y\in S$ on ait $u(x,y)\geq 0$.
+
+Plus généralement, si $S$ est un convexe compact dans un espace affine
+réel de dimension finie, et $u\colon S^2 \to \mathbb{R}$ une
+application antisymétrique (i.e., $u(y,x) = -u(x,y)$) et affine en
+chaque variable séparément, la même conclusion vaut.
\end{cor}
\begin{proof}
-On applique le théorème : il donne $\max_{x\in C}\penalty0 \min_{y\in
- C} u(x,y) = \min_{y\in C}\penalty0 \max_{x\in C} u(x,y)$. Mais
-puisque $u$ est antisymétrique ceci s'écrit encore $\min_{y\in C}
-\max_{x\in C} (-u(y,x))$, soit, en renommant les variables liées,
-$\min_{x\in C}\penalty0 \max_{y\in C} (-u(x,y)) = -\max_{x\in
- C}\penalty0 \min_{y\in C} u(x,y)$. Par conséquent, $\max_{x\in
- C}\penalty0 \min_{y\in C} u(x,y) = 0$ (il est son propre opposé), et
-en prenant un $x$ qui réalise ce maximum, on a $\min_{y\in C} u(x,y) =
+On applique le théorème : il donne $\max_{x\in S}\penalty0 \min_{y\in
+ S} u(x,y) = \min_{y\in S}\penalty0 \max_{x\in S} u(x,y)$. Mais
+puisque $u$ est antisymétrique ceci s'écrit encore $\min_{y\in S}
+\max_{x\in S} (-u(y,x))$, soit, en renommant les variables liées,
+$\min_{x\in S}\penalty0 \max_{y\in S} (-u(x,y)) = -\max_{x\in
+ S}\penalty0 \min_{y\in S} u(x,y)$. Par conséquent, $\max_{x\in
+ S}\penalty0 \min_{y\in S} u(x,y) = 0$ (il est son propre opposé), et
+en prenant un $x$ qui réalise ce maximum, on a $\min_{y\in S} u(x,y) =
0$, ce qu'on voulait prouver.
\end{proof}
-\thingy\label{minimax-for-games} Le théorème \ref{theorem-minimax}
-s'applique à la théorie des jeux de la manière suivante : si on
-considère un jeu à deux joueurs à somme nulle, en notant $S_1$ et
-$S_2$ les ensembles des stratégies mixtes des deux joueurs, et $u
-\colon S_1 \times S_2 \to \mathbb{R}$ le gain espéré du joueur $1$, le
-gain du joueur $2$ étant donc $-u$, le fait que $(x_0,y_0)$ soit un
-équilibre de Nash se traduit par le fait que $x_0$ soit la meilleure
-réponse possible de $1$ contre $y_0$, i.e., $u(x_0,y_0) = \max_{x\in
- S_1} u(x,y_0)$, et le fait que $y_0$ soit la meilleure réponse
-possible de $2$ contre $x_0$, c'est-à-dire $u(x_0,y_0) = \min_{y\in
- S_2} u(x_0,y)$ (puisque $2$ cherche à maximiser $-u$, c'est-à-dire
-minimiser $u$). Comme on l'a expliqué dans la preuve, on a
-\[
-\max_{x\in S_1} \min_{y\in S_2} u(x,y)
-\geq \min_{y\in S_2} u(x_0,y) = u(x_0,y_0)
-= \max_{x\in S_1} u(x,y_0) \geq
-\min_{y\in S_2} \max_{x\in S_1} u(x,y)
-\]
-donc en fait il y a égalité partout : tout équilibre de Nash réalise
-la même valeur $u(x_0,y_0) = \max_{x\in S_1} \min_{y\in S_2} u(x,y) =
-\min_{y\in S_2} \max_{x\in S_1} u(x,y)$, qu'on appelle la
-\defin[valeur (d'un jeu à somme nulle)]{valeur} du jeu à somme nulle.
-
-On peut donc parler de \index{optimale (stratégie)}\defin{stratégie
- optimale} pour le joueur $1$, resp. $2$ pour désigner une composante
-$x_0$, resp. $y_0$, d'un équilibre de Nash, i.e., vérifiant
-$\min_{y\in S_2} u(x_0,y) = \max_{x\in S_1} \min_{y\in S_2} u(x,y)$,
-resp. $\max_{x\in S_1} u(x,y_0) = \min_{y\in S_2} \max_{x\in S_1}
-u(x,y)$, ces deux quantités étant égales à la valeur du jeu.
-
-Moralité : \emph{dans un jeu à somme nulle, un profil de stratégies
+\smallskip
+
+Considérons maintenant les applications du
+théorème \ref{theorem-minimax} dans le cadre de la théorie des jeux.
+Commençons par définir les concepts suivants :
+
+\begin{defn}\label{definition-zero-sum-game-value-and-optimum}
+Pour un jeu à somme nulle défini par une matrice de gains $u \colon
+A_1 \times A_2 \to \mathbb{R}$ :
+\begin{itemize}
+\item La \defin[valeur (d'un jeu à somme nulle)]{valeur} est la valeur
+ commune $\max_{x\in S_1} \min_{y\in S_2} u(x,y) = \min_{y\in S_2}
+ \max_{x\in S_1} u(x,y)$ où $S_1,S_2$ sont les simplexes de
+ stratégies mixtes des joueurs $1$ et $2$ respectivement (et $u(x,y)$
+ l'espérance de gain explicitée
+ en \ref{definition-mixed-strategy-gain}).
+\item Une stratégie mixte du joueur $1$ qui réalise $\max_{x\in S_1}
+ \min_{y\in S_2} u(x,y)$ (resp. une stratégie mixte du joueur $2$ qui
+ réalise $\min_{y\in S_2} \max_{x\in S_1} u(x,y)$) est dite
+ \index{optimale (stratégie)}\defin{stratégie optimale} pour le
+ joueur en question.
+\end{itemize}
+\end{defn}
+
+Le théorème \ref{theorem-minimax} (et, pour le dernier point, le
+corollaire \ref{symmetric-zero-sum-game}) a alors les conséquences
+suivantes :
+
+\begin{prop}\label{minimax-for-games} Pour un jeu à somme nulle défini
+par une matrice de gains $u \colon A_1 \times A_2 \to \mathbb{R}$ :
+\begin{itemize}
+\item[(i)] Un profil $(x_0,y_0) \in S_1\times S_2$ de stratégies
+ mixtes est un équilibre de Nash si et seulement si $x_0$ et $y_0$
+ sont chacun une stratégie optimale pour le joueur correspondant.
+\item[(ii)] Tous les équilibres de Nash ont la même valeur de gain
+ espéré, à savoir la valeur du jeu $v$ (pour le joueur 1).
+\item[(iii)] Une stratégie mixte $x_0 \in S_1$ du joueur $1$ est une
+ stratégie optimale pour celui-ci si et seulement si $u(x_0,b) \geq
+ v$ pour tout $b\in A_2$ (où $v$ désigne la valeur du jeu). Une
+ stratégie mixte $y_0 \in S_2$ du joueur $2$ est une stratégie
+ optimale pour celui-ci si et seulement si $u(a,y_0) \leq v$ pour
+ tout $a\in A_1$.
+\item[(iv)] L'ensemble $T_1$ des stratégies optimales du joueur $1$
+ est un convexe, de même que l'ensemble $T_2$ des stratégies
+ optimales du joueur $2$. (L'ensemble des équilibres de Nash est
+ donc aussi un convexe, à savoir $T_1 \times T_2$.)
+\item[(v)] Si le jeu est symétrique, c'est-à-dire $A_2 = A_1 =: A$ et
+ $u(b,a) = -u(a,b)$ (matrice de gains antisymétrique), alors la
+ valeur du jeu est nulle.
+\end{itemize}
+\end{prop}
+
+\begin{proof}
+(i) : On a vu au cours de la preuve de \ref{theorem-minimax} que si
+ $(x_0,y_0)$ est un équilibre de Nash, alors $\max_{x\in S_1}
+ \min_{y\in S_2} u(x,y) \geq \min_{y\in S_2} u(x_0,y) = u(x_0,y_0) =
+ \max_{x\in S_1} u(x,y_0) \geq \min_{y\in S_2} \max_{x\in S_1}
+ u(x,y)$, mais comme les deux termes extrêmes sont égaux, en fait
+ toutes ces inégalités sont des égalités, donc $x_0$ réalise bien le
+ maximum $\max_{x\in S_1} \min_{y\in S_2} u(x,y)$ et $y_0$ le minimum
+ $\min_{y\in S_2} \max_{x\in S_1} u(x,y)$, i.e., ils sont des
+ stratégies optimales. Réciproquement, si $x_*$ est une stratégie
+ optimale du joueur $1$ et $y_*$ du joueur $2$, on a vu au cours de
+ la même preuve qu'on a $\max_{x\in S_1} \min_{y\in S_2} u(x,y) =
+ \min_{y\in S_2} u(x_*,y) \leq u(x_*,y_*) \leq \max_{x\in S_1}
+ u(x,y_*) = \min_{y\in S_2} \max_{x\in S_1} u(x,y)$, et, de nouveau,
+ totues ces inégalités sont des égalités, donc $x_*$ est une
+ meilleure réponse possible pour le joueur $1$ contre $y_*$ et $y_*$
+ est une meilleure réponse possible pour le joueur $2$ contre $x_*$,
+ ce qui signifie que $(x_*,y_*)$ est un équilibre de Nash.
+
+Le point (ii) résulte de ce qui vient d'être dit.
+
+(iii) : Par définition, $x_0$ est optimale ssi $\min_{y\in S_2}
+u(x_0,y) = v$, ou, comme on ne peut pas faire plus que le maximum,
+$\min_{y\in S_2} u(x_0,y) \geq v$. Mais on a vu
+en \ref{affine-functions-take-extrema-at-boundary} que $\min_{y\in
+ S_2} u(x_0,y) = \min_{b\in A_2} u(x_0,b)$, c'est-à-dire que $x_0$
+est optimale si et seulement si $\min_{b\in A_2} u(x_0,b) \geq v$, ce
+qui revient bien à dire $u(x_0,b) \geq v$ pour tout $b$. Le cas de
+l'autre joueur est analogue.
+
+(iv) : Il résulte du point précédent que $T_1$ est l'intersections des
+ensembles $\{x \in S_1 : u(x,b)\geq v\}$ où $b$ parcourt $A_2$ : mais
+chacun de ces ensembles est convexe, donc leur intersection l'est
+aussi. Le cas de $T_2$ est analogue. La parenthèse est une redite du
+point (i).
+
+Le point (v) n'est qu'une redite du
+corollaire \ref{symmetric-zero-sum-game}.
+\end{proof}
+
+Répétons : \emph{dans un jeu à somme nulle, un profil de stratégies
est un équilibre de Nash si et seulement si chaque joueur joue une
- stratégie optimale} (l'ensemble des stratégies optimales étant
-défini pour chaque joueur indépendamment).
-
-Le corollaire \ref{symmetric-zero-sum-game} nous apprend (de façon peu
-surprenante) que si le jeu à somme nulle est \emph{symétrique} (ce qui
-signifie que $u$ est antisymétrique), alors la valeur du jeu est
-nulle.
-
-\thingy Dans le contexte ci-dessus, on peut légèrement reformuler le
-minimax : si on se rappelle (cf. \ref{stupid-remark-best-mixed-strategies})
-qu'une fonction affine sur un
- simplexe prend son maximum (ou son minimum) sur un des sommets du
-simplexe, cela signifie que, quel que soit $x\in S_1$ fixé, le minimum
-$\min_{y\in S_2} u(x,y)$ est en fait atteint sur une stratégie
-\emph{pure}, $\min_{y\in S_2} u(x,y) = \min_{b\in A_2} u(x,b)$ (avec
-$A_2$ l'ensemble des sommets de $S_2$, i.e., l'ensemble des stratégies
-pures du joueur $2$), et de même $\max_{x\in S_1} u(x,y) = \max_{a\in
- A_1} u(a,y)$ quel que soit $y \in S_2$. \emph{Ceci ne signifie pas}
-qu'il existe un équilibre de Nash en stratégies pures (penser à
-pierre-papier-ciseaux). Néanmoins, cela signifie que pour calculer la
-pire valeur possible $\min_{y\in S_2} u(x,y)$ d'une stratégie $x$ du
-joueur $1$, celui-ci peut ne considérer que les réponses en stratégies
-pures du joueur $2$.
-
-Si on appelle $v$ la valeur du jeu, l'ensemble des $x$ tels que
-$u(x,y) \geq v$ pour tout $y\in S_2$, c'est-à-dire l'ensemble des
-stratégies optimales pour le joueur $1$, coïncide donc avec l'ensemble
-des $x$ tels que $u(x,b) \geq v$ pour tout $b\in A_2$. En
-particulier, c'est un convexe compact dans $S_1$ (puisque chaque
-inégalité $u(x,b) \geq v$ définit un convexe compact dans $S_1$ vu que
-$x \mapsto u(x,b)$ est affine) : \emph{en moyennant deux stratégies
- optimales pour un joueur on obtient encore une telle stratégie}
-(notamment, l'ensemble des équilibres de Nash est un convexe de
-$S_1\times S_2$ — puisque c'est le produit du convexe des stratégies
-optimales pour le premier joueur par celui des stratégies optimales
-pour le second — affirmation qui n'est pas vraie en général pour des
-jeux qui ne sont pas à somme nulle).
+ stratégie optimale}, l'ensemble des stratégies optimales étant un
+convexe (défini pour chaque joueur indépendamment).
+
+L'ensemble des stratégies optimales d'un joueur donné est « très
+souvent » un singleton, ce qui fait qu'on parle parfois abusivement de
+« la » stratégie optimale.
+
+Reste maintenant à expliquer comment calculer les stratégies
+optimales :
\begin{algo}\label{zero-sum-games-by-linear-programming-algorithm}
Donnée une fonction $u\colon A_1 \times A_2 \to \mathbb{R}$ (avec
@@ -1609,6 +1934,9 @@ $\#A_1 + 1$ variables avec des contraintes de positivité sur $\#A_1$
d'entre elles, une contrainte d'égalité et $\#A_2$ inégalités affines.
\end{algo}
+(La correction de cet algorithme découle à peu près immédiatement du
+point (iii) ci-dessus.)
+
\thingy Pour ramener ce problème à un problème de programmation
linéaire en \emph{forme normale} (maximiser $\textbf{p} x$ sous les
contraintes $\textbf{M} x \leq \textbf{q}$ et $x\geq 0$), on sépare la
@@ -1619,7 +1947,7 @@ devient de maximiser $v_+ - v_-$ sous les contraintes
\[-\sum_{a\in A_1} x_a \leq -1\]
\[(\forall b\in A_2)\;v_+ - v_- - \sum_{a \in A_1} u(a,b)\, x_a \leq 0\]
Le problème dual (minimiser ${^{\mathrm{t}}\textbf{q}} y$ sous les
-contraintes ${^{\mathrm{t}}\textbf{M}} y \geq {^\mathrm{t}\textbf{q}}$
+contraintes ${^{\mathrm{t}}\textbf{M}} y \geq {^\mathrm{t}\textbf{p}}$
et $y\geq 0$) est alors de minimiser $w_+ - w_-$ sous les contraintes
\[w_+\geq 0,\; w_- \geq 0,\;\; (\forall b\in A_2)\;y_b \geq 0\]
\[\sum_{b\in A_2} y_b \geq 1\]
@@ -2833,8 +3161,8 @@ existe un cycle $x_0,\ldots,x_{n-1}$, on en déduit une suite infinie
en posant $x_i = x_{i\mod n}$) ; pour un graphe \emph{fini}, la
réciproque est vraie : en effet, s'il existe une suite infinie
$x_0,x_1,x_2,\ldots$ avec une arête de $x_i$ à $x_{i+1}$ pour
-chaque $i$, il doit exister $n$ tel que $x_n = x_0$, et on obtient
-alors un cycle $x_0,\ldots,x_{n-1}$. En général, cependant, les
+chaque $i$, il doit exister $p<q$ tels que $x_q = x_p$, et on obtient
+alors un cycle $x_p,\ldots,x_{q-1}$. En général, cependant, les
notions sont distinctes, l'exemple le plus évident (de graphe
acyclique mais mal fondé) étant sans doute celui de $\mathbb{N}$ dans
lequel on fait pointer une arête de $i$ à $i+1$ pour chaque $i$.
@@ -4037,6 +4365,10 @@ y$ pour $y\geq x$ et $x<y$ pour $y>x$.
Un ensemble partiellement ordonné est dit \defin[totalement ordonné (ensemble)]{totalement ordonné}
lorsque pour tous $x\neq y$ on a soit $x>y$ soit $y>x$.
+%% TODO: éclaircir le fait que dans ce qui suit, « bien-fondé » se
+%% comprend pour le graphe reliant $x$ à $y$ ssi $x>y$ (voir aussi la
+%% précédente occurrence du terme « bien-ordonné »).
+
Un ensemble totalement ordonné bien-fondé $W$ est dit
\defin[bien-ordonné (ensemble)]{bien-ordonné}. D'après \ref{well-founded-induction}, ceci
peut se reformuler de différentes façons :
@@ -4317,7 +4649,8 @@ toujours comparables (on a toujours $\beta<\alpha$ ou $\beta>\alpha$
ou $\beta=\alpha$), et il n'existe pas de suite infinie strictement
décroissante d'ordinaux.
-Autrement dit : dans tout ensemble d'ordinaux il y en a un plus petit.
+Autrement dit : dans tout ensemble non vide d'ordinaux il y en a un
+plus petit.
\end{thm}
\begin{proof}
Le théorème \ref{comparison-of-well-ordered-sets} signifie exactement
@@ -5052,7 +5385,7 @@ Soit $\alpha$ un ordinal. On appelle alors \defin{nimbre} associé
l'ensemble $\alpha+1$),
\item la relation d'arête (définissant le graphe) est $>$,
c'est-à-dire que les voisins sortants de $\beta\leq\alpha$ sont les
- ordinaux $\beta'<\alpha$, et
+ ordinaux $\beta'<\beta$, et
\item la position initiale est $\alpha$.
\end{itemize}
Autrement dit, il s'agit du jeu où, partant de l'ordinal $\beta =
@@ -5219,7 +5552,7 @@ L'ordinal $0$ est neutre pour $\oplus$.
Par induction sur $\alpha$, on prouve $\alpha \oplus 0 = \alpha$ : en
effet, $\alpha \oplus 0 = \mex \{\beta\oplus 0: \beta<\alpha\}$, et
par hypothèse d'induction ceci vaut $\mex \{\beta: \beta<\alpha\} =
-\mex \alpha = \alpha$.
+\alpha$.
\end{proof}
\begin{proof}[Seconde démonstration]
Cela résulte de l'observation que $\alpha\oplus 0 = \gr(*\alpha_1
@@ -5241,7 +5574,7 @@ ensemble contenant $\alpha_1\oplus\alpha_2'$, donc il ne peut pas lui
\begin{thm}\label{nim-sum-for-games-versus-ordinals}
Si $G_1,G_2$ sont deux jeux combinatoires impartiaux bien-fondés ayant
valeurs de Grundy respectivement $\alpha_1,\alpha_2$, alors la valeur
-de Grundy de $G_1\oplus G_2$ est $\alpha_2\oplus\alpha_2$.
+de Grundy de $G_1\oplus G_2$ est $\alpha_1\oplus\alpha_2$.
\end{thm}
\begin{proof}
On procède par induction bien-fondée sur les positions de $G_1\oplus
@@ -5942,7 +6275,7 @@ est le jeu combinatoire partisan bien-fondé dont
\alpha$,
\item la relation d'arête (définissant le graphe) est $>$,
c'est-à-dire que les voisins sortants de $\beta\leq\alpha$ sont les
- ordinaux $\beta'<\alpha$,
+ ordinaux $\beta'<\beta$,
\item l'arête $(\beta,\beta')$ est colorée en bleu si $\sigma(\beta')
= +$ et en rouge si $\sigma(\beta') = -$, et
\item la position initiale est $\alpha$.
@@ -6968,20 +7301,20 @@ jeu où les coups de Bob sont purement et simplement ignorés).
\smallbreak
(7) Soit $u \colon \{0,1\}^{\mathbb{N}} \to \mathbb{R}$ qui à
-$(x_0,x_1,x_2,\ldots)$ associe $\sum_{i=0} x_i 2^{-i}$ (le nombre réel
+$(x_0,x_1,x_2,\ldots)$ associe $\sum_{i=0} x_i 2^{-i-1}$ (le nombre réel
dont la représentation binaire est donnée par $0$ virgule la suite
des $x_i$). Vérifier que $u$ est continue et calculer la valeur du
jeu qu'elle définit (quelle est la stratégie optimale pour Alice et
pour Bob ?).
\begin{corrige}
-La fonction $u$ est continue car si $\varepsilon < 2^{-\ell-1}$
+La fonction $u$ est continue car si $\varepsilon < 2^{-\ell}$
alors la valeur $u(\dblunderline{x})$ est définie à $\varepsilon$ près
par la donnée des $\ell$ premiers termes de la
suite $\dblunderline{x}$. Il est évident qu'Alice a intérêt à ne jouer
que des $1$ (jouer autre chose ne ferait que diminuer son gain) et
Bob que des $0$. La valeur du jeu est donc $u(0,1,0,1,0,1,\ldots) =
- \frac{1}{2}$.
+ \frac{1}{3}$.
\end{corrige}
diff --git a/sample-2020qcm.tex b/sample-2020qcm.tex
new file mode 100644
index 0000000..4cf4b02
--- /dev/null
+++ b/sample-2020qcm.tex
@@ -0,0 +1,335 @@
+%% This is a LaTeX document. Hey, Emacs, -*- latex -*- , get it?
+\documentclass[12pt,a4paper]{article}
+\usepackage[francais]{babel}
+\usepackage[utf8]{inputenc}
+\usepackage[T1]{fontenc}
+%\usepackage{ucs}
+\usepackage{times}
+% A tribute to the worthy AMS:
+\usepackage{amsmath}
+\usepackage{amsfonts}
+\usepackage{amssymb}
+\usepackage{amsthm}
+%
+\usepackage{mathrsfs}
+\usepackage{wasysym}
+\usepackage{url}
+%
+\usepackage{graphics}
+\usepackage[usenames,dvipsnames]{xcolor}
+\usepackage{tikz}
+\usetikzlibrary{matrix,calc}
+\usepackage{hyperref}
+%
+%\externaldocument{notes-mitro206}[notes-mitro206.pdf]
+%
+\newenvironment{qcm}{\relax}{\relax}
+\newenvironment{qvar}{\relax}{\relax}
+\newcounter{quescnt}
+\newenvironment{question}%
+{\stepcounter{quescnt}\bigskip\noindent\textbf{Question~\arabic{quescnt}.}\par\nobreak}
+{\relax}
+\newcounter{answcnt}[quescnt]
+\newcommand\answer{%
+\stepcounter{answcnt}\smallskip\textbf{(\Alph{answcnt})}~}
+\let\rightanswer=\answer
+%
+\newcommand{\outnb}{\operatorname{outnb}}
+\newcommand{\downstr}{\operatorname{downstr}}
+\newcommand{\precs}{\operatorname{precs}}
+\newcommand{\mex}{\operatorname{mex}}
+\newcommand{\id}{\operatorname{id}}
+\newcommand{\limp}{\Longrightarrow}
+\newcommand{\gr}{\operatorname{gr}}
+\newcommand{\rk}{\operatorname{rk}}
+\newcommand{\fuzzy}{\mathrel{\|}}
+%
+\DeclareUnicodeCharacter{00A0}{~}
+%
+\DeclareMathSymbol{\tiret}{\mathord}{operators}{"7C}
+\DeclareMathSymbol{\traitdunion}{\mathord}{operators}{"2D}
+%
+\newif\ifcorrige
+\corrigetrue
+\newenvironment{corrige}%
+{\ifcorrige\relax\else\setbox0=\vbox\bgroup\fi%
+\smallbreak\noindent{\underbar{\textit{Corrigé.}}\quad}}
+{{\hbox{}\nobreak\hfill\checkmark}%
+\ifcorrige\par\smallbreak\else\egroup\par\fi}
+%
+%
+%
+\begin{document}
+\ifcorrige
+\title{MITRO206\\Échantillon de questions — Corrigé\\{\normalsize Théories des jeux}}
+\else
+\title{MITRO206\\Échantillon de questions\\{\normalsize Théories des jeux}}
+\fi
+\author{}
+\date{26 juin 2020}
+\maketitle
+
+\pretolerance=8000
+\tolerance=50000
+
+\vskip1truein\relax
+
+\noindent\textbf{Consignes.}
+
+Ce contrôle de connaissances est un QCM (questionnaire à choix
+multiples). Chaque question admet une unique réponse correcte. Les
+questions sont totalement indépendantes les unes des autres (mais
+certaines peuvent se ressembler). La sélection des questions et
+l'ordre ont été tirés aléatoirement et n'obéissent donc à aucune
+logique particulière.
+
+La réponse est attendue sous forme d'une liste de numéros de question
+suivie de la réponse proposée : par exemple, « \verb=1A 2B 4D= » pour
+signifier que la réponse proposée à la question 1 est (A), la réponse
+proposée à la question 2 est (B), et la réponse proposée à la
+question 4 est (D).
+
+Une réponse incorrecte sera (possiblement jusqu'à deux fois) plus
+fortement pénalisée qu'une absence de réponse : il est donc préférable
+de ne pas répondre à une question que de répondre aléatoirement.
+
+\medbreak
+
+Durée : 1h de 17h30 à 18h30
+
+\vfill
+
+%% \noindent
+%% Sujet généré pour : \texttt{\seedval}
+
+\medskip
+
+{\tiny\noindent
+\immediate\write18{sh ./vc > vcline.tex}
+Git: \input{vcline.tex}
+\immediate\write18{echo ' (stale)' >> vcline.tex}
+\par}
+
+\pagebreak
+
+\begin{qcm}
+
+\begin{question}
+
+Auquel ordinaux suivants est égal $2^{\omega^2}$ (lire : $2$ puissance
+$\omega^2$) ?
+
+\answer
+$\omega^{\omega^\omega}$
+
+\rightanswer
+$\omega^\omega$
+
+\answer
+$\omega^{\omega^2}$
+
+\answer
+$\omega^2$
+
+\answer
+$\omega$
+
+\end{question}
+
+\begin{corrige}
+On a $2^{\omega^2} = 2^{\omega\times\omega} = (2^\omega)^\omega =
+\omega^\omega$, donc réponse \textbf{(B)}.
+\end{corrige}
+
+
+%
+%
+%
+
+\begin{question}
+
+Considérons le jeu à somme nulle, symétrique, entre Alice et Bob, dont
+la matrice des gains est donnée par le tableau suivant (Alice choisit
+la ligne, Bob choisit la colonne, le tableau donne le gain d'Alice et
+le gain de Bob est l'opposé de la valeur indiquée) :
+
+\begin{center}
+\begin{tabular}{r|rrrrr}
+$\downarrow$Alice, Bob$\rightarrow$&U&V&W&X&Y\\\hline
+U&$0$&$+2$&$0$&$-2$&$+1$\\
+V&$-2$&$0$&$+1$&$+1$&$+2$\\
+W&$0$&$-1$&$0$&$+1$&$+1$\\
+X&$+2$&$-1$&$-1$&$0$&$+2$\\
+Y&$-1$&$-2$&$-1$&$-2$&$0$\\
+%% m = Matrix(QQ, 5, 5, [[0, 2, 0, -2, 1], [-2, 0, 1, 1, 2], [0, -1, 0, 1, 1], [2, -1, -1, 0, 2], [-1, -2, -1, -2, 0]])
+\end{tabular}
+\end{center}
+
+Laquelle des réponses suivantes est une stratégie optimale à ce jeu ?
+(Chaque réponse proposée est la liste des probabilités de jouer les
+options U,V,W,X,Y dans cet ordre.)
+
+\answer
+$(\frac{1}{3}, \frac{1}{3}, 0, \frac{1}{3}, 0)$
+
+\answer
+$(0, \frac{1}{2}, 0, \frac{1}{2}, 0)$
+
+\answer
+$(0, 0, 0, 0, 1)$
+
+\rightanswer
+$(\frac{1}{5}, \frac{2}{5}, 0, \frac{2}{5}, 0)$
+
+\end{question}
+
+\begin{corrige}
+Dans un jeu à somme nulle, les équilibres de Nash sont exactement les
+paires de stratégies optimales. Ici le jeu est symétrique, donc les
+stratégies optimales seront les mêmes pour les deux joueurs et la
+valeur $v$ du jeu sera $0$. Pour vérifier qu'une stratégie est
+optimale, il s'agit donc de vérifier que si les deux joueurs la joue,
+aucun ne peut faire mieux en jouant une stratégie pure différente. On
+calcule donc les combinaisons des lignes du tableau dont les
+coefficients sont donnés par les probabilités dans les différentes
+réponses, et la seule dont toutes les valeurs sont $\geq v$ est
+$(\frac{1}{5}, \frac{2}{5}, 0, \frac{2}{5}, 0)$.
+Réponse \textbf{(D)}.
+\end{corrige}
+
+
+%
+%
+%
+
+\begin{question}
+
+On considère le jeu combinatoire (impartial, à information parfaite)
+associé au graphe orienté acyclique représenté ci-dessous, la position
+de départ étant notée $s$ :
+
+\begin{center}
+\begin{tikzpicture}[>=stealth,thick,text width=5bp,text height=5bp,text depth=0bp]
+\node (n00) at (0bp,0bp) [draw,circle] {};
+\node (n01) at (40bp,0bp) [draw,circle] {};
+\node (n02) at (80bp,0bp) [draw,circle] {};
+\node (n10) at (0bp,-40bp) [draw,circle] {};
+\node (n11) at (40bp,-40bp) [draw,circle] {};
+\node (n12) at (80bp,-40bp) [draw,circle] {};
+\node (n20) at (0bp,-80bp) [draw,circle] {};
+\node (n21) at (40bp,-80bp) [draw,circle] {};
+\node (n22) at (80bp,-80bp) [draw,circle] {$s$};
+\draw[->] (n01) -- (n00); \draw[->] (n02) -- (n01);
+\draw[->] (n11) -- (n10); \draw[->] (n12) -- (n11);
+\draw[->] (n21) -- (n20); \draw[->] (n22) -- (n21);
+\draw[->] (n10) -- (n00); \draw[->] (n20) -- (n10);
+\draw[->] (n11) -- (n01); \draw[->] (n21) -- (n11);
+\draw[->] (n12) -- (n02); \draw[->] (n22) -- (n12);
+\end{tikzpicture}
+\end{center}
+
+Quelle est la valeur de Grundy du jeu (i.e., celle de la
+position $s$) ?
+
+\answer
+$4$
+
+\answer
+$2$
+
+\rightanswer
+$0$
+
+\answer
+$1$
+
+\answer
+$3$
+
+\end{question}
+
+\begin{corrige}
+On calcule les valeurs de Grundy de proche en proche (c'est-à-dire par
+induction bien-fondée), la valeur de Grundy d'une position étant le
+mex (= plus petite valeur exclue) des valeurs de Grundy de ses voisins
+sortants. On trouve
+
+\begin{center}
+\begin{tikzpicture}[>=stealth,thick,text width=5bp,text height=5bp,text depth=0bp]
+\node (n00) at (0bp,0bp) [draw,circle] {$0$};
+\node (n01) at (40bp,0bp) [draw,circle] {$1$};
+\node (n02) at (80bp,0bp) [draw,circle] {$0$};
+\node (n10) at (0bp,-40bp) [draw,circle] {$1$};
+\node (n11) at (40bp,-40bp) [draw,circle] {$0$};
+\node (n12) at (80bp,-40bp) [draw,circle] {$1$};
+\node (n20) at (0bp,-80bp) [draw,circle] {$0$};
+\node (n21) at (40bp,-80bp) [draw,circle] {$1$};
+\node (n22) at (80bp,-80bp) [draw,circle] {$0$};
+\draw[->] (n01) -- (n00); \draw[->] (n02) -- (n01);
+\draw[->] (n11) -- (n10); \draw[->] (n12) -- (n11);
+\draw[->] (n21) -- (n20); \draw[->] (n22) -- (n21);
+\draw[->] (n10) -- (n00); \draw[->] (n20) -- (n10);
+\draw[->] (n11) -- (n01); \draw[->] (n21) -- (n11);
+\draw[->] (n12) -- (n02); \draw[->] (n22) -- (n12);
+\end{tikzpicture}
+\end{center}
+
+La réponse correcte est donc \textbf{(C)}.
+\end{corrige}
+
+
+%
+%
+%
+
+\begin{question}
+
+Alice et Bob jouent au jeu de type Gale-Stewart suivant : chacun à son
+tour choisit un chiffre binaire (soit $0$ soit $1$ : Alice choisit
+$a_1$ puis Bob choisit $a_2$ puis Alice choisit $a_3$ et ainsi de
+suite). Au bout d'un nombre infini de tours, on considère le nombre
+réel $x$ entre $0$ et $1$ dont l'écriture binaire fractionnaire est
+formée de ces chiffres (c'est-à-dire $x = \sum_{i=1}^{+\infty}
+a_i\,2^{-i}$, ou $0{.}a_1a_2a_3\ldots$ en écriture binaire) : si $x <
+\frac{1}{3}$, Alice gagne, tandis que si $x \geq \frac{1}{3}$, Bob
+gagne. (À toutes fins utiles, on rappelle que $\frac{1}{3}$ s'écrit
+$0{.}01010101\ldots$ en binaire.) Que pensez-vous de ce jeu ?
+
+\answer
+un joueur a une stratégie gagnante, mais il est impossible de savoir
+lequel
+
+\rightanswer
+Bob a une stratégie gagnante
+
+\answer
+Alice a une stratégie gagnante
+
+\answer
+aucun joueur n'a de stratégie gagnante
+
+\end{question}
+
+\begin{corrige}
+On peut faire remarquer que $[\frac{1}{3};1]$ est fermé (ou plus
+correctement, que l'ensemble des représentations binaires des réels
+de $[\frac{1}{3};1]$ est fermé pour la topologie produit) pour se
+convaincre qu'il existe forcément une stratégie gagnante pour au moins
+un joueur, mais en fait peu importe : Alice va manifestement jouer $0$
+à tous les coups et Bob jouer $1$ à tous les coups (on peut tracer le
+début de l'arbre binaire infini des possibilités pour y voir plus
+clair), si bien qu'on va tomber sur $\frac{1}{3}$ et Bob a une
+stratégie gagnante. Réponse \textbf{(B)}.
+\end{corrige}
+
+
+%
+%
+%
+
+\end{qcm}
+
+%
+%
+%
+\end{document}